Anda di halaman 1dari 104

FINAL BLOK MUSKULOSKELETAL GRANULOSIT | 2022/2023

1. Seseorang/ andi saja :). Mengalami kecelakaan dan terjadi fraktur terbuka di bagian eks
atas sehingga kelihatan tulang humeri dan juga hilang hulit (lose skin) serta terjadi
kerusankan pada fx teersebut kurang lebih 10cm, berikut termasuk dalam kategori pada
klasifikasi open fx apa….?
a. I -Broken <1 cm
b. II -> broken >1cm
c. IIIA
d. IIIB -> lose skin, bose exp, and >10cm broken fragment bone
e. IIIC -> arteries are very damaged

2. Laki-laki, 65 tahun, datang dengan keluhan nyeri sendi di kedua pergelangan tangan dan
jari-jari dikedua tangannya. Pasien juga mengeluhkan kekakuan di pagi hari. Pada
pemeriksaan fisik didapatkan tanda vital baik, tampak swan neck deformity pada jari jari
tangan kanan dan kiri. Kriteria remisi pada pasien rheumatoid arthritis adalah ?
a. Nyeri sendi berkurang
b. Kekakuan sendi pagi hari >30 menit
c. Erosi berkurang pada pemeriksaan foto rontgen
d. LED < 20 mm/jam I untuk laki-laki dan <30 mm untuk wanita
e. Faktor rheumatoid sudah negative

3. Apakah tipe kolagen yang disintesis pada tulang rawan?


a. Kolagen tipe I
b. Kolagen tipe II
c. Kolagen tipe V
d. Kolagen tipe VII
e. Kolagen tipe VI

4. Seorang Laki-laki berusia 25 tahun datang ke dokter dengan keluhan demam tinggi
dengan bengkak dan nyeri tungkai bawah kiri. Riwayat kecelakaan, yakni pasien ditabrak
motor sekitar 1 minggu yang lalu dan menyebabkan luka pada tungkai kiri. Pada
pemeriksaan laboratorium didapatkan leukositosis, lalu dikirim ke Bagian Radiologi
untuk pemeriksaan radiologi. Apa pemeriksaan foto radiologi apa yang tepat untuk
kasus ini ? *
A. Foto cruris kiri posisi obliq
B. Foto cruris kiri posisi lateral
C. Foto cruris kiri posisi Antero posterior (AP)
D. Foto cruris kiri posisi Antero posterior (AP) + obliq (tdk ada lekositosis)
E. Foto cruris kiri posisi Antero posterior (AP) + lateral (ad lekositosis)

5. Seorang laki” berusia 65 tahun datang ke klinik dengan keluahan nyeri sendi kedua
pergelangan tangan dan kedua jari jari tangannya. Pasien juga mengeluh kekakuan
dipagi hari . Pada pemfis didapatkan tanda vital baik. Tampak swan neck deformity pada
jari jari tangan kanan dan kiri, apa pemeriksaan yg dianjurkan pada kasus di atas
a. Asto titer
b. Biopsy
c. Ana test
d. anti ds-DNA
e. Anti ccp

6. Manakah dibawah ini yang termasuk sel jaringan ikat yang tidak bergerak? *
a.       Sel mast
b.      Sel monosit
c.       Sel lemak
d.      Sel limfosit
e.       Sel mesenkimal

7. Seorang laki-laki umur 67 tahun datang ke klinik dengan keluhan nyeri dan bengkak
serta kemerahan pada ibu jari kaki dan tidak bisa memakai sepatu. Tidak ada riwayat
demam sebelumnya. Hasil pemeriksaan laboratorium ditemukan peningkatan asam urat
dan tidak ada leukositosis. Pemeriksaan foto radiologi kaki apa yang dilakukan untuk
menegakkan diagnosis ?
A. Posisi foto anterior posterior + obliq
B. Posisi foto anterior posterior + lateral
C. Posisi foto posterior anterior + obliq
D. Posisi foto anterior posterior
E. Posisi lateral

8. Seorang laki-laki berusia 55 tahun datang dengan keluhan bengkak dan nyeri pada ibu
jari kaki kanan. Riwayat dari pesta malam sebelumnya dan mengkonsumsi makanan
tinggi purin? Apa tatalaksana yang paling tepat buat pasien tersebut? (diagnosa gout
akut)
A. Memberikan allopurinol (untuk menurunkan gout bertopus)
B. Memberikan NSAID
C. Memberikan allopurinol dan NSAID secara bersamaan
D. Memberikan NSAID dan Kolkisin
E. Memberikan Kolkisin dan Allopurinol (tidak boleh diberi bersamaan pada gout
akut)

9. Ifa seorang mahasiswa yang cantik dan baik hati, datang ke kampus kesiangan, akibatnya
menyebrang tanpa melihat lalu lintas yang rame, sehingga mengalami kecelakaan
akibatnya patah pada lengan kanan bawah..........
A. Ulna dan radius
B. Clavikula
C. Skapula
D. Humeri (kynyaa)
10. seorang pria berusia 25 tahun mengalami kecelakaan saat bersky atau jets kie. Orang itu
mengeluh bahwa dadanya merasa sakit, gangguan apa yang terjadi pada laki-laki
tersebut…
A.Fraktur radialis distal
B.Fraktur pergelangan tangan
C.Fraktur tulang rusuk
D.Fraktur veterbral
E.Fraktur Femur

11. Anak perempuan berusia 15 tahun diantar oleh orang tuanya ke klinik dengan batuk,
nafsu makan kurang, keringat malam, berat badan menurun. Saat ini datang dengan
keluhan nyeri punggung. Pada pemeriksaan terdengar bunyi pernapasan yang
mencurigakan adanya tanda-tanda bronchopneumonia kedua paru. Dan pada
pemeriksaan tulang belakang ada terlihat scoliosis dan gibbus pada torakolumbal. Apa
diagnosis yang paling tepat untuk kasus ini?
A. Ewing’s tumor
B. Spondylitis TBC
C. Avasculer necrosis
D. Ankilosing spondilitis
E. Rheumatoid arthritis

12. Bu Sarah merupakan lansia berumur 50 tahun, ia mengalami perubahan pada tulang
belakang, miring bahu, nyeri punggung, postur membungkuk dan perut buncit, kelainan
apakah yang dialami ibu Sarah?
a. Dislokasi
b. Tumor
c. Osteoporosis
d. Skleroderma

13. Trauma otot dan tendon disebabkan peregangan otot dan tendon yang melebihi batas
normal, merupakan definisi dari?
a. Atrofi
b. Skleroderma: penyakit autoimun yang menyerang jaringan ikat, sehingga jaringan
ikat menebal dan mengeras
c. Rabdomiosis: disebabkan oleh kerusakan dan kematian jaringan otot rangka
d. Sprain: meregang ligament dan robek tendonnya ( yang kena tulang dan sendi)
e. Strain: Cedera yang terjadi karena regangan berlebihan atau terjadi robekan
pada otot maupun tendon (penghubung tulang dan otot)1

14. Diagnosis banding dari SLE...


Jawaban: sindrom sjogre

15. Berkas (bundel) serat kolagen yang melekatkan otot ke tulang, serat kolagen dianggap
sebagai jaringan ikat dan dihasilkan oleh sel-sel fibroblas 
Dari keterangan diatas merupakan pengertian dari………. 
a. Otot Jantung 
b. Otot Polos 
c. Tulang 
d. Tendon

16. Trauma pada otot atau tendon disebabkan oleh peradangan otot atau tendon yang
melebihi batas normal,ini merupakan definisi dari..
A. Atrofi: kondisi dimana otot tampak jauh mengecil dibandingkan ukuran normal.

B. Sprain: Cedera yang terjadi karena regangan berlebihan atau terjadi robekan
pada ligamen (penghubung antar tulang)1

C. Strain: Cedera yang terjadi karena regangan berlebihan atau terjadi robekan pada
otot maupun tendon (penghubung tulang dan otot)1

D. pilih yang ini kalau yang ditanyakan tentang peradangan

17. Os humerus saat masa embrio merupakan tulang yang berasal dari proses ?
Jawabannya :Endokondral Ossification

18. Manakah dibawah ini yang termasuk sel jaringan ikat yang tidak bergerak?
A. Sel mast
B. Sel monosit
C. Sel lemak
D. Sel limfosit
E. Sel mesenkimal

19. Manakah dibawah ini yang tidak termasuk sekresi fibroblast? *


A. Proteoglikan
B. Glukosamine
C. Glikosaminoglikan
D. Fibronektin
E. Kolagen

20. Seorang laki-laki 65 tahun,datang dengan keluhan nyeri sendi di kedua pergelangan
tangan dan jari-jari dikedua tangannya. Pasien juga mengeluhkan kekakuan di pagi hari.
Pada pemeriksaan fisik didapatkan tanda vital baik, tampak swan neck deformity pada
jari-jari tangan kanan dan kiri. Apa diagnosis yang paling mungkin untuk kasus ini? *
A. Osteoarthritis
B. Gout Arthritis
C. Rhematoid arthritis
D. Lupus Arthritis
E. Sindrom-Sjogren
21. Apakah nama nervus yang menginnervasi otot-otot quadriceps femoris?
A. N. Tibialis
B. N. Peroneus communis
C. N. Femoralis
D. N. Cutaneus femoris lateralis
E. N. Obturatorius

22. Laki- laki 45 tahun datang dengan keluhan nyeri dan bengkak pada ibu jari kaki kanan
yang dialami 5 jam yang lalu. Riwayat nyeri di ibu jari kaki kanan sejak setahun lalu,
terutama pada malam hari dan jika cuaca dingin. Pemeriksaan fisis : dolor (+), kalor (+)
bulge sign (-), balloon sign (-). Tes laboratorium yang dapat dilakukan untuk memantau
terjadinya komplikasi adalah: (diagnosa gout)
A. Ureum-Kreatinin
B. Tes darah rutin
C. Urin Rutin
D. GOT/GPT
E. GDS

23. Seorang laki-laki berusia 45 tahun datang ke puskesmas dengan keluhan nyeri pasca
amputasi below knee sinistra akibat ganggren diabetik. Skor VAS pasien adalah 7.
Farmakoterapi anti nyeri yang dapat diberikan berdasarkan keluhan pasien adalah?
A. Paracetamol tab
B. Ibuprofen tab
C. Natrium diklofenak tab
D. Meloxicam tab
E. Kodein tab + NSAID

24. Seorang perempuan berusia 52 tahun datang ke klinik dengan keluhan nyeri mendadak
pada sendi kaki sejak 2 hari lalu, disertai dengan bengkak. Riwayat pasien suka minum
alkohol. Pada pemeriksaan fisis didapatkan status lokalis: tampak hiperemis, hangat,
bengkak pada metatarsal phalang 2 dextra. Pemeriksaan laboratorium didapatkan asam
urat 10 mg/dL. Apa tatalaksana yang tepat pada pasien ini? (diagnosa gout akut)
A. Asam mefenamat
B. Allopurinol
C. Natrium diclofenac
D. Kolkisin
E. Meloxicam

25. Seorang perempuan berusia 35 tahun datang dengan keluhan nyeri pada sendi-sendi jari
kedua tangannya. Nyeri terutama dirasakan pada pagi hari disertai keluhan terasa kaku
yang lamanya lebih dari 30 menit. Pemeriksaan fisik didapatkan sendi-sendi bengkak,
eritema, teraba panas, dan nyeri saat digerakkan. Apa farmakoterapi yang tepat
sebelum pasien dirujuk ke ahli reumatologi?
A. Colchisine
B. Metotrexate
C. Glucosamine
D. Sulfasalazine
E. Natrium diclofenac

26. Seorang laki-laki berusia 46 th datang ke praktek dokter dengan keluhan nyeri sendi jari
kaki sejak 1 minggu yang lalu. Nyeri dirasakan mendadak disertai muncul benjolan di
sendi jari kaki yang awalnya terasa nyeri dan merah. Pasien suka makan jeroan dan
kacang-kacangan. Pada Pemeriksaan fisis didapatkan tekanan darah 110/80 mmHg, nadi
84 x/menit, suhu 36.5 C, frekuensi napas 20 x/mnt, tampak benjolan pada sendi MTP-1,
merah (-), nyeri tekan (-). Hasil pemeriksaan asam urat serum 7.8 mg/dL. Apa
patofisiologi terjadinya kasus ini? *
A. Osteofit
B. Terbentuknya pannus
C. Densitas tulang yang berkurang
D. Akumulasi Kristal monosodium urat (MSU)
E. Autoantibodi

27. Laki- laki 45 tahun datang dengan keluhan nyeri dan bengkak pada ibu jari kaki kanan
yang dialami 5 jam yang lalu. Riwayat nyeri di ibu jari kaki kanan sejak setahun lalu,
terutama pada malam hari dan jika cuaca dingin. Pemeriksaan fisis : dolor (nyeri) (+),
kalor (panas) (+) bulge sign (-), balloon sign (-). Hasil pemeriksaan laboratorium apakah
yang dapat menunjang untuk menegakkan diagnosis kasus di atas? (diagnosa gout akut)
A. Laju Endap Darah meningkat
B. Rheumatoid Factor positif
C. Hiperurisemia
D. A dan B benar
E. A dan C benar

28. Apakah nama nervus yang menginnervasi otot-otot hamstring (m.biceps femoris,
m.semitendonosus, m.semimembranosus) ? *
A. N. Ischiadicus
B. N. Iliohypogastricus
C. N. Ilioinguinalis
D. N. Genitofemoralis 
E. N. Gluteus superior

29. Seorang Laki-laki berusia 25 tahun datang ke dokter dengan keluhan demam tinggi
dengan bengkak dan nyeri tungkai bawah kiri. Riwayat kecelakaan, yakni pasien ditabrak
motor sekitar 1 minggu yang lalu dan menyebabkan luka pada tungkai kiri. Pada
pemeriksaan laboratorium didapatkan leukositosis, lalu dikirim ke Bagian Radiologi
untuk pemeriksaan radiologi. Hasil pemeriksaan radiologis ternyata ditemukan destruksi
osteolitik 1/3 distal dengan soft tissue swelling. Maka diagnosis radiologisnya adalah? *
A. Osteomielitis kronis
B. Osteomielitis akut 
C. Avaskuler nekrosis
D. Spondilitis TBC

30. Sel penyusun tulang yang bukan berasal dari tulang (leukosit) yang berfungsi sebagai
reabsorbsi
A. Osteoby
B. Osteoclasts
C. Osteosit
D. .......
E .......

31. Laki – laki umur 42 tahun, datang dengan keluhan nyeri hebat pada lutut kanan, dialami
penderita sejak pagi tadi ketika bangun sholat subuh. Awalnya hanya terasa pegal –
pegal sedikit hangat pada perabaan. Riwayat trauma pada lutut kanan disangkal oleh
penderita. Jika hasil pemeriksaan kadar asam urat darah saat ini adalah 5,2 mg/dl, ada
riwayat keluhan nyeri yang terjadi secara tiba – tiba namun pada sendi tallo-kluralis kiri
sekitar 6 bulan lalu. Maka diagnosis pada scenario adalah ( diagnosis gout )
A. Osteorthtritis genu kanan
B. Artritis septic
C. Artritis gout intermitten
D. Artritis gout akut
E. Hiperukemia asamtomatik

32. Seorang wanita 55 tahun, setelah dilakukan test ANA, hasilnya adalah (+). Terlihat ruam
malar dan ruam discoid. Maka diagnosis adalah
A. RA
B. Osteomyelitis akut
C. SLE
D. Osteoporosis
E. OA

33. Laki-laki 54 datang dengan keluhan utama nyeri pada siku kiri simetris yg sudah
berlangsung sejak 1 tahun lalu, awalnya dimulai dengan nyeri pada sendi-sendi dijarinya
dan pergelangan tangan. Keluhan dirasakan terutama di pagi hari. tidak ada demam,
riwayat tidak pernah jatuh. Apakah tes laboratorium serologi yang dapat diminta untuk
mendukung diagnosis pasien tersebut?
A. RF
B. LE test
C. Tes ANA
D. ASTO
E. Anti-DNA
34. Bagian dari tulang panjang yang menentukan kemampuan tulang tersebut masih dapat
bertumbuh atau tidak, adalah…
A. Epiphysis
B. Diaphysis
C. Articular cartilage
D. Epiphyseal plate

35. Kekuatan kontraksi otot dapat ditingkatkan dengan cara di bawah ini
A. Menaikkan voltage melebihi voltage maksimalnya
B. Meningkatkan jumlah motor unit yang terlibat
C. Menambah jumlah glikogen
D. Menambah frekuensi stimulasi
E. Menambah suplai glukosa

36. Sel yang berperan dalam resorpsi tulang adalah


a. osteoclast
b. osteoblast
c. osteosit
d. osteoprogenik cell
e. stromal sel (sel stromal adalah sel yang membentuk jaringan
dalam tubuh dan memiliki banyak pembuluh darah)

37. Disebut apa bagian yang mendekati batang tubuh …. 


a. Anterior 
b. Superficial 
c. Superior 
d. Proxima

38. Seorang laki-laki umur 60 tahun dating ke poliklinik dengan keluhan nyeri dan bengkak
serta kemerahan pada ibu jari kaki dan tidak bisa memakai sepatu. Hasil pemeriksaan
laboratorium ditemukan peningkatan asam urat. Datang ke dokter untuk melakukan
pemeriksaan Radiologi. Tidak demam, dan tidak menunjukkan adanya elevasi atau
peningkatan jumlah leukosit di dalam darah. Apakah Tanda radiologis yang khas dari
kasus diatas?
a. Ada reaksi perosteal
b. Osteofit
c. Punch out + abses
d. Punch out + tophus
e. kalsifikasi

39. Jika terjadi penurunan ca di dalam darah, maka tubuh akan berusaha menaikkan ca di
dalam darah dengan cara ?
A.meningkatkan ekspresi ca dalam ginjal
B.meningkatkan pelepasan hormon PTH
C. Menurunkan calstriol
D. meningkatkan pelepasan calsitonin

40. Contoh tulang yang berubah tulang rawan hyalin menjadi tulang keras adalah ?
A. Os Femur
B. Os mandibular
C. Os clavicula
D. Os temporal

41. Manakah dibawah ini yang tidak sesuai untuk sel mast? *
A. Granula mengandung heparin
B. Granula diwarnai secara ortochromatik
C. Mengikat immunoglobulin E
D. Banyak dalam lamina propria
E. Berperan dalam reaksi alergi

42. regio tenar hipotenar dan tenar terdapat pada di regio:


a. regio fascialis,
b. regio plantaris,
c. regio abdomen,
d. regio palmaris,
e. regio orbicularis

43. Reina, gadis kurus berlari sekuat tenaga menuju ke arah kantin karena laparnya
setelah terdengar suara bel istirahat. Namun ketika sampai pada jalan menurun, Reina
seperti kehilangan kendali dan terjatuh. Kemudian Reifa yang mengetahui hal itu,
kemudian menolong Reina untuk bangun. Namun Reina meringis kesakitan dan tidak
dapat menegakkan kakinya, setelah dilihat ternyata salah satu tulang lututnya
menonjol dari dalam kulitnya. Menurut hipotesa sementara, apa yang terjadi pada
lutut Reina ?
a. Terjadi dislokasi pada tulang fibula Reina
b. Terjadi dislokasi pada tulang tibia Reina
c. Terjadi dislokasi pada tulang patela Reina
d. Terjadi dislokasi pada tulang patela dn tulang fibula Reina

44. Pasien Prayitno menderita suatu penyakit, dengan gejala tidak bisa menarik sudut
mulutnya ke arah lateral. Menurut kasus singkat di atas, hipotesa sementara dari
pasien Prayitno otot manakah serta syaraf keberapakah yang mengalami gangguan?
a. Otot risorius dan syaraf fasial (VII)
b. Otot orbikularis mulut dan syaraf fasial (VII)
c. Otot businator dan syaraf fasial (VII)
d. Otot businator dan syaraf fasial (VI)
45. Karena hari libur kuliyah faruq diajak ayahnya untuk berlibur kepantai. faruq bersama
ayahnya berlibur dengan mengendarai sepeda motor. sesampai di desa kulonprogo
ban sepeda yang tumpangi faruq dan ayahnya pecah. ayah faruqpun tidak bisa
mengendalikan sepeda montornya karena terlalu kencang dan akhirnya keduanya pun
terjatuh. ayah faruq selamat dari kejadian tersebut tanpa ada luka. tetapi faruq
mengalami luka sobek dibagian paha atas.
Sistem apakah yang terganggu pada pada luka firman?
a. Sistem kardiovaskuler
b. Sistem integumen (kulit)
c. Sistem saraf
d. Sistem

46. Didesa sukabumi tinggalah seorang anak yang bernama Dani dia sangat suka dengan
yang namanya mangga sampai suatu seketika dia pergi kekebun mangga milik
ayahnya. Dia langsung memanjat pohon mangga tersebut dan tanpa disadari ranting
yang diinjaknya patah sehingga dia terjatuh dan mengalami patah tulang di lengan
bagian bawah. Sistem apakah yang terganggu bila dilihat dari kondisi Dani?
a. Sistem saraf
b. Sistem integumen
c. Sistem muskuloskeletal
d. a,b,c benar

47. Trauma pada suatu otot atau tendon biasanya disebabkan oleh peregangan otot atau
tendon yang melebihi batas normal. Ini merupakan definisi dari……
a. Sprain
b. Atrofi
c. Rabdomiolosis
d. Strain

48. Berkas (bundel) serat kolagen yang melekatkan otot ke tulang, serat kolagen dianggap
sebagai jaringan ikat dan dihasilkan oleh sel-sel fibroblas
Dari keterangan diatas merupakan pengertian dari……….
a. Otot Jantung
b. Otot Polos
c. Tulang
d. Tendon

49. Di desa sumbersari ada perkebunan kelapa dan pada saat itu kelapa sudah mulai
masuk pada masa panen. Pak Komar selaku pemilik kebun itu menyuruh seorang
warga yang bernama Sigit untuk memanen buah kelapa. Ketika dia sudah di atas dia
terpeleset dan jatuh sampai mengalami patah tulang di paha bagian atas. Dilihat dari
kondisinya sendi apakah yang terganggu pada Sigit?
a. Sendi fibrus
b. Sendi gerak bebas
c. Sendi sinovial
d. Sendi tulanag rawan
50. Melekat pada tulang dan berperan pada pergerakan tubuh¸serabut multinukleus lurik
dan berkumpul dalam bundel paralel¸di persyarafi oleh syaraf motorik…..
a. Otot rangka
b. Otot polos
c. Otot jantung
d. Otot skelet

51. Seorang anak bernama Budi berumur 13 tahun mengendarai sebuah motor di jalan
raya, saat waktu yang bersamaan datanglah sebuah mobil yang melaju cepat dan
menabrak anak tersebut. Anak tersebut mengalami luka robek di paha yang
mengeluarkan darah dan terlihat tulang anak tersebut patah muncul ke luar. Apakah
nama gangguan yang dialami anak tersebut ?
a. Fraktur terbuka
b. Fraktur tertutup
c. Nyeri sendi
d. Osteoporosis

52. Dio adalah seorang anak berusia 7 tahun. Pada hari minggu kemarin, ia bermain sepak
bola bersama teman-teman sebayanya di lapangan kelurahan sebelah rumahnya. Dio
mendaji penjaga gawang dari permainan tersebut. Tanpa di duga, sesuatu hal yang
tidak di inginkan terjadi. Pada saat Dio ingin menangkap bola yang datang ke arah
gawangnya ia tersandung oleh gundukan rumput yang ada di sekitar gawang sehingga
menyebabkan ia jatuh dan bagian dengkul kaki kirinya membengkak. Hal ini
menyebabkan kaki sebelah kiri Dio tidak bisa digunakan untuk berjalan. Dan apabila ia
ingin berjalan ia harus menjinjit karena sakit. Dari kasus tersebut, dapat diketahui
bahwa jenis sendi apakah yang tidak dapat digerakkan oleh?
a. Sendi luncur
b. Sendi peluru
c. Sendi putar
d. Sendi engsel

53. Ifa adalah Seorang mahasiswi yang rajin dan juga baik hati. Namun pada kemarin ia
bangun kesiangan, akibatnya ia berangkat kuliah dengan terburu-buru. Tanpa
menghiraukan keadaan lalu lintas yang ramai, akhirnya ia mengalami kecelakaan yang
menyebabkan lengan kanannya patah, sehingga ia tidak dapat melakukan pekerjaan
yang seharusnya dapat ia lakukan seperti menulis. Tulang apakah yang mengalami
cedera pada kasis Ifa?
a. Scapula
b. Clavicula
c. Ulna dan hasta
d. Femur

54. Bu Sarah adalah seorang wanita lansia dengan umur 50 tahun, beliau mengalami
perubahan pada tulang belakangnya yaitu miring bahu, nyeri punggung, postur
membungkuk, perut buncit kelainan apa yang dialami oleh bu Sarah ?
a. Skleroderma
b. Tumor
c. Dislokasi
d. Osteoporosis

55. Manakah yang termasuk struktur lapisan tulang…


a. Humerus ->tulang
b. Periosteum
c. Osteoblas -> ini sel
d. b dan c benar -> salah

56. Andi adalah sekolah di SMA kelas 2 yang mengalami kecelakaan lalu lintas. Sampai di
bawa ke rumah sakit, dia masih tidak sadarkan diri. Setelah itu dia mencoba
menggerakkan lengan bawahnya, namun tidak bisa. Menurut kejadian di atas,
mengapa lengan bawahnya tidak dapat digerakkan ?
a. Karena terjadi gangguan pada otot triseps lengan atas
b. Karena terjadi gangguan pada otot brakialis
c. Karena terjadi gangguan pada otot ankoneus
d. Karena terjadi gangguan pada otot maseter

57. Ketika firman pulang dari menonton pertandingan sepak bola. Firman melihat ada
pemuda mabuk yang sedang cekcok mulut dengan warga setempat. Dan taklama
kemudian kedua orang tersebut saling bertengkar dengan saling tonjok dan saling
memukul. Firman yang melihat kejadian tersebut langsung melerai tetapi kedua
pemuda itu tidak minghiraukan firman. Pemuda yang mabuk tersebut malah balik
memukul pipi sebelah kanan firman. firmanpun langsung terjatuh ketanah dengan
luka bengkak membiru dan kepalanya sedikit mengeluarkan darah karena terbentur
batu waktu jatuh ke tanah.  
Sistem apakah yang terganggu bila dilihat dari kondisi firman? 
a. Sistem saraf 
b. Sistem kardiovaskuler 
c. Sistem muskulus 
d. a, b, dan c salah

58. Jam menunjukkan pukul 12.30 siang Ari bersama kawan-kawannya pulang
sekolah. sesampai dipertenhgahan jalan ari melihat ada seorang penjual es
cendol. Karena cuacanya yang sangat panas dan tenggorokan ari terasa kering.
Ari bersama kawan-kawannya langsung menghampiri penjual es cendol
tersebut dan membeli es bersama kawan-kawannya. karena cuaca yang sangat
panas ari langsung meminum es tersebut dan tanpa sadar ari berada di bawah
pohon kelapa yang buahnnya sudah banyak yang kering. Aripun kejatuhan buah
kelapa yang sudah kering tersebut yang mengenai bahu kirinya. yang
mengakibatkan bahu kirinya memar dan tangannya sulit digerakkan.
Sendi apakah yang terganggu bila dilihat dari kasus tersebut?
a. Sendi putar
b. Sendi enggsel
c. Sendi peluru
d. Sendi pelana

59. Dikota malang ada pertandingan sepak bola antara arema melawan persebaya
dimana pada saat Marcio menggiring bola dia dilanggara oleh Andik begitu
keras pada lutut bagian bawah sehingga dia tidak dapat berjalan. Sendi apakah
yang terganggu bila dilihat dari kondisi Marcio?
a. Sendi putar
b. Sendi engsel
c. Sendi pelana
d. Sendi peluru
60. Informasi mengenai perubahan pada otot dapat diperoleh dari
A. Muscle spindel
B. Muscle golgi
C. Organ tendon
D. Myosin

61. Luka pada paha bagian atas akan menganggu sistem


A. Sistem nerfus
B. Sistem gastrointestinal
C. Sistem respirasi
D. Sistem musculo
E. Sistem integument

62. Manifestasi klinis terbanyak pada paru pada penderita SLE


A. Pneumonia
B. Reumatoud Arthritis
C. Effusi Pleura
D. .......
E ........

63. Eksekusi pergerakan motorik dilakukan oleh… dengan mengirim sinyal untuk dilakukan
kontraksi
A.Korteks Premotor
B.kortes suplemen
C.korteks motoric primer
D. Korteks prefrontal

64. Fast fiber memiliki respon yang cepat dibandingkan dengan tipe slow fiber. Ini
dimungkinkan karena?
A. Serat otot yang kecil
B. Banyak pembuluh darah
C. Memiliki mithocondria yang banyak
D. Mengandalkan enzim glikolisis untuk medapatkan ATP
E. Banyak myoglobin

65. Seorang laki laki umur 38 tahun datang dengan keluhan pada panggul setlah sejam
tabrakan. Tttv normal. Pemfis ada deformitas hip flexi, adduksi ,internal rotation.
Sendi yang alami gangguan?
Jawaban : posterior hip dislocation

66. soal kasus jatuh, yang Gold standar MRI


jawaban : xray

67. Pada SLE mempengaruhi hematologi: anemia hemolitik, leukopenia, atau


trombositopenia (hafal tulisan merah)

68. Kaku mayat disebabkan oleh Kaku mayat terjadi akibat hilangnya adenosin trifosfat
(ATP) dari otot-otot tubuh manusia.Kelenturan otot dapat terjadi selama masih
terdapat ATP yang menyebabkan serabut aktin dan miosin tetap lentur. Bila cadangan
glikogen dalam otot habis, maka energi tidak terbentuk lagi, aktin dan miosin
menggumpal dan otot menjadi kaku

Jawaban : hilangnya ATP

69. Tes yang diperlukan untuk …… meniscus adalah

a. Valgus test :Tes valgus melibatkan menempatkan kaki ke ekstensi, dengan satu


tangan ditempatkan sebagai poros di lutut.
b. Lachman Test:Tes ini dilakukan untuk memeriksa cedera atau robekan pada ACL.
ACL menghubungkan dua dari tiga tulang yang membentuk sendi lutut
c. Tes Apley :Tes Applay kompresi dan traksi. Tujuan : Untuk mengetahui apakah
ada gangguan pada meniskus dan ligamen collateral lateral dan medial pada
lutut. 

70. Tulang bersifat lentur karena mengandung

Jawaban : kolagen

71. Derajat SLE


A. Ringan
B. Sedang
C. Berat
D. Mengancam Jiwa

Penjelasan SLE:
Derajat Berat Ringannya Penyakit
 
Kriteria untuk dikatakan SLE ringan adalah:
1. Secara klinis tenang
2. Tidak terdapat tanda atau gejala yang mengancam nyawa
3. Fungsi organ normal atau stabil, yaitu: ginjal, paru, jantung, gastrointestinal, susunan
saraf pusat, sendi, hematologi dan kulit.
Contoh: SLE dengan manifestasi arthritis dan kulit.
 
Penyakit SLE dengan tingkat keparahan sedang manakala ditemukan:
1. Nefritis ringan sampai sedang ( Lupus nefritis kelas I dan II)
2. Trombositopenia (trombosit 20-50×103/mm3)
3. Serositis mayor
 
Penyakit SLE berat atau mengancam nyawa apabila ditemukan keadaan sebagaimana
tercantum di bawah ini, yaitu:
1. Jantung: endokarditis Libman-Sacks, vaskulitis arteri koronaria, miokarditis, tamponade
jantung, hipertensi maligna
2. Paru-paru: hipertensi pulmonal, perdarahan paru, pneumonitis, emboli paru, infark paru,
fibrosis interstisial, shrinking lung.
3. Gastrointestinal: pankreatitis, vaskulitis mesenterika.
4. Ginjal: nefritis proliferatif dan atau membranous.
5. Kulit: vaskulitis berat, ruam difus disertai ulkus atau melepuh (blister).
6. Neurologi: kejang, acute confusional state, koma, stroke, mielopati transversa,
mononeuritis, polineuritis, neuritis optik, psikosis, sindroma demielinasi.
7. Hematologi: anemia hemolitik, neutropenia (leukosit <1.000/mm3), trombositopenia (<
20.000/mm3) , purpura trombotik trombositopenia, trombosis vena atau arteri.
 

72. apabila kadar Ca meningkat dalam darah melebihi batas normal lalu ingin di kembalikan
ke batas normal bagaimana cara yang harus di lakukan?
jawaban : produksi kalsitonin

73. Pada kontraksi otot dengan adanya Ca2+ yang di keluarkan oleh SR makaakan terjadi
adanya?
Jawaban : maka troponin akan berubah posisi lalu menarik tropomyosin menjauh
dari bagian aktif aktin
Catatan :
- fungsi troponin c hanya mengikat kalsium
- fungsi troponin I melekatkan troponin ke aktin
- fungsi troponin T melekatkan troponin ke tropomyosin
-

74. Kadar CA dalam darah menjalankan proses fisiologi dalam tubuh seperti dibawah ini,
kecuali
Catatan peran Ca2+:
1. Transmisi inplus saraf
2. Kontraksi otot
3. Koagulasi darah
4. Respon sel (secondarry massenger)
5. Sekresi saraf dan kelenjar
6. Pembelahan sel

75. Tulang juga berfungsi sebagai cadangan nutrisi, ini disebabkan karena tulang juga
memiliki fungsi
a. tempat melekat organ
b. penyimpanan Mineral dan lemak
c. pembentukan sel darah merah
d. pergerakan
e. pelindung beberapa organ

76. Tulang yg dri lunak ke keras, tulang apa, dan proses apa na maksud?
Kalau :
- endokondral ossifikasi bersal dari kartilago hyalin membentuk tulang
keras. (membentuk tulang Panjang cth femur,humerus dkk)
- kalau intramembanous ossifikasi berasal dari jaringan fibrous yang
membentuk tulang keras. ( membentuk tulang pendek cth
tengkorak,mandibula dan klafikula)
Jawaban : os femur

77. kontraksi otot (Yg CA, Ca2+, Komponen AP, ATP, dll)

78. ujung tulang sendi terdapat


jawaban : epifisis

79. sakit dikedua bahu, simetris? Jawaban PMR

80. tipe sendi apa di lengan?


A. Hinge
B. Ball and Socket
Jawaban : pivot atau sendi putar
CATATAN : tengkorak immofebel
Bahu (ball dan socket)
Lutut (engsel)
Lengan (pivot atau putar)
vertebra (laiding)

81. Perempuan usia 60tahun nyeri bahu kaku pagi hari apa diagnosisnya
Jawaban PMR

82. otot hipotenar dan tenar termasuk regio


jawaban: PALMAR

83. Yang termasuk tulang yang berasal dari intermembranous adalah :


Jawabannya : Os.Mandibula

84. Peran Troponin C


Jawabannya : mengikat calcium

85. Laki laki usia 27 tahun nyeri sendi panggul akibat tabrakan mobil. Pemeriksaan fisik:
posissi sendi oanggu kiri refleksi,adduksi dan internal rotasi. Pemeriksaan x ray : kaput
femur berada di atas acetabulum. Diagnosis ?
A. Fraktur neck femur
B. Fraktur intertrokantal
C. Dislokasi posterior sendi panggul
Anterior
Inferior

86. Pemeriksaan fisik: dislokasi,bengkak lengan kiri, tidak ada luka. Pemeriksan x ray: fraktur
radius distal dan dispalaced dorsal komliksi awal yg memungkinkan ?
A. Infeks
B. Sindrom kompratemen
C. Kekakuan ngeri sendi
D. Neunulion
E. Mulinion

87. Seorang perempuan usia 18 th datang dengan keluhan nyeri di tangan, rambut rontok,
dan demam. Peristiwa ini sudah dirasakan 3 bulan lamanya. Tidak ada riwayat penyakit
sebelumnya. Tidak ada riwayat berobat sebelumnya. Dari hasil pemeriksaan fisis
didapatkan malar rash (+) discoidrash (+). Penatalaksaan pertama yang diberikan pada
kasus diatas berdasarkan derajat penyakitnya adalah....
a. .... intervena
b.
c.aza...
d.
nda jelas opsionnya kesian, nda setahu apa mau se jawab. Tapi kalau ada ini di
opsion pilih itu nanti,
1. NSAID
2. KOrtikostreroid dosis rendah

88. Kadar Ca dalam darah Penting di pertahankan normal untuk proses-proses fisiologi
tubuh seperti dibawah ini, kecuali:
a. Sebagai neurotransmitters saraf
b. Sebagai second messeger dalam sel
c. Membantu proses pembekuan darah
d. Membantu terjadinya kontraksi otot

89. Seorang laki-laki berusia 25 Tahun, datang ke klinik dengan keluhan bengkak dan nyeri
pada lutut kanan setelah terjatuh saat bermain sepak bola. Lutut tidak dirasakan
bergoyang apabila berjalan. Tidak ada riwayat penyakit metabolik dan hipertensi.
Apakah Test khusus pada lutut yang dapat dilakukan pada kasus di atas?
a) Mc. Murray, Lachman test, Posterior drawer Test, Varus test
b) Thomas test, Mc. Murray, Lachman test, Posterior drawer Test
c) Mc. Murray, Lachman test, Posterior drawer Test, Thompson test
d) Thomas test, Mc. Murray, Lachman test, Posterior drawer Test, Finkelstein test
e) Lachman test, Posterior drawer Test, Thompson test, Thomas test

90. Salah satu penyebab terjadinya kelelahan otot adalah


a. Semua benar
b. Aktifnya motor neuron
c. Terputusnya suplai darah pada kontraksi yang lama
d. Mitochondria berhenti bekerja
e. Meningkatnya cadangan glukosa
91. apakah nama nervus percabangan N.Ichiadicus yang berjalan di lateral os fibula ?
a. N. Cutaneus femoris lateralis
b. N. Femoralis
c. N Obturatorius
d. N. Tibialis
e. N. Peroneus communis
92. Achondroplasi terjadi akibat ?
a. Terhambatnya proses ossifikasi
b. Kelainan akibat kekurangan vit.D -> penyakit osteomlasia
c. Terhambatnya perubahan fibroblast jadi kartilago ->sama kya opsi E
d. Hilangnya gen kartilago hyaline reseptor -> bukan hilang tpi mutase
e. Tidak terjadi proses intramembranous ossification
93. Seorang anak laki-laki berusia 34 tahun datang ke IGD rumah sakit dengan keluhan sakit
di pergelangan tangan kiri yang disebabkan oleh jatuh dari sepeda motor. Pasien jatuh
ke sebelah kiri dengan posisi tangan menumpu pada aspal. Pada pemeriksaan fisis
didapatkan deformitas, bengkak pada pergelangan tangan kiri, dan tidak terdapat luka.
Pada pemeriksaan Xray menunjukkan fraktur radius distal dengan displaced dorsal.
Apakah komplikasi awal yang mungkin terjadi bila pasien tidak ditangani segera?
a. Nonunion
b. Kekakuan pada sendi ->kon
c. Malunion
d. Sindrom kompartemen -> kondisi awal
e. Infeksi
Untuk soal ini gezzz saya masih bimbang antara 2 jawaban diatas jadi kembali lagi ke
analisisnya kleann yahh hehehehe. Untuk saat ini saya lebih pro ke D. Kalau ada yang tahu
jawabannya tolong pc kag.
---astrid.

Catatan :
- Nonunion adalah komplikasi dari patah tulang dimana tidak terjadi
penyambungan tulang atau proses penyembuhan patah tulang terhenti
sama sekali.

- Malunion adalah suatu keadaan tulang patah yang telah mengalami


penyatuan dengan fragmen fraktur berada dalam posisi tidak normal (posisi
buruk).

- Sindrom kompartemen adalah kondisi ketika terjadinya peningkatan tekanan


di bagian tubuh yang tertutup (kompartemen) yang memotong suplai darah
ke otot dan saraf. Kondisi ini umumnya disebabkan oleh perdarahan dan
hematoma (kumpulan darah di luar pembuluh darah) di sekitar tulang yang
patah.

94. Pada ujung tulang dari sebuah sendi terdapat...yang penting untuk mengurangi friksi
pada saat terjadi pergerakan sendi
a. Epiphysis
b. Kartilago
c. Pembuluh darah
d. Diaphysis
e. Sponge bone
95. andi yakni sekolah di Sekolah Menengan Atas kelas 2 yang mengalami kecelakaan
kemudian lintas. Sampai di bawa ke rumah akit, dia masih tidak sadarkan diri. Setelah itu
dia mencoba menggerakkan lengan bawahnya, namun tidak bisa. Menurut msiden di
atas, mengapa lengan bawahnya tidak sanggup digerakkan?
a. karena terjadi gangguan pada otot maseter
b. karena berjadi gangguan pada otot brakialis
c. karena berjadi gangguan pada otot ankoneus
d. karena terjadi gangguan pada otot ankoneus dan maseter
e. Karena terjadi gangguan pada otot triseps lengan atas
96. Seorang perempuan berusia 18 tahun datang dengan nyeri sendi pada daerah tangan,
rambut rontok dan sering merasa demam. Hal ini dirasakan sejak 3 bulan terakhir. Tidak
ada riwayat penyakit sebelumnya. Tidak ada riwayat pengobatan sebelulmnya. Pada
pemeriksaan fisis didapatkan anemis (+), malar rash (+), dan discoid rash (+).
Berdasarkan derajat penyakit, maka kondisi di atas termasuk..
a. ringan
b. berat
c. sedang
d. mengancam nyawa
e. tanpa gejala
97. Bajuri memanjat pohon Rambutan untuk mengambil Rambutan yang sudah memerah di
depan halaman rumahnya. Namun sehabis hingga pada puncak, Bajuri menginjak batang
yang lemah, kemudian jatuh dengan posisi tengkurap. Setelah ditolong oleh ayahnya,
ternyata pergelangan tangannya tidak sanggup digerakkan. Hipotes sementara apa yang
terjadi pada Bajuri ?
a. Tulang karpus dengan tulang radius dan tulang ulna mengalami dislokasi
b. tulang karpus mengalami dislokasi
c. Tulang radius dengan tulang karpus mengalami dislokasi
d. Tulang radius dengan tulang karpus mengalami dislokasi
e. Tulang radius dengan tulang ulna mengalami dislokasi
98. Seorang perempuan berusia 60 tahun, datang dengan keluhan nyeri dan kaku pada
kedua bahu, hal ini utamanya dirasakan pada pagi hari. Tidak ada riwayat penyakit
sebelumnya. Pada pemeriksaan fisis didapatkan dalam batas normal. Pada pemeriksaan
laboratorium didapatkan ANA, RF, dan anti CCP negative. Tatalaksana awal pada pasien
adalah.
A. Kortikosteroid dosis rendah
B. fisioterapi
C. paracetamol
D. Kortikosteroid dosis tinggi
E. NSAID
Catatan : ini diagnosisnya toh PMR, nah kalau tatalaksana dari PMR itu diberikan
kortikosteroid dosis rendah, contohnya PREDNISON.

99. Molekul ATP dibutuhkan untuk kontraksi pada saat ?


a. tropomyosin menjauh dari aktin
b. aktin menarik myosin
c. troponin menarik tropomyosin
d. melepaskan myosin dari aktin
e. kepala myosin menarik aktin
100. Jembatan silang terjadi saat sel otot mulai berkontraksi dimana myosin akan
berikatan dengan ?
a. Troponin t
b. tropomyosin
c. albumin
d. aktin
e. troponin I
101. lon yang penting dalam pembentukan aksi potential dipermukaan otot akibat
terlepasnya acetylchcoline adalah:
a. Ca2+
b. CI-
c. K+
d. H+
e. Na+
102. Laki laki berusia 25 tahun datang ke puskesmas dengan keluhan nyeri lutut kanan
dialami sejak 2 bular disertai adanya sensasi "pop" pada lutut kanan. 2 bulan lalu pasien
pasien riwayat terjatuhsaat bermain dengan betis kanan terasa sedikit berputar. Pada
pemeriksaan fisis ditemukan Anterior drawer test posi negatif. Apakah pemeriksaan fisis
yang khas lain pada kasus ini adalah
a. Posterior Drawer test
b. Posterolateral Drawer test
c. Lachmann Test
d. Mc Murray test
e. Valgus stress test

Jadi pasien ini dicurigai cedera ACL ( Anterior crusiatum ligament )

Catatan :
- Tes mcmuray: digunakan untuk mengevalusai individu untuk robekan pada
meniscus lutut.

- Lachman Test: merupakan tes yang dilakukan untuk menilai instabilitas gerak sendi
lutut dengan menilai gerakan abnormal ke arah depan dari tibia.

- Thomas Test: digunakan untuk menyingkirkan kontraktur fleksi pinggul dan


sindrom psoas

- Posterior drawer test: digunakan dalam penilaian klinis awal untuk dugaan
pecahnya ligamen cruciatum di lutut. Pasien harus terlentang dengan pinggul
tertekuk hingga 45 derajat, lutut ditekuk hingga 90 derajat, dan kaki rata di atas meja
- Thompson Test: digunakan dalam pemeriksaan ekstremitas bawah untuk menguji
pecahnya tendon Achilles. Pasien berbaring telungkup dengan kaki menggantung di
tepi tempat tidur

103. Seorang perempuan berusia 18 tahun datang dengan nyeri sendi pada daerah
tangan, rambut rontok dan sering merasa demam. Hal ini dirasakan sejak 3 bulan
terakhir. Tidak ada riwayat penyakit sebelumnya. Tidak ada riwayat pengobatan
sebelulmnya. Pada pemeriksaan fisis didapatkan anemis (+), malar rash (+), dan discoid
rash (+). Diagnose yang paling mungkin adalah.
a. Fibromyalgia
b. Polimyalgia rheumatica
c. Rheumatoid arthritis
d. Systemic Lupus eritomatosus
e. Osteoarhtritis shoulder
104. Seorang perempuan berusia 60 tahun, datang dengan keluhan nyeri dan kaku pada
kedua bahu, hal ini utamanya dirasakan pada pagi hari. Tidak ada riwayat penyakit
sebelumnya. Pada pemeriksaan fisis didapatkan dalam batas normal. Pada pemeriksaan
laboratorium didapatkan ANA, RF, dan anti CCP negatif. Otot-otot rotator cuff yang
berperan dalam pergerakan struktur sendi di atas adalah?
a. M, Supraspinatus, M.infraspinatus, M.teres minor, M.subscapularis
b. M, Supraspinatus, M.infraspinatus, M.teres mayor, M.abductor longus
c. M, Supraspinatus, M.infraspinatus, M.teres minor, M.biceps brachii caput
longum
d. M, Supraspinatus, M.infraspinatus, M.teres minor, M.biceps brachii caput brevis
e. M, Supraspinatus, M.infraspinatus, M.teres mayor, M.subscapularis

Catatan : Otot – otot rotator cuff itu HANYA disusun oleh 4 otot nahh gezzz. Dan itumi
jawabannya diatas...

105. Seorang perempuan berusia 18 tahun datang dengan nyeri sendi pada daerah
tangan, rambut rontok dan serin Hal ini dirasakan sejak 3 bulan terakhir. Tidak ada
riwayat penyakit sebelumnya. Tidak ada riwayat pengobatan Pada pemeriksaan fisis
didapatkan anemis (+), malar rash (+), dan discoid rash (+). Patofisiologi yang mendasan
adalah .
a. toleransi imun yang meningkat
b. hipoaktivitas sel B dan sel T
c. Produksi sitokin pro inflamasi menurun
d. infeksi virus akut menyebabkan hiperaktivitas imun
e. gangguan pembersihan sel apoptosis
106. Seorang perempuan berusia 60 tahun, datang dengan keluhan nyeri dan kaku pada
kedua bahu, hal ini utamanya pada pagi hari. Tidak ada riwayat penyakit sebelumnya.
Pada pemeriksaan fisis didapatkan dalam batas normal. pemeriksaan laboratorium
didapatkan ANA, RF, dan anti CCP negative. Nyeri yang dirasakan pada pasien ini adalah
a. nyeri kanker
b. nyeri mekanik
c. nyeri inflamasi
d. nyeri nosiseptif
e. nyeri infeksi
107. Seorang perempuan berusia 25 tahun dibawa oleh keluarganya ke UGD RS dengan
penurunan kesadaran. Pasien sebelumnya mengalami kejang selama 15 menit.
Sebelumnya pasien sering mengeluh tidak tahan cahaya dan nyeri pada sendi tangan.
Pada pemeriksaan fisis didapatkan GCS E1M3V4, anemis (+), malar rash (+). Pada
pemeriksaan laboratorium didapatkan Hb 5,0 g/dl dan trombosit 10.000. Berdasarkan
kondisi di atas, maka system yang diserang pada kondisi ini adalah ...
a. gastrointestinal dan hematologi
b. kulit dan paru-paru
c. paru-paru dan gastrointestinal
d. hematologi dan neurologi
e. neurologi dan gastrointestinal
108. Seorang perempuan berusia 45 tahun, diagnosis suspek RA dan diberikan
prednisone sebagai bridging therapy untuk meredakan keluhan nyeri. Manakah
pernyataan yang benar mengenai prednisone?
a. Tidak memiliki efek pada metabolisme karbohidrat, protein dan lipid
b. Tidak memiliki efek imunosupresive
c. Merupakan anti inflamasi non steroid
d. Tidak memiliki efek permissive pada katekolamin
e. Menggangu axis hipotalamus-pituitari-adrenal bila diberikan pada dosis besar
dan berhenti secara mendadak
109. Seorang laki-laki 65 tahun,datang dengan keluhan nyeri sendi di kedua pergelangan
tangan dan jari-jari dikedua tangannya. Pasien juga mengeluhkan kekakuan di pagi hari.
Pada pemeriksaan fisik didapatkan tanda vital baik, tampak swan neck deformity pada
jari-jari tangan kanan dan kiri. Apa diagnosis yang paling mungkin untuk kasus ini?
a. Sindrom Sjogren
b. Gout arthritis
c. Osteoarthritis
d. Lupus arthritis
e. rheumatoid arthritis
catatan sedikit dari saya, ada itu soal tentang DIAGNOSIS BANDING DARI SLE, jawabannya
itu sindrom sjorgen nah gezzz… tapi kalau ANA TESNYA POSITIF jawabannya itu
FIBROMIALGIA

OKAYY SEKIAN DAN TERIMA KASIH, SELAMAT BELAJAR


Kalau ada jawaban salah, tolong dibantu perbaiki, bisa pc kag
----Astrid.

Terima kasih juga bantuannya teman teman yg lain

Ortopedi & Traumatologi (21/01/2022)

1. Seorang wanita 45th datang ke poli bedah RS UMM dengan jalan terbungkuk
disertai kaki pincang, keluhan nyeri lutut kanan yg hilang timbul sejak 3 bln yg
lalu terutama pada saat istirahat dan disertai kekakuan sendi. Pada pemeriksaan
didapatkan pembengkakan pada lutut kanan, atrofi pada betis kanan, dengan
pergerakan sendi yg terbatas. Pada pemeriksaan radiologi didapatkan
penyempitan celah sendi, sklerosis subchondral, pembentukan kista
subchondral, dan osteofit. Dari temuan klinis diatas, diagnosa kerja yg mungkin
adalah
a. Osteoarthritis
b. Myositis
c. RA
d. Dislokasi genu
e. Disuse atropi otot quadricep femur
2. Seorang laki-laki berusia 25th datang ke IGD RS dengan keluhan nyeri pada
tungkai bawah kiri, pada pemeriksaan fisik ditemukan adanya kulit mengkilat
disertai bulla multiple. Riwayat KLL 8 jam yg lalu. Dari pemeriksaan xray
ditemukan adanya fraktur segmental tibia fibula. Apakah tatalaksana yg tepat
pada pasien tersebut

a. Pemberian IV analgetik
b. Pemberian IV antibiotik
c. Penyedotan bulla
d. Fasciotomy
e. Debridement luka
3. Bayi laki-laki berusia 18bln datang ke PKM dibawa oleh orangtuanya dengan
keluhan kedua kaki bengkok. Dialami sejak lahir. Pemeriksaan vital sign dbn.
Pada pemeriksaan fisis pada kedua kaki ditemukan deformitas berupa cavus
ada midfoot, adductus pada forefoot, varus, dan equinus pada hint foot seperti
gambar dibawah. Pemeriksaan motorik dan sensorik dbn. Tatalaksana yg tepat
pada kasus

a. Kocher casting
b. Boot casting
c. Serial casting ponseti
d. Hip spica casting
e. Long leg casting
4. Seorang anak laki-laki berusia 18bln datang ke RS dengan keluhan tungkai
kanan lebih pendek dari sisi sebelah kiri. Riwayat jatuh dan demam tidak ada.
Pada pemeriksaan fisis ditemukan galleazzi sign ada, pergerakan abduksi sisi
kanan terbatas dibanding sisi yg kiri. Pada pemeriksaan xray tampak pada
gambar dibawah. Apakah dx yg tepat pada kasus

a. Neglected fraktur collum femur


b. Fraktur femur dextra
c. Leg calve perthes disease
d. DDH
e. Hip septic arthritis
5. Seorang laki-laki berusia 34th datang ke IGD RS dengan kaluhan sakitdi
pergelangan tangan kiri disebabkan oleh jatuh dari motor. Pasien jatuh ke
sebelah kiri dengan posisi tangan menunpu pada aspal. Pada pemeriksaan fisis
didapatkan deformitas dan bengkak pada pergelangan tangan kiri. Pada
pemeriksaan xray menunjukkan fraktur radius distal dengan displaced
dorsal. Apa dx yg tepat
a. Colles fracture
b. Smith fracture
c. Montegia fracture
d. Galeazzi fracture
e. Barton fracture
6. Anak laki-laki berusi 9th datang dengan keluhan tidak dapat berjalan dialami
sejak 1 minggu terakhir. Pasien mengeluh nyeri panggul kiri. Selama seminggu
pasien kadang merasakan demam tinggi. Pada pemeriksaan vital sign
didapatkan TD 110/70mmHg, Nadi 110x/mnt, Suhu 38.9oC. Pada pemeriksaan
fisis panggul kiri didapatkan dalam posisi flexi hip, abduksi, dan external rotasi.
ROM panggul kiri terbatas nyeri dan hangat perabaan. Pada pemeriksaan lab
ditemukan leukosit 30.000/mm3. Pada pemeriksaan MRI ditemukan kesan
penumpukan cairan di persendian panggul kiri. Apakah dx yg mungkin pada
kasus

a. Acute osteomyelitis
b. Chronic osteomyelitis
c. Dislokasi hip
d. Septic arthritis
e. Infeksi periprosthetic
7. Seorang anak laki-laki berusia 34th datang ke IGD RS dengan keluhan sakit di
pergelangan tangan kiri yang disebabkan oleh jatuh dari motor. Pasien jatuh ke
sebelah kiri posisi tangan menumpu pada aspal. Pada pemeriksaan didapatkan
deformitas, bengkak pada pergelangan, tangan kiri, dan tidak terdapat luka.
Pada pemeriksaan xray menunjukkan fraktur radius distal dengan displaced
dorsal. Apakah komplikasi awal yg mungkin terjadi apabila tidak ditangani
segera
a. Mal union
b. Non union
c. Infeksi
d. Sindrom kompartemen
e. Kekakuan pada sendi
8. Seorang anak laki-laki berusia 34th datang ke IGD RS dengan keluhan sakit di
pergelangan tangan kiri yang disebabkan oleh jatuh dari motor. Pasien jatuh ke
sebelah kiri posisi tangan menumpu pada aspal. Pada pemeriksaan didapatkan
deformitas, bengkak pada pergelangan, tangan kiri, dan tidak terdapat luka.
Pada pemeriksaan xray menunjukkan fraktur radius distal dengan displaced
dorsal. Apakah tx lanjutan yg tepat pada kasus
a. Pemasangan spalak
b. Elevasi lengan atas
c. Pemasangan gips
d. Dilakukan reduksi tertutup dan fiksasi internal
e. Dilakukan reduksi terbuka dan fiksasi internal
9. Laki-laki berusia 27th masuk IGD RS dengan keluhan nyeri pada daerah sendi
panggul akibat tabrakan mobil. Pada pemeriksaan fisik ditemukan posisi sendi
panggul kiri fleksi, adduksi, dan internal rotasi. Dari pemeriksaan xray
tampak caput femur berada diatas acetabulum, tidak tampak pecahan tulang.
Apakah kemungkinan dx pasien
a. Fraktur intertrokanter
b. Dislokasi anterior sendi panggul
c. Fraktur neck femur
d. Dislokasi posterior sendi panggul
e. Dislokasi anterior sendi panggul
10. Laki-laki berusia 27th masuk IGD RS dengan keluhan nyeri pada daerah sendi
panggul akibat tabrakan mobil. Pada pemeriksaan fisik ditemukan posisi sendi
panggul kiri fleksi, adduksi, dan internal rotasi. Dari pemeriksaan xray tampak
caput femur berada diatas acetabulum, tidak tampak pecahan tulang. Apakah
komplikasi yg dapat terjadi bila tidak ditangani segera
a. Osteoarthritis
b. Osteonecrosis
c. Septic arthritis
d. Mal union
e. Non union
11. Laki-laki berusi 25th datang ke PKM dengan keluhan nyeri lutut kanan dialami
sejak 2 bulan lalu. Nyeri kadang disertai adanya sensasi “pop” pada lutut kanan.
2 bulan lalu pasien riwayat terjatuh saat bermain futsal saat bermain dengan
betis kanan terasa sedikit berputar. Pada pemeriksaan fisik ditemukan anterior
drawer test positif, Mcmurray test negatif. Apakah pemeriksaan fisik khas lain
pada kasus ini
a. Posterior drawer test
b. Posterolateral drawer test
c. Mc murray test
d. Lachmann test
e. Valgus stress test
12. Laki-laki berusi 25th datang ke PKM dengan keluhan nyeri lutut kanan dialami
sejak 2 bulan lalu. Nyeri kadang disertai adanya sensasi “pop” pada lutut kanan.
2 bulan lalu pasien riwayat terjatuh saat bermain futsal saat bermain dengan
betis kanan terasa sedikit berputar. Pada pemeriksaan fisik ditemukan anterior
drawer test positif, Mcmurray test negatif. Apakah pemeriksaan penunjang yg
menjadi gold standar menegakkan dx kasus ini
a. USG
b. MRI
c. CT scan
d. Xray knee
e. Bone scan
13. Laki-laki berusi 25th datang ke PKM dengan keluhan nyeri lutut kanan dialami
sejak 2 bulan lalu. Nyeri kadang disertai adanya sensasi “pop” pada lutut kanan.
Pasien riwayat 3 bulan lalu salah mendarat ketika melakukan jump smash saat
bermain badminton. Pada pemeriksaan fisik ditemukan anterior drawer test
positif, Mcmurray test positif. Cedera lain yg mungkin terjadi menyertai kasus ini
dimana pada pemeriksaan fisis ditemukan Mcmurray test positif adalah
a. Fracture distal femur
b. Fracture proximal tibia
c. Meniscus injury
d. LCL injury
e. MCL injury
14. Laki-laki berusi 25th datang ke PKM dengan keluhan nyeri lutut kanan dialami
sejak 2 bulan lalu. Nyeri kadang disertai adanya sensasi “pop” pada lutut kanan.
2 bulan lalu pasien riwayat terjatuh saat bermain futsal saat bermain dengan
betis kanan terasa sedikit berputar. Pada pemeriksaan fisik ditemukan anterior
drawer test positif, Mcmurray test negatif. Untuk dapat kembali bermain
sepakbola apakah tx yg tepat untuk pasien
a. RICE
b. Analgetik
c. Pemasangan gips
d. Injeksi asam hyaluronic
e. Rekonstruksi ACL
15. Laki-laki berusia 28th datang ke PKM dengan keluhan nyeri lutut kanan dan
kadang lutut terasa terkunci dialami sejak 2 bulan lalu. Pasien riwayat terjatuh
saat bermain sepakbola dengan betis kanan terasa sedikit berputar. Pada
pemeriksaan fisis ditemukan anterior dan posterior drawer test negatif. Apakah
pemeriksaan untuk mengetahui cedera meniscus
a. Posterior drawer test
b. Posterolateral drawer test
c. Thesaly test
d. Lachmann test
e. Valgus stress test
16. Anak laki-laki berusia 6th masuk ke RS dengan bengkok pada cruris kanan.
Dialami sejak 6 bulan lalu. Pasien riwayat terjatuh dari tempat tidur saat
berbaring. Pasien riwayat patah tulang pada paha kanan dan lengan kanan 1
tahun sebelumnya karena terpleset di rumah. Pada pemeriksaan vital dbn. Pada
mata tampak sklera berwarna kebiruan dan ditemukan deformitas pada paha
kanan, lengan kanan, dan betis kanan. Tidak ditemukan luka pada anggota
gerak. Apakah dx yg mungkin pada kasus
a. Perthes disease
b. Patologic fracture ec chronic osteomyelitis tibia
c. Child abuse
d. Osteogenesis imperfecta
e. Tibia hemimelia
17. Bayi laki-laki berusia 18bln datang ke PKM dibawa oleh orangtuanya dengan
keluhan kedua kaki bengkok. Dialami sejak lahir. Pemeriksaan vital sign dbn.
Pada pemeriksaan fisis pada kedua kaki ditemukan deformitas berupa cavus
ada midfoot, adductus pada forefoot, varus, dan equinus pada hint foot seperti
gambar dibawah. Pemeriksaan motorik dan sensorik dbn. Apakah dx yg tepat
pada kasus

a. Leg calve perthes disease


b. DDH
c. Club foot
d. Pes planovalgus
e. Equinus contracture
18. Pasien laki-laki berusia 58th MRS dengan keluhan nyeri dan luka terbuka sekitar
10cm pada tungkai bawah kanan, luka kotor, dan tampak tulang akibat KLL. Dari
hasil pemeriksaan xray ditemukan fracture comminuted 1/3 middle right tibia nd
fracture 1/3 middle right fibula. Apakah early complications yg dapat timbul
pada kasus
a. Infeksi, vascular injury, compartment syndrome
b. Delayed union, non union, mal union
c. Delayed union, mal union, joints stiffness
d. Infeksi, joint stiffness, delayed union
e. Compartment syndrome, delayed union, mal union
19. Seorang anak laki-laki usia 14th datang ke PKM dengan keluhan tidak dapat
berjalan sejak 1 minggu yterakhir. Pasien mengeluh awalnya nyeri pinggul dan
kadang disertai demam sejak 5 hari yg lalu. Pada pemeriksaan fisik didapatkan
TD 110/80mmHg, HR 90x/mnt, RR 24x/mnt, Suhu 38.5oC. Pada pemeriksaan
fisik ditemukan pasien dengan panggul kiri berada dalam posisi abduction,
flexion external rotation. Terdapat edema, hangat perabaan. Pada pemeriksaan
aspirasi cairan sendi kesan serous, terdapat sel darah putih. Apakah tx yg tepat
a. Pemberian broad spectrum antibiotic
b. Pemberian oral antibiotic
c. Tindakan operatif berupa debridement drainase abses dan antibiotik
sesuai kultur
d. Antibiotik sesuai kultur dan pemberian antibiotik
e. Pemberian antibiotik golongan cephalosporin
20. Seorang anak laki-laki usia 14th datang ke PKM dengan keluhan tidak dapat
berjalan sejak 1 minggu yterakhir. Pasien mengeluh awalnya nyeri pinggul dan
kadang disertai demam sejak 5 hari yg lalu. Pada pemeriksaan fisik didapatkan
TD 110/80mmHg, HR 90x/mnt, RR 24x/mnt, Suhu 38.5oC. Pada pemeriksaan
fisik ditemukan pasien dengan panggul kiri berada dalam posisi abduction,
flexion external rotation. Terdapat edema, hangat perabaan. Pada pemeriksaan
aspirasi cairan sendi kesan serous, terdapat sel darah putih. Apakah
mikroorganisme yg paling sering pada kasus
a. Staphylococcus aureus
b. Pseudomonas aeroginosa
c. Streptococcus betahemolitikus
d. Mycobacterium tuberculosis
e. Azetobacter
21. Laki-laki berusia 65th datang ke IGD RS dengan keluhan nyeri pada panggul
kanan. Pasien riwayat terpleset saat BAK di toilet 1 jam yg lalu. Pada
pemeriksaan primary survey dbn. Pada pemeriksaan panggul kanan ditemukan
deformitas berupa shortening dan external rotasi. Pasien tidak dapat menekuk
panggul kanan dengan baik. Pada pemeriksaan xray pelvis tampak pada
gambar. Apa dx yg tepat

a. Fraktur collum femur


b. Fraktur intertrochanter femur
c. Fraktur subtrochanter femur
d. Fraktur caput femur
e. Fraktur diaphysis femur
22. Seorang perempuan usia 32th datang ke RS dengan keluhan nyeri pada regio
antebrachium kiri yg disebabkan oleh jatuh dari sepeda. Pasien riwayat terjatuh
posisi tangan menopang tubuh. Xray menunjukkan fraktur radius distal
dengan displaced kearah volar. Apa dx kasus
a. Colles fracture
b. Smith fracture
c. Montegia fracture
d. Galeazzi fracture
e. Barton fracture
23. Pasien laki-laki berusia 58th MRS dengan keluhan nyeri dan luka terbuka sekitar
10cm pada tungkai bawah kanan, luka kotor, dan tampak tulang akibat KLL. Dari
hasil pemeriksaan xray ditemukan fracture comminuted 1/3 middle right tibia nd
fracture 1/3 middle right fibula. Berdasarkan klasifikasi gustilo and anderson,
fraktur terbuka yg membutuhkan repair cedera vaskular digolongkan pada grade
berapa
a. Grade IIC
b. Grade IIIA
c. Grade IIIB
d. Grade IIIC
e. Grade IIID
24. Pasien laki-laki berusia 58th MRS dengan keluhan nyeri dan luka terbuka sekitar
10cm pada tungkai bawah kanan, luka kotor, dan tampak tulang akibat KLL. Dari
hasil pemeriksaan xray ditemukan fracture comminuted 1/3 middle right tibia nd
fracture 1/3 middle right fibula. Apakah late complications yg dapat timbul pada
kasus tersebut
a. Infeksi, vacular injury, compartment syndrome
b. Delayed union, non union, infeksi
c. Delayed union, mal union, joints stiffness
d. Infeksi, joints stiffness, delayed union
e. Compartment syndrome, delayed union, mal union
25. Seorang anak laki-laki usia 14th datang ke PKM dengan keluhan tidak dapat
berjalan sejak 1 minggu yterakhir. Pasien mengeluh awalnya nyeri pinggul dan
kadang disertai demam sejak 5 hari yg lalu. Pada pemeriksaan fisik didapatkan
TD 110/80mmHg, HR 90x/mnt, RR 24x/mnt, Suhu 38.5oC. Pada pemeriksaan
fisik ditemukan pasien dengan panggul kiri berada dalam posisi abduction,
flexion external rotation. Terdapat edema, hangat perabaan. Pemeriksaan
penunjang apa yg dapat dilakukan untuk mendeteksi bilamana terdapat
penumpukan cairan sendi
a. Xray
b. Bone scan
c. USG
d. CT scan
e. Fluoroscopy
26. Laki-laki usia 30th datang dengan keluhan nyeri pada pangkal paha akibat KLL.
Dari xray didapatkan fraktur 1/3 proximal femur. Berikut ini yg merupakan arah
displaced dari segmen proksimal pada deforming force dari feaktur shaft femur
adalah
a. Fleksi, eksternal rotasi, dan abduksi
b. Fleksi, interna rotasi, dan abduksi
c. Ekstensi, interna rotasi, dan adduksi
d. Ekstensi, interna rotasi, dan abduksi
e. Fleksi, eksterna rotasi, dan adduksi
27. Seorang wanita usia 37th datang ke IGD RS dengan keluhan nyeri pada regio
kanan antebrachium yg disebabkan oleh jatuh di kamar mandi. Pada
pemeriksaan xray menunjukkan fraktur proksimal ulna dan caput radial dislokasi
kearah volar. Apakah dx yg tepat pada kasus
a. Colles fracture
b. Smith fracture
c. Montegia fracture
d. Galeazzi fracture
e. Barton fracture
28. Seorang laki-laki 29th datang ke RS dengan kepala mengeluh sakit di
antebrachium kiri yg disebabkan oleh jatuh di luar tangan yg diregangkan. Xray
menunjukkan distal 1/3 fraktur radial shaft, pemendekan kekuatan menghasilkan
dislokasi radioulnar distal. Apa dx
a. Colles fracture
b. Smith fracture
c. Montegia fracture
d. Galeazzi fracture
e. Barton fracture
29. Gadis 6th datang ke ER dengan rasa sakit di siku kanannya. Menurut ibunya, dia
jatuh dari sepedanya 30 menit yg lalu. Karena survei utama jelas anda
menemukan pembengkakan dan nyeri di siku kananya, apa xray yg harus anda
lakukan
a. Right elbow AP/Lat
b. Right humerus AP/Lat
c. Bilateral elbow AP/Lat
d. Bilateral forearm AP/Lat
e. Right forearm AP/Lat
30. Pasien laki-laki berusia 58th MRS dengan keluhan nyeri dan luka terbuka sekitar
10cm pada tungkai bawah kanan, luka kotor, dan tampak tulang akibat KLL. Dari
hasil pemeriksaan xray ditemukan fracture comminuted 1/3 middle right tibia nd
fracture 1/3 middle right fibula. Berdasarkan klasifikasi gustilo and anderson,
fraktur terbuka yg membutuhkan tx penutupan luka dengan metode skin
graft/skin flap digolongkan pada grade
a. Grade IIC
b. Grade IIIA
c. Grade IIIB
d. Grade IIIC
e. Grade IIID
31. Seorang anak laki-laki usia 14th datang ke PKM dengan keluhan tidak dapat
berjalan sejak 1 minggu yterakhir. Pasien mengeluh awalnya nyeri pinggul dan
kadang disertai demam sejak 5 hari yg lalu. Pada pemeriksaan fisik didapatkan
TD 110/80mmHg, HR 90x/mnt, RR 24x/mnt, Suhu 38.5oC. Pada pemeriksaan
fisik ditemukan pasien dengan panggul kiri berada dalam posisi abduction,
flexion external rotation. Terdapat edema, hangat perabaan. Apakah dx yg
mungkin
a. Dislokasi hip
b. Hip osteomyelitis
c. Fraktur femur
d. Hip septic arthritis
e. Fraktur neck femur
32. Seorang laki-laki berusia 25th datang ke IGD RS dengan keluhan nyeri pada
tungkai bawah kanan, pada pemeriksaan fisik ditemukan adanya kulit mengkilat
disertai bulla multiple. Riwayat KLL 8 jam yg lalu. Dari pemeriksaan xray
ditemukan adanya fraktur segmental tibia fibula. Apakah dx yg tepat
a. Selulitis
b. Sindrom kompartemen
c. Fraktur terbuka grade IIIA
d. Fraktur terbuka grade IIIB
e. Fraktur terbuka grade IIIC
33. Laki-laki usia 49th datang ke RS dengan keluhan nyeri pada lengan bawah
kirinya. Pada pemeriksaan fisik ditemukan adanya luka robek sebesar 10cm,
nampak ada skin loss, dan tulang ulna tampak keluar, luka kotor. Pasien riwayat
KLL 2 jam yg lalu. Pada pasien ini mengalami fraktur terbuka grade berapa
a. I
b. II
c. IIIA
d. IIIB
e. IIIC
34. Laki-laki berusi 25th datang ke PKM dengan keluhan nyeri lutut kanan dialami
sejak 2 bulan lalu. Nyeri kadang disertai adanya sensasi “pop” pada lutut kanan.
2 bulan lalu pasien riwayat terjatuh saat bermain futsal saat bermain dengan
betis kanan terasa sedikit berputar. Pada pemeriksaan fisik ditemukan anterior
drawer test positif, Mcmurray test negatif. Apakah tx lanjutan untuk kasus
a. Pemberian analgetik
b. Pemberian kompres es
c. Latihan ROM dan penguatan otot
d. Repair meniscus
e. Rekonstruksi ligamen
35. Laki-laki usia 37th datang dengan keluhan nyeri pada siku kanan setelah terjatuh
saat bersepeda. Pada pemeriksaan fisis didapatkan bengkak, luka tidak ada,
terdapat deformitas, dan siku kanan tidak dapat ditekuk. Pada pemeriksaan xray
tampak olecranon terlepas dari distal humerus kearah posterior. Apakah dx
yg tepat
a. Fraktur distal humerus
b. Fraktur olecranon
c. Dislokasi elbow
d. Subluksasi elbow
e. Dislokasi caput radius
36. Seorang laki-laki berusia 25th datang ke IGD RS dengan keluhan nyeri pada
tungkai bawah kanan, pada pemeriksaan fisik ditemukan adanya kulit mengkilat
disertai bulla multiple. Riwayat KLL 8 jam yg lalu. Dari pemeriksaan xray
ditemukan adanya fraktur segmental tibia fibula. Apakah komplikasi yg dapat
terjadi bila tidak dilakukan tindakan segera
a. Infeksi
b. Kematian anggota gerak
c. Pasien meninggal
d. Kekakuan sendi
e. Mal union
37. Laki-laki berusia 25th datang ke IGD dengan keluhan nyeri pada pergelangan
kaki kanan dialami sejak 1 jam sebelum MRS. pasien riwayat terjatuh dengan
posisi pergelangan kaki kanan terlipat kedalam. Primary survey dbn. Pada
secondary survey pergelangan kaki ditemukan swelling, hematom tidak ada,
deformitas tidak ada, dan pergerakan pergelangan kaki kanan terbatas nyeri.
Xray ankle tidak ditemukan kelainan. Cedera yg mungkin pada pasien
a. Fraktur ankle
b. Fraktur distal tibia
c. Fraktur distal fibula
d. Fraktur talus
e. Ankle sprain
38. Laki-laki berusia 65th datang ke IGD RS dengan keluhan nyeri pada panggul
kanan. Pasien riwayat terpleset saat BAK di toilet 1 jam yg lalu. Pada
pemeriksaan primary survey dbn. Pada pemeriksaan panggul kanan ditemukan
deformitas berupa shortening dan external rotasi. Pasien tidak dapat menekuk
panggul kanan dengan baik. Pada pemeriksaan xray pelvis tampak pada
gambar. Apakah tx lanjutan yg tepat

a. Pemasangan infus dan pemberian analgetik


b. Pemasangan gips sirkuler
c. Pemasangan fiksasi screw pada proksimal femur
d. Panggantian sendi panggul baik secara partial maupun total
e. Pemasangan fiksasi plate and screw
39. Laki-laki berusia 25th datang ke IGD dengan keluhan nyeri pada pergelangan
kaki kanan dialami sejak 1 jam sebelum MRS. pasien riwayat terjatuh dengan
posisi pergelangan kaki kanan terlipat kedalam. Primary survey dbn. Pada
secondary survey pergelangan kaki ditemukan swelling, hematom tidak ada,
deformitas tidak ada, dan pergerakan pergelangan kaki kanan terbatas nyeri.
Xray ankle tidak ditemukan kelainan. Tx cedera pada pasien ini
a. RICE
b. Analgetik
c. Pemasangan gips
d. Injeksi asam hyaluronic
e. Rekonstruksi ligamen
40. Laki-laki berusia 65th datang ke IGD RS dengan keluhan nyeri pada panggul
kanan. Pasien riwayat terpleset saat BAK di toilet 1 jam yg lalu. Pada
pemeriksaan primary survey dbn. Pada pemeriksaan panggul kanan ditemukan
deformitas berupa shortening dan external rotasi. Pasien tidak dapat menekuk
panggul kanan dengan baik. Pada pemeriksaan xray pelvis tampak pada
gambar. Apakah arteri yg memberi suplai pada femoral head
a. Arteri femoral circumflexus dan ligamentum teres
b. Arteri obturator dan arteri femoralis
c. Arteri femoralis dan poplitea
d. Arteri poplitea dan tibialis anterior
e. Arteri tibialis anterior dan ligamentum teres
41. Seorang anak laki-laki berusia 12bln datang ke RS dengan keluhan tungkai
kanan lebih pendek dari sisi sebelah kiri. Riwayat jatuh dan demam tidak ada.
Pada pemeriksaan fisis ditemukan galleazzi sign ada, pergerakan abduksi sisi
kanan terbatas dibanding sisi yg kiri. Pada pemeriksaan xray tampak pada
gambar dibawah. Apakah tx yg tepat

a. Pavlik harness
b. Closed reduction hip spica cast
c. Open reduction hip spica cast
d. Pelvic osteotomy
e. Subtrochanteric osteotomy
42. Seorang laki-laki berusia 25th datang ke IGD RS dengan keluhan nyeri pada
tungkai bawah kanan, pada pemeriksaan fisik ditemukan adanya kulit mengkilat
disertai bulla multiple, nyeri hebat saat dilakukan passive stretching pada ujung-
ujung jari kakinya. Riwayat KLL 8 jam yg lalu. Dari pemeriksaan xray ditemukan
adanya fraktur segmental tibia fibula. Apakah dx yg tepat

a. Mal union
b. Non union
c. Infeksi
d. Sindrom kompartemen
e. Kekakuan pada sendi
43. Bayi perempuan usia 3bln dibawa oleh orang tuanya dengan keluhan jari
tangannya berjumlah 6 yg dialami sejak lahir. Ibu dari bayi rutin melakukan
pemeriksaan antenatal. Pada pemeriksaan tangan kiri bayi ditemukan terdapat
ibu jari tangan tambahan. Pada pemeriksaan status NVD dbn. Apakah dx yg
tepat

a. Syndtactyly
b. Macrosyndactyly
c. Complex macrosyndactyly
d. Preaxial polydactyly
e. Postaxial polydactyly
44. Bayi perempuan usia 7hari lahir dari persalinan normal di PKM dengan
pemeriksaan barlow dan ortolani positif. Apakah tx yg tepat pada keadaan
bayi
a. Pavlik harness
b. Closed reduction hip spica cast
c. Openreduction hip spica cast
d. Pelvic osteotomy
e. Subtrochanteric osteotomy
45. Bayi laki-laki usia 7bln dibawa oleh ibunya ke RS dengan keluhan kaki bengkok,
diperhatikan sejak lahir. Pada pemeriksaan fisis didapatkan forefoot adduksi,
heel varus, ankle equinus, midfoot cavus. Apakah kemungkinan dx pasien
a. Metatarsus adductus
b. Hallux valgus
c. Pesplano valgus
d. Hallux varus
e. Club foot
46. Seorang anak perempuan usia 13th datang ke RS dengan keluhan
trendelenburg gait sejak mulai berjalan, tidak ada riwayat trauma. Riwayat
demam disangkal. Apakah kemungkinan dx pasien
a. Club foot
b. DDH
c. CTS
d. Fracture hip
e. Septic arthritis
47. Seorang anak perempuan usia 13th datang ke RS dengan keluhan trendelenburg
gait sejak mulai berjalan, tidak ada riwayat trauma. Riwayat demam disangkal.
Apakah yg ditemukan pada pemeriksaan xray pasien diatas
a. Kite angle >10 derajat
b. Normal xray
c. Angulasi shaft femur
d. Disruption shenton line
e. Parallelism talus dan calcaneus
48. Seorang wanita usia 58th dengan IMT 23kg/m2 nyeri pada kedua sendi lutut,
terutama saat berjalan. Jari-jari tangan kanan dan kiri juga dirasakan kaku dan
nyeri berlangsung sekitar 15-30 menit di pagi hari. Pemeriksaan sendi tampak
nodul pada sendi DIP dan suara berderak pada sendi lutut saat digerakkan.
Kadar asam urat serum 8.0 md/dL. Dx yg paling mungkin
a. Arthritis rheumatoid
b. Arthritis gout
c. Arthritis septic
d. Osteoarthritis
e. Ankylosing spondylitis
49. Pasien bayi perempuan usia 4bln dibawa ibunya ke RS karena kaki kanan dan
kiri panjang sebelah. Dari anamnesis ternyata riwayat kelahiran bayi ini sulit
karena sungsang. BB lahir 4.1kg. sang ibu bercerita padasaat menggantikan
popok terdengar bunyi “klik”. Pada pemeriksaan fisik ditemukan tes barlow dan
ortolani (+). Apakah dx yg tepat
a. Osteoarthritis
b. Fraktur caput femur
c. Fraktur acetabulum
d. Displasia panggul kongenital
e. Rheumatoid arthritis
50. Bayi laki-laki usia 7bln dibawa oleh ibunya ke RS dengan keluhan kaki bengkok,
diperhatikan sejak lahir. Pada pemeriksaan fisis didapatkan forefoot adduksi,
heel varus, ankle equinus, midfoot cavus. Apakah penanganan yg tepat
a. Pemasangan kocher cast
b. Pemasangan hip spica
c. Pemasangan pavlik harness
d. Pemasangan serial manipulation and casting
e. Pemasangan long leg back slab
110.

111. Laki-laki, 65 tahun, datang dengan keluhan nyeri sendi di kedua pergelangan
tangan dan jari-jari dikedua tangannya. Pasien juga mengeluhkan kekakuan di pagi
hari. Pada pemeriksaan fisik didapatkan tanda vital baik, tampak swan neck deformity
pada jari jari tangan kanan dan kiri. Kriteria remisi pada pasien rheumatoid arthritis
adalah ?
a. Nyeri sendi berkurang
b. Kekakuan sendi pagi hari >30 menit
c. Erosi berkurang pada pemeriksaan foto rontgen
d. LED < 20 mm/jam I untuk laki-laki dan <30 mm untuk wanita
e. Faktor rheumatoid sudah negative

112. Apakah tipe kolagen yang disintesis pada tulang rawan?


a. Kolagen tipe I
b. Kolagen tipe II
c. Kolagen tipe V
d. Kolagen tipe VII
e. Kolagen tipe VI

113. Seorang Laki-laki berusia 25 tahun datang ke dokter dengan keluhan demam
tinggi dengan bengkak dan nyeri tungkai bawah kiri. Riwayat kecelakaan, yakni
pasien ditabrak motor sekitar 1 minggu yang lalu dan menyebabkan luka pada tungkai
kiri. Pada pemeriksaan laboratorium didapatkan leukositosis, lalu dikirim ke Bagian
Radiologi untuk pemeriksaan radiologi. Apa pemeriksaan foto radiologi apa yang
tepat untuk kasus ini

114. Seorang laki” berusia 65 tahun datang ke klinik dengan keluahan nyeri sendi
kedua pergelangan tangan dan kedua jari jari tangannya. Pasien juga mengeluh
kekakuan dipagi hari . Pada pemfis didapatkan tanda vital baik. Tampak swan neck
deformity pada jari jari tangan kanan dan kiri, apa pemeriksaan yg dianjurkan pada
kasus di atas
a. Asto titer
b. Biopsy
c. Ana test
d. anti ds-DNA
e. Anti ccp

115. Manakah dibawah ini yang termasuk sel jaringan ikat yang tidak bergerak? *
a.       Sel mast
b.      Sel monosit
c.       Sel lemak
d.      Sel limfosit
e.       Sel mesenkimal

116. apabila kadar Ca meningkat dalam darah melebihi batas normal lalu ingin di
kembalikan ke batas normal bagaimana cara yang harus di lakukan?
a.mengurangi kadar ca melalu ekskresi ginjal

117. Trauma pada otot atau tendon disebabkan oleh peradangan otot atau tendon
yang melebihi batas normal,ini merupakan definisi dari..
A. Atrofi
B.

118. Ada tulang kuat dan lentur, kelenturan itu d sebabkan oleh?
Kolagen

119. Contoh tulang kartilago hyline menjadi tulang keras adalah


Os. femur
120. Seorang pasien 55 tahun datang kedengab keluhan bengkak dan nyeri di ibu
jari kanan, dengan riyawat pesta semalam makann makanan mengandung tinggi
purin? Apa tatalaksananya?
A. Memberikan apurinol
B. Memberikan nsaid
C. Memberikan nsaid dan apurinol bersamaan
D. Memberikan kol"an

121. Ifa seorang mahasiswa yang cantik dan baik hati, datang ke kampus
kesiangan, akibatnya menyebrang tanpa melihat lalu lintas yang rame, sehingga
mengalami kecelakaan akibatnya patah pada lengan kanan ..........
A. Ulna dan ...
B. Clavikula
C. Skapula
D. Humeri (kynyaa)

122. seorang pria berusia 25 tahun mengalami kecelakaan jetski dia mengeluh
sesak dada gangguan yang terjadi adalah
A.Fraktur radialis distal
B.Fraktur pergelangan tangan
C.Fraktur tulang rusuk
D.Fraktur veterbral
E.Fraktur Femur
14. Pada kontraksi otot rangka dengan adanya Ca2+ yang dialami oleh SR maka terjadi
adanya

123. Diagnosis lain dari SLE...

124. Berkas bundel serat yang melekatkan otot pada tulang

125. Kadar CA dalam darah menjalankan proses fisiologi dalam tubuh seperti
dibawah ini, kecuali

a. transmisi impuls saraf,


b. kontraksi otot
c. koangulasi darah
d. sekresi kelenjar
e. saraf (respon sel dan pembelaan sel)

126. Tulang juga berfungsi sebagai cadangan nutrisi, ini disebabkan karena tulang
juga memiliki fungsi

127. cedera apa kalau meniscus? Contact pivoting dan non contact pivoting

128. soal kasus, yang Gold standar ( MRI)


129. bronkeknia di kedua paru, apa penyakitnya ? TBC

130. Jatuh dari pohon, patah tulang paha, sendi apa yg berhubungan?

131. kasus asam urat, bengkak ibu jari, ?

132. Komponen tulang yg kuat jdi lentur, kalsium natrium dll?

133. Tulang yg dri lunak ke keras, tulang apa, dan proses apa na maksud?

134. kontraksi otot (Yg CA, Ca2+, Komponen AP, ATP, dll)

135. ujung tulang sendi terdapat ....

136. kecelakan, bengkak pergelangan tangan, komplikasi awal?

137. mulut nya tidak bisa na angkat ke atas lateral

138. sakit dikedua bahu, simetris?

139. tipe sendi apa di lengan? Sendi engsel

140. achondroplasia karena? Kelainan genetic chormosom 4p lengan pendek

141. Perempuan usia 60tahun nyeri bahu kaku pagi hari apa diagnosisnya

142. Komplikasi SLE

143. Penyebab terjadinya kelelahan otot otot hipotenar dan tenar termasuk regio

144. Diagnosis banding SLE…?

145. Dani jatuh dari pohon mangga, sistem apa yang terganggu?
A. Sistem Muskuloskeletal,
B. sistem integumen,
C. sistem gastrointestinal

146. Tulang bersifat lentur karena mengandung


A. Fosfor
B. Kalsium
C. Kolagen
D. fosfatlen
147. Laki-laki, nyeri di lutut kanan dan rasa terkunci sejak 2 bulan yang lalu,
riwayat main sepak bola dengan keadaan betis terasa

148. Anterior dan posterior drawer test negative

149. Tes yang diperlukan untuk …… meniscus ( Mc Murray, thesaly test, and
appley grind test) adalah

d. Valgus test
e. Lachman Test

150. Bu Sarah merupakan lansia berumur 50 tahun, ia mengalami perubahan pada


tulang belakang, miring bahu, nyeri punggung, postur membungkuk dan perut buncit,
kelainan apakah yang dialami ibu Sarah?
a. Dislokasi
b. Tumor
c. Osteoporosis
d. Sleroderma

151. Trauma otot dan tendon disebabkan peregangan otot dan tendon yang melebihi
batas normal, merupakan definisi dari?
a. Atrofi
b. Skleroderma
c. Rabdomiosis
d. Sprain
e. Strail

152. Jika terjadi penurunan ca di dalam darah, maka tubuh akan berusaha
menaikkan ca di dalam darah dengan cara ?
A.meningkatkan ekspresi ca dalam ginjal
B.meningkatkan pelepasan hormon PTH
C. Menurunkan calstriol
D. meningkatkan pelepasan calsitonin

153. Contoh tulang yang berubah tulang rawan hyalin menjadi tulang keras
adalah ?
A. Os Femur
B. Os mandibular
C. Os clavicula
D. Os temporal

154. Perempuan yang di temukan mengapung di sungai, pertanyaannya itu kakunya


di sebabkan krna knpa
155. kecelakaan lalu lintas kalau nda salah terus dia bermasalah di tangannya nda
bisa menulis ditnya tulang apa yang bermasalah krna nda bisa menulis

156. regio tenar hipotenar :


a. regio fascialis,
b. regio plantaris,
c. regio abdomen,
d. regio palmaris,
e. regio orbicularis

157. Manifestasi klinis terbanyak pada paru


A. Pneumonia
B. Reumatoud Arthritis
C. Effusi Pleura
D. .......
E ........

158. Luka pada paha bagian atas akan menganggu sistem


A. Sistem nerfus
B. Sistem gastrointestinal
C. Sistem respirasi
D. Sistem musculo
E. Sistem integument

159. Sel penyusun tulang yang bukan berasal dari tulang (leukosit) yang berfungsi
sebagai reacsorbsi
A. Osteoby
B. Osteoclasts
C. Osteosit
D. .......
E .......

160. Eksekusi pergerakan motorik dilakukan oleh… dengan mengirim sinyal untuk
dilakukan kontraksi
A.Korteks Premotor
B.kortes suplemen
C.korteks motoric primer
D. Korteks prefrontal

161. Otot thenar dan hipotenar terletak di regio

162. Tipe kolagen yang di sintesis pada tulang rawan adalah


A. Tipe 1
B. Tipe 2
C. Tipe 5
D. Tope 6
E. Tipe 7

163. Informasi mengenai perubahan pada otot dapat diperoleh dari


A. Muscle spindel
B. Muscle golgi
C. Organ tendon
D. Myosin

164. Yang termasuk tulang yang berasal dari intermembranous adalah :


Jawabannya : Os.Mandibula

165. Os humerus saat masa embrio merupakan tulang yang berasal dari proses ?
Jawabannya : Endokondral Ossification

166. Peran Troponin C


Jawabannya : mengikat calcium

167. Laki laki usia 27 tahun nyeri sendi panggul akibat tabrakan mobil.
Pemeriksaan fisik: posissi sendi oanggu kiri refleksi,adduksi dan internal rotasi.
Pemeriksaan x ray : kaput femur berada di atas acetabulum. Diagnosis ?
A. Fraktur neck femur
B. Fraktur intertrokantal
C. Dislokasi posterior sendi panggil
Anterior
Inferior

168. Laki laki usia 35 tahun sakit pada lengan kiri akibat jatuh dari motor. Posis
jatuh di seblah kiri dan tangan menumpu aspal.

169. Pemeriksaan fisik: dislokasi,bengkak lengan kiri, tidak ada luka. Pemeriksan x
ray: fraktur radius distal dan dispalaced dorsal komliksi awal yg memungkinkan ?
A. Infeks
B. Sindrom kompratemen
C. Kekakuan ngeri sendi
D. Neunulion
E. Mulinion

170. Raina sangat kelaparan dan sehingga bunyi bel istirahat Raina sangat terburu
buru ke kantin karna Raina kelaparan sehingga pada saat di tangga Raina terjatuh dan
meringis kesakitan Sehingga susah menegakan kaki nya dan di lihat ada salah satu
tulang lutut menonjol ,dan hipotesis sementara apa yang terjadi pada Raina....
A.dislokasi pada tulang fibula Raina
B.dislokasi pada tulang tiba Raina
C.dislokasi pada tulang patela Raina
D.dislokasi pada tulang patela dan fibula raina
171. Seorang perempuan usia 18 th datang dengan keluhan nyeri di tangan, rambut
rontok, dan demam. Peristiwa ini sudah dirasakan 3 bulan lamanya. Tidak ada riwayat
penyakit sebelumnya. Tidak ada riwayat berobat sebelumnya. Dari hasil pemeriksaan
fisis didapatkan malar rash (+) ... rash (+). Penatalaksaan pertama yang diberikan
pada kasus diatas berdasarkan derajat penyakitnya adalah....
a. .... intervena
b.
c.aza...
d.

172. Kadar Ca dalam darah Penting di pertahankan normal untuk proses-proses


fisiologi tubuh seperti dibawah ini, kecuali:
a. Sebagai neurotransmitters saraf
b. Sebagai second messeger dalam sel
c. Membantu proses pembekuan darah
d. Membantu terjadinya kontraksi otot

173. Manakah dibawah ini yang termasuk sel jaringan ikat yang tidak bergerak?
A. Sel mast
B. Sel monosit
C. Sel lemak
D. Sel limfosit
E. Sel mesenkimal

UJIAN FINAL BLOK MUSKULOSKELETAL UNIVERSITAS BOSOWA TAHUN


AJARAN 2020/2021
 
1. Seorang laki-laki berusia 66 tahun datang ke puskesmas dengan keluhan utama luka pada
tungkai kanan yang tidak sembuh sejak 4 minggu yang lalu. Dari anamnesis diketahui bahwa
pada awalnya kaki pasien terbentur pada tangga. Pasien tidak sadar ada luka pada kakinya
karena tidak timbul nyeri. Pasien tidak pernah memeriksaan kondisi kakinya ke pelayanan
kesehatan, dan hanya merawat lukanya di rumah. Pasien memiliki riwayat diabetes mellitus
10 tahun terakhir, dan tidak minum obat teratur. Pada pemeriksaan laboratorium didapatkan
leukosit 11.000/mm3, hemoglobin 10.8 gr/dl, dan gula darah sewaktu 350 gr/dl.Apakah
diagnosis yang paling mungkin menurut anda ? *
A. Ulkus diabetik
B. Nefropati diabetik
C. Ulkus gangrene
D. Neuropati diabetik
E. Angiopati diabetik
 
2. Laki-laki, 65 tahun, datang dengan keluhan nyeri sendi di kedua pergelangan tangan dan
jari-jari dikedua tangannya. Pasien juga mengeluhkan kekakuan di pagi hari. Pada
pemeriksaan fisik didapatkan tanda vital baik, tampak swan neck deformity pada jari jari
tangan kanan dan kiri. Kriteria remisi pada pasien rheumatoid arthritis adalah ? *
A. Nyeri sendi berkurang
B. Kekakuan sendi pagi hari >30 menit
C. Erosi berkurang pada pemeriksaan foto rontgen
D. LED < 20 mm/jam I untuk laki-laki dan <30 mm untuk wanita (kadar normal LED)
E. Faktor rheumatoid sudah negatif
 
3. Manakah dibawah ini yang tidak termasuk sekresi fibroblast? *
A. Proteoglikan
B. Glukosamine
C. Glikosaminoglikan
D. Fibronektin
E. Kolagen
 
 
4. Apakah yang tidak termasuk dalam faktor intrinsik yang berperan terhadap terjadinya
cedera olahraga ?
A. Kelenturan otot
B. Fleksibilitas sendi
C. Panjang tungkai yang sama atau tidak sama
D. Daya tahan dan kelenturan otot
E. Nutrisi
 
5. Seorang perempuan berusia 52 tahun datang ke klinik dengan keluhan nyeri mendadak
pada sendi kaki sejak 2 hari lalu, disertai dengan bengkak. Riwayat pasien suka minum
alkohol. Pada pemeriksaan fisis didapatkan status lokalis: tampak hiperemis, hangat, bengkak
pada metatarsal phalang 2 dextra. Penyebab yang paling mungkin dari kasus diatas adalah?
(diagnosa atritis gout akut)
A. Defisiensi Estrogen (osteoporosis juga)
B. Defisiensi Vitamin D (rickhets, osteomalasia)
C. Defisiensi Calsium (osteoporosis)
D. Adanya sel Pannus (gout kronik)
E. Autoimmun (RA, SLE)
 
6. Seorang laki-laki berusia 46 th datang ke praktek dokter dengan keluhan nyeri sendi jari
kaki sejak 1 minggu yang lalu. Nyeri dirasakan mendadak disertai muncul benjolan di sendi
jari kaki yg awalnya terasa nyeri dan merah. Pasien suka makan jeroan dan kacang-kacangan.
Pada Pemeriksaan Fisis didapatkan tekanan darah 110/80 mmHg, nadi 84 x/menit, suhu 36.5
C, frekuensi napas 20 x/mnt, tampak benjolan pada sendi MTP-1, merah (-), nyeri tekan (-).
Hasil pemeriksaan asam urat serum 7.8 mg/dL. Apa diagnosis yang paling tepat untuk kasus
ini?
A. Septic arthritis
B. Reumathoid arthritis
C. Gout arthritis std akut
D. Gout arthritis std interkritikal
E. Gout kronik bertopus
 
 
7. Apakah tujuan dilakukannya pencegahan sekunder ?
A. Menghentikan perkembangan penyakit
B. Menghentikan kejadian suatu penyakit (primer)
C. Memulihkan seseorang yang sakit
D. Memberikan rambu-rambu lalu lintas di jalan raya
E. Perlindungan kesehatan manula
 
8. Apakah nama nervus yang menginnervasi otot-otot hamstring (m.biceps femoris,
m.semitendonosus, m.semimembranosus) ? *
A. N. Ischiadicus
B. N. Iliohypogastricus
C. N. Ilioinguinalis
D. N. Genitofemoralis 
E. N. Gluteus superior

9. Anak perempuan berusia 15 tahun diantar oleh orang tuanya ke klinik dengan batuk, nafsu
makan kurang, keringat malam, berat badan menurun. Saat ini datang dengan keluhan nyeri
punggung. Pada pemeriksaan terdengar bunyi pernapasan yang mencurigakan adanya tanda-
tanda bronchopneumonia kedua paru. Dan pada pemeriksaan tulang belakang ada terlihat
scoliosis dan gibbus pada torakolumbal. Apa diagnosis yang paling tepat untuk kasus ini?
A. Ewing’s tumor
B. Spondylitis TBC
C. Avasculer necrosis
D. Ankilosing spondilitis
E. Rheumatoid arthritis
 
10. Seorang laki-laki berusia 38 tahun dibawa oleh keluarganya ke unit gawat darurat rumah
sakit dengan keluhan utama sulit menggerakan panggul kiri setelah mengalami kecelakaan
kendaraan bermotor 1 jam yang lalu. Dari pemeriksaan fisik didapatkan tanda-tanda vital
dalam batas normal, dan tampak tungkai bawah mengalami deformitas berupa hip flexion,
adduksi dan internal rotation.Apakah penatalaksanaan awal yang paling tepat ?
A. Merujuk pasien ke dokter ahli
B. Memberikan analgetik untuk pencegahan nyeri
C. Memposisikan tungkai pasien seanatomis mungkin 
D. Memindahkan pasien ke tempat yang nyaman untuk pasien
E. Imobiliasi menggunakan spalk dengan posisi pasien seadanya
 
11. Seorang laki-laki berusia 60 tahun datang ke puskesmas dengan keluhan utama luka pada
kaki kiri yang tidak sembuh sejak 4 minggu yang lalu. Dari anamnesis diketahui bahwa pada
awalnya kaki pasien terbentur pada tangga. Pasien tidak sadar ada luka pada kakinya karena
tidak timbul nyeri. Pasien memiliki riwayat diabetes mellitus 10 tahun terakhir, dan tidak
minum obat teratur. Pasien tidak pernah memeriksaan kondisi kakinya ke pelayanan
kesehatan, dan hanya merawat lukanya di rumah. Setelah 4 bulan kemudian pasien
mengeluhkan kaki menghitam. Pada pemeriksaan laboratorium didapatkan leukosit
11.000/mm3, hemoglobin 10.8 gr/dl, dan gula darah sewaktu 350 gr/dl. Apakah terapi yang
paling tepat menurut anda ? (diagnosa osteomyelitis kronik)
A. Terapi operasi berupa amputasi
b. Antibiotik oral untuk penanganan infeksi
C. Pemberian antibiotik local luka
D. Terapi non operasi dengan modifikasi sepatu
E. Terapi operasi dengan osteotomy
 
12. Seorang laki-laki berusia 65 tahun datang ke klinik dengan keluhan nyeri sendi di kedua
tangan dan jari-jari dikedua tangannya. Pasien juga mengeluhkan kekakuan di pagi hari yang
berlangsung lebih dari 30 menit. Pada pemeriksaan fisik didapatkan tanda vital dalam batas
normal. Hasil pemeriksaan laboratorium didapatkan rheumatoid factor (+). Apa tatalaksana
yang paling tepat pada kasus diatas ? *
A. Non-Steroidal Anti-Inflammatory Drugs 
b. Disease Modifying Anti Rheumatic Drugs (DMARDs)
c. Acetaminophen
d. Opioid
e. Kortikosteroid (sebagai bridging therapy)
 
13. Peningkatan konsumsi zat gizi tertentu akan meningkatkan ekskresi asam urat. Zat apakah
yang dimaksud?
A. Protein
B. Karbohidrat
C. Lemak
D. Vitamin C
E. Vitamin K
 
14. Manakah dibawah ini yang termasuk sel jaringan ikat yang tidak bergerak?
A. Sel mast
B. Sel monosit
C. Sel lemak
D. Sel limfosit
E. Sel mesenkimal
 
15.Seorang laki-laki berusia 34 tahun datang ke puskesmas dengan keluhan utama keluar
cairan pada luka di kaki kiri sejak 2 bulan yang lalu. Dari anamnesis diketahui pasien
memiliki riwayat operasi pemasangan implant pada betis kirinya akibat patah tulang terbuka
3 tahun yang lalu. Pasien saat ini masih merasakan nyeri saat berjalan, sehingga
menggunakan bantuan tongkat saat berjalan. Dari pemeriksaan fisik didapatkan tanda-tanda
vital dalam batas normal. Pada pemeriksaan laboratorium didapatkan leukosit 16.000/mm3,
hemoglobin 12.0 gr/dl, dan laju endap darah 80/60 mm/jam. Apakah penanganan awal yang
paling tepat untuk pasien diatas ?
A. Rujuk langsung ke dokter Orthopaedi terdekat
B. Merawat luka hingga kering
C. Memberikan Antibiotik dan irigasi luka
D. Memberikan anti nyeri dan tetap menggunakan tongkat
E. Mengedukasi pasien agar merawat luka dan memberikan anti nyeri.
Option 6
 
16. Apakah nama nervus yang menginnervasi otot-otot quadriceps femoris?
A. N. Tibialis
B. N. Peroneus communis
C. N. Femoralis
D. N. Cutaneus femoris lateralis
E. N. Obturatorius
 
17. Seorang laki-laki berusia 55 tahun datang dengan keluhan bengkak dan nyeri pada ibu jari
kaki kanan. Riwayat dari pesta malam sebelumnya dan mengkonsumsi makanan tinggi purin?
Apa tatalaksana yang paling tepat buat pasien tersebut? (diagnosa gout akut)
A. Memberikan allopurinol (untuk menurunkan gout bertopus)
B. Memberikan NSAID
C. Memberikan allopurinol dan NSAID secara bersamaan
D. Memberikan NSAID dan Kolkisin
E. Memberikan Kolkisin dan Allopurinol (tidak boleh diberi bersamaan pada gout akut)
 
18. Penggunaan Helm pada pengendara sepeda motor adalah upaya pencegahan cedera
kepala, menurut Leavell & Clark merupakan pencegahan tingkat?
A. 1
B. 2
C. 3
D. 4
E. 5
 
19. Apakah sel tulang yang berfungsi membebaskan kalsium dari matriks tulang ?
A. Sel osteoblast
B. Sel osteosit
C. Sel osteoklast
D. Giant osteklast
E. Giant osteosit
 
20. Seorang Laki-laki berusia 25 tahun datang ke dokter dengan keluhan demam tinggi
dengan bengkak dan nyeri tungkai bawah kiri. Riwayat kecelakaan, yakni pasien ditabrak
motor sekitar 1 minggu yang lalu dan menyebabkan luka pada tungkai kiri. Pada pemeriksaan
laboratorium didapatkan leukositosis, lalu dikirim ke Bagian Radiologi untuk pemeriksaan
radiologi. Hasil pemeriksaan radiologis ternyata ditemukan destruksi osteolitik 1/3 distal
dengan soft tissue swelling. Maka diagnosis radiologisnya adalah? *
A. Osteomielitis kronis
B. Osteomielitis akut 
C. Avaskuler nekrosis
D. Spondilitis TBC
 
21. Seorang laki-laki berusia 42 tahun, dating ke unit gawat darurat rumah sakit dengan
keluhan utama nyeri pada panggul kanan sejak 30 menit yang lalu akibat kecelakaan lalu
lintas. Dari pemeriksaan fisik didapatkan tanda-tanda vital dalam batas normal, panjang
tungkai kanan lebih pendek 2 cm dibandingkan yang kiri. Pada pemeriksaan radiologis
didapatkan gambaran seperti dibawah ini. Apa komplikasi yang paling sering terjadi pada
kasus ini? *

a. Nonunion (tdk sembuh 6-8 bulan, ada pseudoarthritis)


b. Malunion (sembuh, ada deformitas)
c. Osteoarthritis
d. Osteonecrosis (kematian jaringan tulang karna kekurangan suplai darah)
e. Dislocation
 
 
22. Seorang perempuan berusia 45 tahun, diagnosis suspek RA dan diberikan prednisone
sebagai bridging therapy untuk meredakan keluhan nyeri. Manakah pernyataan yang benar
mengenai prednisone?
A. Merupakan anti inflamasi non steroid
B.Tidak memiliki efek pada metabolisme karbohidrat, protein dan lipid
C. Tidak memiliki efek permissive pada katekolamin
D. Menggangu axis hipotalamus-pituitari-adrenal bila diberikan pada dosis besar dan berhenti
secara mendadak
E. Tidak memiliki efek imunosupresive (punya efek antinflamasi/immunosupresive)
 
23. Seorang laki-laki berusia 38 tahun dibawa oleh keluarganya ke unit gawat darurat rumah
sakit dengan keluhan utama sulit menggerakan panggul kiri setelah mengalami kecelakaan
kendaraan bermotor 1 jam yang lalu. Dari pemeriksaan fisik didapatkan tanda-tanda vital
dalam batas normal, dan tampak tungkai bawah mengalami deformitas berupa hip flexion,
adduksi dan internal rotation. Pada pemeriksaan radiologis didapatkan gambaran sebagai
berikut. Apakah diagnosis yang paling tepat menurut anda ? *

A. Dislokasi anterior panggul kiri


B. Dislokasi anterior panggul kanan
C. Dislokasi posterior panggul kiri 
D. Dislokasi posterior panggul kanan
E. Dislokasi sentral panggul kiri
 
24. Apa penanda laboratorium yang menunjang diagnosis SLE berdasarkan kriteria ARA ?
a. ANA Tes positif
b. Anemia
c. Limfositosis
d. A & B benar
e. A, B, C benar
 
25. Seorang laki-laki berusia 45 tahun datang ke puskesmas dengan keluhan nyeri pasca
amputasi below knee sinistra akibat ganggren diabetik. Skor VAS pasien adalah 7.
Farmakoterapi anti nyeri yang dapat diberikan berdasarkan keluhan pasien adalah?
A. Paracetamol tab
B. Ibuprofen tab
C. Natrium diklofenak tab
D. Meloxicam tab
E. Kodein tab + NSAID
 
26. Seorang wanita, 55 tahun datang ke puskesmas dengan keluhan nyeri lutut kanan setahun
belakangan. Biasanya pasien merasakan lututnya kaku pada pagi hari sekitar 15 menit dan
setelahnya bisa bergerak namun disertai rasa sakit terutama jika berusaha jalan lebih cepat.
Pasien mengaku mengkonsumsi acetaminophen untuk menghilangkan nyeri yang didapat dari
apotek, namun nyerinya bertambah belakangan ini. Pada pemeriksaan anthropometric
didapatkan tinggi badan: 155 cm, berat badan 80 kg, dan lingkar perut 98 cm. Apa suplemen
nutrisi yang bisa dipertimbangkan untuk pasien di atas ? (diagnosa OA)
A. Suplemen Vitamin D
B. Suplemen Kalsium
C. Suplemen Kalsium + Vitamin D
D. Suplemen Chondroitin
E. Suplemen Glucosamine-Chondroitin Sulphate
 
27. Seorang wanita usia 60 tahun, datang ke RS dengan keluhan nyeri pada punggung sejak 2
minggu sebelum masuk rumah sakit. Pasien juga mengaku telah mengalami menopause sejak
1 tahun yang lalu. Riwayat persalinan 9 kali dengan pemberian ASI eksklusif.Pada
pemeriksaan fisik didapatkan TD 130/90 mmHg, HR 90x/mnt, RR 22x/mnt dan suhu 36,5C.
Pada pemeriksaan rontgen didapatkan fraktur kompresi L3-L4. Apakah kemungkinan
diagnosis pasien tersebut? *
A. Osteoporosis primer (postmenopausal dan senile : karena usia tua)
B. Osteoporosis sekunder (karena penyakit endokrin. Ex: DM)
C. Osteoporosis tersier
D. Osteoporosis senilis
E. Osteopenia
 
28. Seorang perempuan berusia 35 tahun datang dengan keluhan nyeri pada sendi-sendi jari
kedua tangannya. Nyeri terutama dirasakan pada pagi hari disertai keluhan terasa kaku yang
lamanya lebih dari 30 menit. Pemeriksaan fisik didapatkan sendi-sendi bengkak, eritema,
teraba panas, dan nyeri saat digerakkan. Apa farmakoterapi yang tepat sebelum pasien
dirujuk ke ahli reumatologi?
A. Colchisine
B. Metotrexate
C. Glucosamine
D. Sulfasalazine
E. Natrium diclofenac
 
29. Seorang wanita usia 34 tahun datang ke klinik dengan keluhan nyeri pada pinggang
bawah dialami sejak kurang lebih 5 bulan yang lalu,riwayat trauma jatuh dari ketinggian 2
meter dengan pantat jatuh duluan, tidak ada defisit neurologis. Apa tindakan konservatif yang
tepat untuk kasus ini? *
A. Pemasangan collar neck
B. kompres es
C. Kompres panas
D. Lumbar Brace 
E. Analgetik
 
30. Manakah dibawah ini yang tidak sesuai untuk sel mast? *
A. Granula mengandung heparin
B. Granula diwarnai secara ortochromatik
C. Mengikat immunoglobulin E
D. Banyak dalam lamina propria
E. Berperan dalam reaksi alergi
 
31. Seorang laki-laki berusia 34 tahun datang ke puskesmas dengan keluhan utama keluar
cairan pada luka di kaki kiri sejak 2 bulan yang lalu. Pasien memiliki riwayat operasi
pemasangan implant pada betis kirinya akibat patah tulang terbuka 3 tahun yang lalu. Pasien
saat ini masih merasakan nyeri saat berjalan, sehingga menggunakan bantuan tongkat saat
berjalan. Dari pemeriksaan fisik didapatkan tanda-tanda vital dalam batas normal. Pada
pemeriksaan laboratorium didapatkan leukosit 16.000/mm3, hemoglobin 12.0 gr/dl, dan laju
endap darah 80/60 mm/jam. Apakah diagnosis yang paling mungkin ? *
A. Karsinoma squamous sel
B. Sindroma Kompartemen
C. Osteomyelitis Akut
D. Osteomyelitis Kronis
E. Sarkoma soft tissue
 
32. Seorang laki-laki berusia 46 th datang ke praktek dokter dengan keluhan nyeri sendi jari
kaki sejak 1 minggu yang lalu. Nyeri dirasakan mendadak disertai muncul benjolan di sendi
jari kaki yang awalnya terasa nyeri dan merah. Pasien suka makan jeroan dan kacang-
kacangan. Pada Pemeriksaan fisis didapatkan tekanan darah 110/80 mmHg, nadi 84 x/menit,
suhu 36.5 C, frekuensi napas 20 x/mnt, tampak benjolan pada sendi MTP-1, merah (-), nyeri
tekan (-). Hasil pemeriksaan asam urat serum 7.8 mg/dL. Apa patofisiologi terjadinya kasus
ini? *
A. Osteofit
B. Terbentuknya pannus
C. Densitas tulang yang berkurang
D. Akumulasi Kristal monosodium urat (MSU)
E. Autoantibodi
 
33. Apakah yang tidak termasuk edukasi yang tepat pada pasien OA? *
A. Sementara menghindari atau mencegah untuk posisi/gerakan jongkok
B. Lebih sering naik tangga
C. Berenang
D. Mengurangi berat badan
E. Berjalan dengan tongkat atau walker
 
34. Apakah jenis latihan yang membantu proses rehabilitasi cedera olahraga yang
menekankan gerakan yang sama, simple, monoton, berulang-ulang? *
A. Latihan fleksibilitas sendi
B. CLatihan endurance kardio-respirasi dan otot
C. Latihan ketangkasan (agility)
D. Latihan koordinasi
E. Latihan kecepatan
 
35. Seorang wanita, 55 tahun datang ke puskesmas dengan keluhan nyeri lutut kanan setahun
belakangan. Biasanya pasien merasakan lututnya kaku pada pagi hari sekitar 15 menit dan
setelahnya bisa bergerak namun disertai rasa sakit terutama jika berusaha jalan lebih cepat.
Pasien mengaku mengkonsumsi acetaminophen untuk menghilangkan nyeri yang didapat dari
apotek, namun nyerinya bertambah belakangan ini. Pada pemeriksaan anthropometric
didapatkan tinggi badan: 155 cm, berat badan 80 kg, dan lingkar perut 98 cm. Jika diputuskan
untuk melakukan penurunan berat badan, berapa target minimal penurunan berat badan yang
harus dicapai agar bisa efektif membantu mengurangi gejala ?
A. 10 kg
B. 15 kg
C. 4 kg
D. 20 kg
E. 25 kg
 
 
36. Seorang laki laki usia 70 tahun datang dengan keluhan sulit untuk membengkokkan dan
meluruskan punggungnya. keluhan ini sudah lama dirasakan oleh pasien. Gejala awal yang
dialami berupa demam. Pada pemeriksaan fisik didapatkan adanya spasme otot punggung.
Pada pemeriksaan X ray thoracolumbal didapatkan adanya diskus intervertebralis yang
menyatu dengan vertebra. Apakah diagnosis yang tepat untuk kasus ini ? *
A. Osteoporosis
B. Degenerative disc disease
C. Dlumbal canal stenosis
D. spondylosis
E. Ankylosing spondylitis
 
37. Seorang pasien laki-laki berusia 59 tahun berkunjung ke dokter umum mengeluhkan nyeri
pada bahu kanan dan diresepkan etoricoxib. Setelah mengkonsumsi obat tersebut selama 20
hari, pasien tiba-tiba mengeluh nyeri dada secara tiba-tiba. Pasien memiliki riwayat penyakit
jantung dan pasca pemasangan stent 1 bulan lalu. Apa yang mendasari terjadinya nyeri dada
pasca konsumsi etirocoxib?
A. Penurunan tekanan darah secara tiba-tiba
B. Vasodilatasi pembuluh darah koroner
C. Terjadi trombosis di pembuluh darah jantung
D. Penurunan denyut jantung
E. Peningkatan kontraktilitas otot jantung
 
38. Seorang perempuan berusia 70 tahun datang dengan keluhan nyeri pada punggung,
dialami sejak 3 tahun terakhir memberat 2 bulan terakhir. Nyeri menjalar ke pantat sebelah
kanan, nyeri memberat ketika berdiri dan berjalan. Kelemahan pada kedua tungkai bawah
tidak ditemukan. Pasien tidak merasakan gangguan berkemih maupun buang air besar.
Riwayat trauma disangkal oleh pasien. Apa diagnosis yang paling tepat pada pasien ini ? *
A. Lumbal Spinal Stenosis
B. Cervical Radiculopathy
C. Cervical Myelopathy
D. Muscle Spasme
E. Equina Syndrome
 
39. Seorang wanita usia 60 tahun, datang ke RS dengan keluhan nyeri pada punggung sejak 2
minggu sebelum masuk rumah sakit. Pasien juga mengaku telah mengalami menopause sejak
1 tahun yang lalu. Riwayat konsumsi steroid jangka panjang (+). Riwayat persalinan 9 kali
dengan pemberian ASI eksklusif. Pada pemeriksaan fisik didapatkan TD 130/90 mmHg, HR
90x/mnt, RR 22x/mnt dan suhu 36,5C. Pada pemeriksaan rontgen didapatkan fraktur
kompresi L3-L4. Apa terapi yang dapat diberikan untuk penanganan kasus ini? (diagnosa
osteoporosis)
A. Pemberian NSAID+hentikan steroid
B. Pemberian NSAID+diet tinggi kalsium
C. Pemberian kalsitonin+Vitamin A
D. Bifosfonat+preparat kalsium
E. Fisioterapi+preparat kalsium
 
40. Seorang laki-laki 65 tahun,datang dengan keluhan nyeri sendi di kedua pergelangan
tangan dan jari-jari dikedua tangannya. Pasien juga mengeluhkan kekakuan di pagi hari. Pada
pemeriksaan fisik didapatkan tanda vital baik, tampak swan neck deformity pada jari-jari
tangan kanan dan kiri. Apa diagnosis yang paling mungkin untuk kasus ini? *
A. Osteoarthritis
B. Gout Arthritis
C. Rhematoid arthritis
D. Lupus Arthritis
E. Sindrom-Sjogren
 
41. Seorang laki-laki 38 tahun datang ke rumah sakit dengan keluhan sulit melakukan
gerakan plantar fleksi secara tiba-tiba saat meloncat ketika bermain bola volley. Pasien
merasa nyeri pada daerah sendi pergelangan kaki dan tampak deformitas pada daerah
posterior cruris. Apakah kemungkinan diagnosis pasien tersebut di atas bila ditemukan
bulging pada daerah gastrocnemius? *
A. fracture tibia
B. ankle sprain
C. fracture ankle joint
D. rupture Achilles tendon
E. rupture otot gastrocnemius
 
42. Apakah nama nervus yang merupakan percabangan N. Ischiadicus yang berjalan di lateral
os fibula? *
A. N. Tibialis
B. N. Peroneus communis
C. N. Femoralis (otot quadriceps femoralis)
D. N. Cutaneus femoris lateralis
E. N. Obturatorius
 
43. Apa penyakit muskuloskeletal yg terbanyak di RSUP DR Wahidin Sudiro Husodo ?
A. Fraktur & cedera ligamen
B. Sindroma nyeri
C. Tumor sistem musculoskeletal
D. Penyakit rematik
E. Kelainan degenerative tulang dan sendi
 
44. Laki- laki 45 tahun datang dengan keluhan nyeri dan bengkak pada ibu jari kaki kanan
yang dialami 5 jam yang lalu. Riwayat nyeri di ibu jari kaki kanan sejak setahun lalu,
terutama pada malam hari dan jika cuaca dingin. Pemeriksaan fisis : dolor (+), kalor (+) bulge
sign (-), balloon sign (-). Hasil pemeriksaan laboratorium apakah yang dapat menunjang
untuk menegakkan diagnosis kasus di atas? (diagnosa gout akut)
A. Laju Endap Darah meningkat
B. Rheumatoid Factor positif
C. Hiperurisemia
D. A dan B benar
E. A dan C benar
 
45. Seorang perempuan usia 60 tahun, datang ke RS dengan keluhan nyeri pada punggung
sejak 2 minggu sebelum masuk rumah sakit. Pasien juga mengaku telah mengalami
menopause sejak 1 tahun yang lalu. Riwayat konsumsi steroid jangka panjang (+). Riwayat
persalinan 9 kali dengan pemberian ASI eksklusif.Pada pemeriksaan fisik didapatkan TD
130/90 mmHg, HR 90x/mnt, RR 22x/mnt dan suhu 36,5C. Pada pemeriksaan rontgen
didapatkan fraktur kompresi L3-L4. Apa faktor resiko yang dapat diubah pada kasus ini? *
A. Usia
B. Jenis kelamin
C. Riwayat keluarga
D. Diet kalsium
E. Menopause
 
46. Seorang laki-laki berusia 65 tahun datang ke klinik dengan keluhan nyeri sendi di kedua
pergelangan tangan dan jari-jari dikedua tangannya. Pasien juga mengeluhkan kekakuan di
pagi hari. Pada pemeriksaan fisik didapatkan tanda vital baik, tampak swan neck deformity
pada jari jari tangan kanan dan kiri. Apa pemeriksaan yang dianjurkan untuk kasus diatas ?
(diagnosa RA)
A. Biopsy
B. Asto titer (rheumatic fever)
C. Anti ds-DNA (SLE)
D. ANA test (SLE)
E. anti-CCP (berganti jadi ACPA)
 
47. Seorang Laki-laki berusia 25 tahun datang ke dokter dengan keluhan demam tinggi
dengan bengkak dan nyeri tungkai bawah kiri. Riwayat kecelakaan, yakni pasien ditabrak
motor sekitar 1 minggu yang lalu dan menyebabkan luka pada tungkai kiri. Pada pemeriksaan
laboratorium didapatkan leukositosis, lalu dikirim ke Bagian Radiologi untuk pemeriksaan
radiologi. Apa pemeriksaan foto radiologi apa yang tepat untuk kasus ini ? *
A. Foto cruris kiri posisi obliq
B. Foto cruris kiri posisi lateral
C. Foto cruris kiri posisi Antero posterior (AP)
D. Foto cruris kiri posisi Antero posterior (AP) + obliq
E. Foto cruris kiri posisi Antero posterior (AP) + lateral
 
 
48. Seorang perempuan usia 60 tahun, datang ke RS dengan keluhan nyeri pada punggung
sejak 2 minggu sebelum masuk rumah sakit. Pasien juga mengaku telah mengalami
menopause sejak 1 tahun yang lalu. Riwayat konsumsi steroid jangka panjang (+). Riwayat
persalinan 9 kali dengan pemberian ASI eksklusif. Pada pemeriksaan fisik didapatkan
tekanan darah 130/90 mmHg, nadi 90x/mnt, frekuensi napas 22x/mnt dan suhu 36,5C. Apa
pemeriksaan baku emas pada kasus ini? (diagnosa osteoporosis)
A. X-Ray
B. Computed Tomography Scan
C. PET-Scan
D. Bone Mineral Densitometry
E. Magnetic Resonance Imaging
 
 
49. Apa bentuk arthritis yang paling sering dijumpai di Amerika Serikat?
A. LES
B. Arthritis rheumatoid
C. Gout
D. Pseudogout
E. Osteoartritis
 
50. Seorang perempuan berusia 52 tahun datang ke klinik dengan keluhan nyeri mendadak
pada sendi kaki sejak 2 hari lalu, disertai dengan bengkak. Riwayat pasien suka minum
alkohol. Pada pemeriksaan fisis didapatkan status lokalis: tampak hiperemis, hangat, bengkak
pada metatarsal phalang 2 dextra. Pemeriksaan laboratorium didapatkan asam urat 10 mg/dL.
Apa tatalaksana yang tepat pada pasien ini? (diagnosa gout akut)
A. Asam mefenamat
B. Allopurinol
C. Natrium diclofenac
D. Kolkisin
E. Meloxicam
 
51. Laki- laki 45 tahun datang dengan keluhan nyeri dan bengkak pada ibu jari kaki kanan
yang dialami 5 jam yang lalu. Riwayat nyeri di ibu jari kaki kanan sejak setahun lalu,
terutama pada malam hari dan jika cuaca dingin. Pemeriksaan fisis : dolor (+), kalor (+) bulge
sign (-), balloon sign (-). Tes laboratorium yang dapat dilakukan untuk memantau terjadinya
komplikasi adalah: (diagnosa gout)
A. Ureum-Kreatinin
B. Tes darah rutin
C. Urin Rutin
D. GOT/GPT
E. GDS
 
52. Seorang laki-laki umur 67 tahun datang ke klinik dengan keluhan nyeri dan bengkak serta
kemerahan pada ibu jari kaki dan tidak bisa memakai sepatu. Tidak ada riwayat demam
sebelumnya. Hasil pemeriksaan laboratorium ditemukan peningkatan asam urat dan tidak ada
leukositosis. Pemeriksaan foto radiologi kaki apa yang dilakukan untuk menegakkan
diagnosis ?
A. Posisi foto anterior posterior + obliq
B. Posisi foto anterior posterior + lateral
C. Posisi foto posterior anterior + obliq
D. Posisi foto anterior posterior
E. Posisi lateral
 
53. Seorang laki-laki berusia 66 tahun datang ke puskesmas dengan keluhan utama luka pada
tungkai kanan yang tidak sembuh sejak 4 minggu yang lalu. Dari anamnesis diketahui bahwa
pada awalnya kaki pasien terbentur pada tangga. Pasien tidak sadar ada luka pada kakinya
karena tidak timbul nyeri. Pasien tidak pernah memeriksaan kondisi kakinya ke pelayanan
kesehatan, dan hanya merawat lukanya di rumah. Pasien memiliki riwayat diabetes mellitus
10 tahun terakhir, dan tidak minum obat teratur. Pada pemeriksaan laboratorium didapatkan
leukosit 11.000/mm3, hemoglobin 10.8 gr/dl, dan gula darah sewaktu 350 gr/dl.Apakah
tatalaksana awal yang dapat diberikan kepada pasien ? (diagnosa ulkus diabetik)
A. Terapi operasi berupa amputasi
B. Antibiotik oral untuk penanganan infeksi
C. Pemberian antibiotik local luka
D. Terapi non operasi dengan modifikasi sepatu (seperti gips)
E. Terapi operasi dengan osteotomy
 
54. Seorang laki-laki berusia 43 tahun datang ke puskesmas dengan keluhan tiba-tiba nyeri
sendi ibu jari kaki kiri disertai bengkak kemerahan. Kemarin pasien makan sate kambing.
Keluhan serupa pernah dirasakan 2 tahun yang lalu. Pada pemeriksaan fisik didapatkan tanda
vital dalam batas normal, metatarsophalangeal joint digiti 1 pedis sinistra: edema (+),
hiperemis (+), pada perabaan hangat, nyeri saat disentuh (+). Pada pemeriksaan laboratorium
didapatkan asam urat 14 mg/dL. Apakah terapi yang paling tepat? (diagnosa gout akut)
A. Parasetamol + allopurinol
B. Na-diclofenac + allopurinol
C. Kolkisin + allopurinol
D. Kolkisin + natrium diclofenac
E. Steroid + aspirin
 
55. Seorang laki-laki berusia 60 tahun dengan riwayat penyakit jantung coroner, Saat ini
datang ke klinik akibat pembengkakan akut pada sendi MTP-1 kiri. Pemeriksaan fisik
menunjukkan sendi yang bengkak, hangat dan eritema. Tidak ada riwayat trauma
sebelumnya. Apakah farmakoterapi untuk menghilangkan nyeri akut? * (diagnosa gout akut)
A. Allopurinol (untuk gout bertofus)
B. Kalium diklofenak
C. Celecoxib (tidak dianjurkan untuk riwayat PJK)
D. Tramadol (tidak dianjurkan untuk riwayat PJK)
E. Kodein (tidak dianjurkan untuk riwayat PJK)
 
56. Seorang laki-laki umur 67 tahun datang ke klinik dengan keluhan nyeri dan bengkak serta
kemerahan pada ibu jari kaki dan tidak bisa memakai sepatu. Tidak ada riwayat demam
sebelumnya. Hasil pemeriksaan laboratorium ditemukan peningkatan asam urat dan tidak ada
leukositosis. Apa tanda radiologis yang khas pada kasus ini? *
A. Abses
B. Ada kalsifikasi
C. Punch out + abses
D. Ada reaksi perosteal
E. Punch out + tophus
 
 
57. Seorang laki-laki berusia 34 tahun datang ke puskesmas dengan keluhan utama keluar
cairan pada luka di kaki kiri sejak 2 bulan yang lalu. Pasien memiliki riwayat operasi
pemasangan implant pada betis kirinya akibat patah tulang terbuka 3 tahun yang lalu. Pasien
saat ini masih merasakan nyeri saat berjalan, sehingga menggunakan bantuan tongkat saat
berjalan. Dari pemeriksaan fisik didapatkan tanda-tanda vital dalam batas normal. Pada
pemeriksaan laboratorium didapatkan leukosit 16.000/mm3, hemoglobin 12.0 gr/dl, dan laju
endap darah 80/60 mm/jam. dari gambaran klinis seperti gambar dibawah ini . Apakah
mikrorganisme yang paling mungkin menyebabkan infeksi pada kasus diatas ? (diagnosa
osteomyelitis kronis)
A. Streptococcus Group A
B. Staphylococcus Aureus
C. Pneumokokus
D. Propionibacterium Acnes
E. Clostridium Tetani
 
58. Apakah tipe kolagen yang disintesis pada tulang rawan?
A. Kolagen tipe I (fibroblast, jar.ikat, osteoblast, sel otot polos, beberapa epitel)
B. Kolagen tipe II (Disintesa : kondroblast & kondrosit, sel saraf retina, notochord.)
C. Kolagen tipe V (disintesa : kondrosit, sel otot polos)
D. Kolagen tipe VII
E. Kolagen tipe VI
 

59. Seorang wanita usia 70 tahun datang dengan keluhan nyeri pada punggung bawah dialami
sejak 6 bulan yang lalu, disertai dengan rasa kram pada kedua tungkai bawah terutama
sebelah kanan, tidak ada riyawat trauma sebelumnya. Pada pemeriksaan x ray didapatkan
adanya osteofit (disebabkan RA, ankylosing spondylitis, SLE, gondok, stenosis spinal, OA).
Apakah diagnosis yang tepat pada pasien ini ? *
A. spondylolisthesis (tulang belakang bergeser terlalu jauh menyebabkan penekanan pada
saraf sehingga punggung sakit, mati rasa, nyeri kaki)
B. spondylolisis
C. spondylitis (nama lainnya spondyloatrosis)
D. spondyloatrosis
E. spondylosis (degenerasi tulbel akibat penuaan, mirip OA tulbel)
 
60. Seorang laki-laki berusia 65 tahun datang ke klinik dengan keluhan nyeri sendi di kedua
tangan dan jari-jari dikedua tangannya. Pasien juga mengeluhkan kekakuan di pagi hari yang
berlangsung lebih dari 30 menit. Pada pemeriksaan fisik didapatkan tanda vital dalam batas
normal. Hasil pemeriksaan laboratorium didapatkan rheumatoid factor (+). Apa patofisiologi
terjadinya kasus diatas ? (diagnosis RA)
A. Sinovitis
B. Osteofit
C. Akumulasi Kristal MSU
D. Penyempitan celah sendi
E. Osteoporosis
 
61. Apakah aspek utama di dalam pencegahan primer ?
A. Skrining (sekunder)
B. Promosi kesehatan (modifikasi determinan/faktor resiko kausa penyakit, perlindungan
spesifik)
C. Deteksi dini (termasuk pengobatan segera) (sekunder)
D. Rehabilitasi (termasuk pengobatan) (tersier)
E. Pembatasan kecacatan (tersier)

UJIAN PRAKTIKUM BLOK MUSKULOSKELETAL FK UNIVERSITAS BOSOWA


 
1. Persiapan sampel yang benar untuk pemeriksaan CRP adalah

Dengan antikoagulan (Antikoagulan adalah obat yang berfungsi mencegah


penggumpalan darah. Obat ini bekerja dengan cara menghambat kerja protein yang
terlibat dalam proses pembekuan darah)
Tidak hemolisis (tidak lipemik dan isterik)
Menggunakan plasma darah
Pasien diminta berpuasa
 
2. Pemeriksaan Sel LE positif dapat ditemukan pada penyakit
Chronic Heart Failure
Systemic Lupus Erytematosus (RA, lupoid hepatitis, dan penyakit kolagen)
Rosasea (gangguan kulit wajah yang ditandai dengan kulit kemerahan dan bintik yang
menyerupai jerawat.)
Gout
 
3. Pada pemeriksaan di bawah mikroskop, sel apa yang kita cari pada pemeriksaan Sel LE?
Neutrofil (sitoplasma ungu pucat)
basofil
eosinofil
trombosit
 
4. Kadar CRP dapat dihitung menggunakan rumus
Pengenceran tertinggi yang positif x 6mg/L
Pengenceran x 20 IU/ml (RF)
Pengenceran tertinggi yang positif x 200 IU/ml (ASTO)
Pengenceran x 60 mg/L
 
5. 1. Reumatoid Arthritis 2. Covid 19 3. Demam Rematik 4. Crohn's disease (peradangan
usus)
.Kadar CRP positive (pemeriksaan untuk penyakit inflamasi) dapat ditemukan pada
Jika pilihan 1, 2, dan 3 benar
Jika pilihan 1 dan 3 benar
Jika pilihan 2 dan 4 benar
Jika semua pilihan benar
 
6. Berikut ini yang sesuai dengan hasil tes ASTO metode kualitatif adalah...
tidak terjadi aglutinasi, hasil positif
tidak terjadi aglutinasi, lanjutkan ke metode semi kuantitatif
aglutinasi positif, lanjutkan ke metode semi kuantitatif
aglutinasi positif, kadar kurang dari 200 IU/mL
 
7. 1. Penderita harus puasa 2. Penderita tidak mendapatkan pengobatan kortikosteroid 3.
penderita mendapatkan pengobatan kortikosteroid 4. Perhatikan riwayat minum obat . Hal
yang harus diperhatikan pada persiapan penderita pada pemeriksaan Sel LE adalah
Jika pilihan 1, 2, dan 3 benar
Jika pilihan 1 dan 3 benar
Jika pilihan 2 dan 4 benar
Jika semua pilihan benar
 

8. Untuk pemeriksaan RF menggunakan darah vena yang dimasukkan ke dalam tabung


vakum berwarna
merah
ungu
biru
hitam
 
9. Metode yang digunakan pada tes reumatoid faktor adalah
Imunokromatografi
Kolorimetrik Enzimatik
Aglutinasi Latex (ASTO, CRP, RF)
Pemeriksaan makroskopis
 
10. Bahan yang diperlukan pada pemeriksaan ASTO adalah kecuali
latex
kontrol positif (merah)
buffer
kontrol negatif (biru)
bahan lain : serum, Nacl 0,9%
 
11. Pada Tes RF, kontrol positif adalah:
Serum yang mengandung anti RF
Serum yang mengandung RF
Serum yang mengandung constant region dari human IgG 
Semua jawaban di atas salah
 
12. Hal apa yang ingin diukur pada TES ASTO ?
Kadar glukosa dalam urin
Kadar beta HCG dalam urin
Kadar asam urat dalam darah
Kadar Antibodi terhadap streptolisin O
 
 
13. Nilai rujukan negative untuk pemeriksaan asto adalah
< 2000 iu/ml
< 200 iu/ml
> 200 iu/ml (positif ASTO)
> 20 iu/ml (positif RF)
 
14. Untuk mendiganosis SLE, selain melakukan tes Sel LE dapat juga melalukan tes
Pemeriksaan BTA
Analisis sum sum tulang
Biopsy
ANA test (anti dsDNA, anti-sm)
 
 
15. Tes yang dapat diminta oleh dokter untuk mengetahui adanya inflamasi pada pasien
adalah kecuali
darah rutin
LED
tes golongan darah
CRP
 
16. Pada tabung merah yang telah di sentrifugasi, darah terbagi menjadi 3 komponen.
Komponen mana yg digunakan pada pemeriksaan Sel LE?
plasma
eritrosit
serum
buffy coat (lapisan leukosit)
 
 
17. Pada tes RF, yang akan dideteksi dalam sampel pasien adalah:
Autoantibodi (ada dalam sample pasien)
Autoantigen (ada dalam reagen latex)
A dan B benar
A dan C salah
 
18. Bila didapatkan kadar RF > 20, maka pada sampel ditemukan
Berubah warna menjadi merah
Bertambah encer
Muncul dua garis
Terjadi Aglutinasi
 
 
19. Test ASTO positif dapat ditemukan pada kasus di bawah ini:
Demam reumatik
Tonsilofaringitis (radang tenggorokan)
A dan B benar
A dan B salah
Termasuk : penyakit yang disebabkan Streptococcus beta hemolyticus A
 
20. Sampel yang digunakan pada pemeriksaan ini adalah
Serum (ASTO, CRP, RF)
plasma
darah kapiler
urine
 UJIAN REMEDIAL BLOK MUSKULOSKELETAL UNIVERSITAS BOSOWA TAHUN
AJARAN 2020/2021

1. Apa suplemen yang disarankan baru diberikan pada Osteoarthritis sedang sampai
berat karena ikut membantu meringankan rasa sakit ?
a) Omega-3
b) Vitamin D
c) Vitamin C
d) Glucosamine + Chondroitin Sulphate
e) NSAIDs
 
 
2. Bayi laki-laki berusia 3 bulan di bawah oleh ibunya ke poliklinik dengan keluhan
kelainan bentuk pada kedua kaki. Pada pemeriksaan fisik ditemukan sesuai
gambaran klinis berikut. Apakah diagnosis pada pasien tersebut?

a) Developmental dysplasia of the Hip-> Developmental displasia of the hip adalah


pertumbuhan abnormal dari hip yang meliputi subluksasi
caput femur, displasia acetabulum, dan dislokasi caput femur dari acetabulum.
b) Achondroplasia
c) Riketsia
d) Ostemalasia -> Osteomalacia disebabkan oleh tidak sempurnanya proses
perkembangan tulang, sehingga tulang tidak mengeras(mudah patah). Hal ini
terjadi akibat tubuh kekurangan kalsium, fosfor, atau vitamin D. 

e) Conginetal Talipes Equinovarus -> Congenital Talipes Equinovarus (CTEV)


adalah merupakan kelainan yang dibawa sejak lahir (kongenital). CTEV adalah
kelainan yang sering pula disebut Club Foot. Kelainan ini meliputi fleksi dari
pergelangan kaki, inversi dari tungkai, adduksi kaki depan dan rotasi media dari
tibia.

 
3. Seorang perempuan berusia 56 tahun datang ke praktek dokter dengan keluhan nyeri
panggul yang dialami 1 bulan terakhir. Panggul terasa kaku pada pagi hari.
Pemeriksaan fisik didapatkan tanda vital dalam batas normal dan Indeks Massa
Tubuh 30 hasil laboratorium WBC 6600/mm2, Hb 12 mg/dl, trombosit
200.000/mm2 dan LED 15 mm/jam.Apa pemeriksaan lanjutan untuk menegakkan
diagnosis pada pasien ini?
a) Reumatoid faktor
b) X-Ray
c) PET-Scan
d) CT-Scan
e) Tes Imunologi
 
4. Seorang laki-laki berusia 30 tahun datang ke klinik dengan keluhan nyeri saat
bergerak dan terasa kaku pada pinggang serta panggul. Pemeriksaan fisik ditemukan
tanda vital dalam batas normal dan tes schober positif ( Pemeriksaan tes
Schober dilakukan dengan membuat tanda pada sendi lumbosakral dan diberi
diberi tanda kedua pada jarak 10 cm diatasnya. Pasien membungkuk dan).
Apa Diagnosis pada kasus ini?

a) Sindroma Reiter -> Sindrom Reiter atau atau arthritis reaktif adalah nyeri sendi
disertai pembengkakan yang dipicu oleh infeksi pada bagian lain dari tubuh
b) Artritis Psoriatik->  penyakit autoimun, yaitu sistem kekebalan tubuh mulai
menyerang sel dan jaringan sehat. Psoriatic Arthritis (PsA) umumnya
menunjukkan gejala yang mirip tetapi biasanya disertai dengan psoriasis pada
kulit.
c) Spondilitis ankilosa -> penyakit peradangan sendi, terutama pada sendi tulang
belakang. Penyakit ankylosing spondylitis bersifat kronis atau jangka panjang, dan
umumnya dialami oleh remaja atau dewasa muda
d) Skelerosis sistemik-> Skleroderma atau Sklerosis Sistemik (SS) adalah penyakit
jaringan ikat sistemik yang melibatkan jaringan, otot, dan organ dalam. SS
termasuk dalam penyakit autoimun, yaitu ketika jaringan tubuh yang sehat dikenali
sebagai infeksi atau substansi asing
e) reumatoid arthritis
 
5. Seorang laki-laki berusia 42 tahun, datang ke unit gawat darurat rumah sakit dengan
keluhan utama nyeri pada panggul kanan sejak 30 menit yang lalu akibat kecelakaan
lalu lintas. Dari pemeriksaan fisik didapatkan tanda-tanda vital dalam batas normal,
panjang tungkai kanan lebih pendek 2 cm dibandingkan yang kiri. Pada pemeriksaan
radiologis didapatkan gambaran seperti dibawah ini. Apakah patomekanisme yang
terjadi pada Osteonecrosis femoral head ?
a) Suplai darah terputus
b) Robekan ligament
c) Pendarahan yang massif pada sendi
d) Cidera nervus
e) Malunion pada tulang

6. Seorang laki-laki berusia 50 tahun datang ke puskesmas dengan keluhan nyeri


punggung bawah yang terutama dirasakan pada saat berubah posisi dari duduk ke
berdiri atau saat duduk lama. Keluhan ini dialami sejak 3 minggu yang lalu. Tidak
ada riwayat trauma, tidak ada riwayat penyakit metabolik dan riwayat demam
disangkal. Pada pemeriksaan fisis didapatkan adanya defisit sensoris dan penurunan
refleks fisiologis tungkai bawah. Apakah kemungkinan diagnosis pada pasien ini ?
a) Muscle spasme
b) Radikulopati
c) Miopati
d) Spondylosis
e) Spondyloarthropati
 
 
7. Apakah nama nervus yang menginnervasi otot-otot hamstring?
a) N. Ischiadicus
b) N. Iliohypogastricus
c) N. Ilioinguinalis
d) N. Genitofemoralis
e) N. Gluteus superior
 
8. Anak perempuan berusia 15 tahun diantar oleh orang tuanya ke klinik dengan batuk,
nafsu makan kurang, keringat malam, berat badan menurun. Saat ini datang dengan
keluhan nyeri punggung. Pada pemeriksaan terdengar bunyi pernapasan yang
mencurigakan adanya tanda-tanda bronchopneumonia kedua paru. Dan pada
pemeriksaan tulang belakang ada terlihat scoliosis dan gibbus pada torakolumbal.
Apa pemeriksaan radiologi konvensional yang dilakukan untuk menegakkan
diagnosis pada kasus ini?
a) Torakolumbal posisi AP dan lateral
b) Torakolumbal posisi AP + obliq
c) Torakolumbal posisi lateral
d) Torakolumbal posisi obliq
e) Torakolumbal posisi AP
 
9. Untuk mengurangi nyeri dan inflamasi pada penderita Osteoarthritis,apa
suplementasi di bawah ini bisa diberikan?
a) Omega-6 Fatty acids
b) Fructooligosaccharide
c) Omega-3 fatty acids
d) Glucosamine
e) Ekstrak ikan gabus

10. Seorang laki-laki berusia 55 tahun mengeluh sesak napas sesak setelah
mengkonsumsi analgetik. untuk nyeri bahu kanan yang di alaminya. Pasien
memiliki riwayat asma sejak kecil. Apakah obat yang kemungkinan dikonsumsi
oleh pasien tersebut?
a) Asam Asetil Salisilat
b) Kodein
c) Tramadol
d) Fentanyl
e) Morfin

11. Anak perempuan berusia 15 tahun diantar oleh orang tuanya ke klinik dengan batuk,
nafsu makan kurang, keringat malam, berat badan menurun. Saat ini datang dengan
keluhan nyeri punggung. Pada pemeriksaan terdengar bunyi pernapasan yang
mencurigakan adanya tanda-tanda bronchopneumonia kedua paru. Dan pada
pemeriksaan tulang belakang ada terlihat scoliosis dan gibbus pada
torakolumbal.Apa tanda radiologi yang didapatkan pada kasus ini?
a) Cold abses / abses paravertebral
b) Onion skin appearance
c) Sunburst appearance
d) Punch out lessions
e) Codman triangle
 
12. Wanita usia 56 tahun datang dengan keluhan tangan kanannya sering kram terutama
pada pagi hari, dirasakan kurang lebih 2 bulan yang lalu. Pada pemeriksaan fisis
didapatkan adanya atropi pada otot thenar, hipostesi pada jari II,III dan IV. Apa tipe
kelainan saraf pada kasus ini ?
a) polineuropati
b) mononeuropati
c) mononueritis
d) polineuritis
e) buka salah satu diatas
 
13. Seorang perempuan berusia 56 tahun datang ke praktek dokter dengan keluhan nyeri
panggul yang dialami 1 bulan terakhir. Panggul terasa kaku pada pagi hari.
Pemeriksaan fisik didapatkan tanda vital dalam batas normal dan Indeks Massa
Tubuh 30 hasil laboratorium WBC 6600/mm2, Hb 12 mg/dl, trombosit
200.000/mm2 dan LED 15/-.Apa pemeriksaan lanjutan untuk menegakkan diagnosis
pada pasien ini?
a) Kelainan sel T
b) Kelainan sel B
c) Kelainan sel kondrosit
d) Kelainan sel T helper
e) Kelainan remodelling tulang
 
 
14. Seorang Laki-laki berusia 25 tahun datang ke dokter dengan keluhan demam tinggi
dengan bengkak dan nyeri tungkai bawah kiri. Riwayat kecelakaan, yakni pasien
ditabrak motor sekitar 1 minggu yang lalu dan menyebabkan luka pada tungkai kiri.
Pada pemeriksaan fisik didapatkan tekanan darah 110/70 mmHg, nadi 92 kali/menit,
frekuensi napas 20 kali/menit, dan suhu 37.5 C. Pemeriksaan laboratorium
didapatkan leukositosis, lalu dikirim ke Bagian Radiologi untuk pemeriksaan
radiologi. Hasil pemeriksaan radiologis ternyata ditemukan destruksi osteolitik 1/3
distal dengan soft tissue swelling. Apa gambaran radiologis yang mungkin
ditemukan pada kasus ini?
a) Osteomalasea
b) Osteosklerotik
c) Osteoporosis
d) Defect osteosklerotik dan osteolitik
e) Destruksi osteolitik daerah metafisis
 
 
15. Anak perempuan umur 15 tahun datang diantar oleh orang tuanya ke klinik dengan
keluhan batuk, nafsu makan kurang, keringat malam, berat badan menurun. Pasien
juga mengeluh nyeri pada punggungnya. Pada pemeriksaan fisik ditemukan tanda
vital normal dan terdengar bunyi pernapasan tambahan yang mencurigakan adanya
tanda-tanda bronchopneumonia kedua paru. Pada pemeriksaan tulang belakang ada
terlihat scoliosis dan gibbus pada torakolumbal. Apa tiga tipe kerusakan korpus
vertebra yang mungkin terjadi pada kasus ini?
a) Tipe marginal, tipe osteolitik dan tipe sklerotik
b) Tipe marginal, tipe sentral dan tipe anterior
c) Tipe marginal, tipe sentral dan tipe osteolitik
d) Tipe perifer, tipe marginal dan tipe osteolitik
e) Tipe sentral, tipe perifer dan tipe anterior

16. Wanita usia 56 tahun datang dengan keluhan tangan kanannya sering kram terutama
pada pagi hari, dirasakan kurang lebih 2 bulan yang lalu. Pada pemeriksaan fisis
didapatkan adanya atropi pada otot thenar, hipostesi pada jari II,III dan IV. Saraf
apakah yang terkenan pada kasus ini?
a) Radialis
b) Medianus
c) Ulnaris
d) Posterior interosseus
e) Anterior interosseus
 
17. Seorang laki-laki 29 tahun datang ke poliklinik dengan keluhan tidak dapat
mengangkat pergelangan tangan kanan sejak 6 bulan lalu. Pasien memeiliki riwayat
patah tulang lengan atas kanan akibat kecelakaan dan tidak dioperasi. Pada
pemeriksaan fisik didapatkan seperti gambaran berikut.Apa Cedera saraf yang
paling mungkin terjadi pada kasus ini ?

a) Radial
b) Ulnar
c) Median
d) Muskulokutaneus
e) Axila
 
18. Faktor diet dibawah ini bisa mengakibatkan peningkatan produksi sekaligus
mengurangi ekskresi asam urat dalam tubuh adalah?
a) Konsumsi fruktosa berlebihan
b) Konsumsi alkohol berlebihan
c) Dehidrasi
d) Konsumsi daging
e) Konsumsi jeroan
f) Seorang laki-laki berusia 50
 
19. Wanita usia 56 tahun datang dengan keluhan tangan kanannya sering kram terutama
pada pagi hari, dirasakan kurang lebih 2 bulan yang lalu. Pada pemeriksaan fisis
didapatkan adanya atropi pada otot thenar, hipostesi pada jari II,III dan IV. Apa
pemeriksaan khusus untuk kasus ini ?
a) Phalen test
b) lachman test
c) drawer test
d) thompson test
e) fromen test
 
20. Seorang perempuan berusia 45 tahun datang ke puskesmas dengan keluhan utama
nyeri dan bengkak pada pergelangan tangan kiri sejak 1 jam yang lalu. Dari
anamnesis diketahui pasien terjatuh dengan tangan kiri menumpu badan. Pada
pemeriksaan fisik didapatkan bengkak dan gambaran tulang seperti bentuk garpu.
Pasien juga mengeluhkan hilangnya rasa pada perabaan di telapak tangan kirinya.
pada pemeriksaan radiologi didapatkan gambaran sebagai berikut. Apakah diagnosis
yang paling mungkin pada kasus ini ?

a) Fraktur tertutup tulang Ulna


b) Fraktur tertutup tulang Metacarpal
c) Fraktur tertutup tulang scaphoid
d) Fraktur tertutup tulang Radius
e) Frakur tertutup tulang Capitatum
 
21. Apakah nama nervus yang merupakan percabangan N. Ischiadicus yang berjalan di
lateral os fibula?
a) N. Tibialis
b) N. Peroneus communis
c) N. Femoralis
d) N. Cutaneus femoris lateralis
e) N. Obturatorius

22. Seorang laki-laki berusia 38 tahun dibawa oleh keluarganya ke unit gawat darurat
rumah sakit dengan keluhan utama sulit menggerakan tungkai kiri setelah
mengalami kecelakaan kendaraan bermotor 1 jam yang lalu. Dari pemeriksaan fisik
didapatkan tanda-tanda vital dalam batas normal, dan tampak deformitas tungkai
kiri berupa flexion, adduksi dan internal rotation. Apakah sendi yang paling
mungkin mengalami kelainan ?
a) Sendi tumit
b) Sendi panggul
c) Sendi lutut
d) Sendi lumbo-sacral
e) Sendi simphisis pubis
 
23. Seorang perempuan berusia 45 tahun datang ke puskesmas dengan keluhan utama
nyeri dan bengkak pada pergelangan tangan kiri sejak 1 jam yang lalu. Dari
anamnesis diketahui pasien terjatuh dengan tangan kiri menumpu badan. Pada
pemeriksaan fisik didapatkan bengkak dan gambaran tulang seperti bentuk garpu.
Pasien juga mengeluhkan hilangnya rasa pada perabaan di telapak tangan kirinya.
Apakah klasifikasi yang sering digunakan pada kasus fraktur distal radius ?
a) Klasifikasi Tshcerne
b) Klasifikasi Frykman
c) Klasifikasi Gustilo-Anderson
d) Klasifikasi Le Fort
e) Klasifikasi Wagner
 
24. Wanita usia 56 tahun datang dengan keluhan tangan kanannya sering kram terutama
pada pagi hari, dirasakan kurang lebih 2 bulan yang lalu. Pada pemeriksaan fisis
didapatkan adanya atropi pada otot thenar, hipostesi pada jari II,III dan IV. Apa
diagnosis yang tepat pada kasus ini ?
a) cubital tunnel syndrome
b) carpal tunnel syndrome
c) de quervain syndrome
d) trigger finger
e) wartenberg syndrome
 
25. Seorang laki-laki berusia 38 tahun dibawa oleh keluarganya ke unit gawat darurat
rumah sakit dengan keluhan utama sulit menggerakan panggul kiri setelah
mengalami kecelakaan kendaraan bermotor 1 jam yang lalu. Dari pemeriksaan fisik
didapatkan tanda-tanda vital dalam batas normal, dan tampak tungkai bawah
mengalami deformitas berupa hip flexion, adduksi dan internal rotation. Apakah
jenis sendi yang mengalami gangguan diatas ?
a) Sendi putar
b) Sendi pelana
c) Sendi peluru
d) Sendi engsel
e) Sendi luncur
 
26. Seorang pria berusia 70 tahun mengeluh demam dan batuk yang disebabkan oleh
influenza. Pasien memiliki riwayat gastritis akut satu bulan yang lalu. Apakah
pilihan antipiretik yang efektif dan aman untuk pria tersebut?
a) Ibuprofen
b) Paracetamol
c) Asam mefenamat
d) Kalium diklofenak
e) Piroxicam
 
27. Seorang laki-laki, 20 tahun jatuh dari motor ke sisi kiri jalan, bengkak dan memar
pada pergelangan tangan kiri. Pada pemeriksaan fisis didapatkan status general
dalam batas normal, pada X Ray area wrist terdapat Fraktur distal Os. Radius.
Berapa lamakah prediksi waktu penyembuhan tulang (expected time of bone
healing) pada kasus di atas ?
a) 3-4 minggu
b) 4-6 minggu
c) 6-8 minggu
d) 10-12 minggu
e) 12-16 minggu
 
28. Seorang laki-laki berusia 20 tahun jatuh dari motor ke sisi kiri jalan, bengkak dan
memar pada pergelangan tangan kiri, jari-jari tangan sakit bila gerakkan. Pada
pemeriksaan fisis didapatkan status general dalam batas normal dan pemeriksaan
radiologis didapatkan fraktur Os. Ulna. Pada kasus di atas, apakah yang TIDAK
termasuk tujuan utama penatalaksanaan rehabilitasi medis?
a) Mempertahankan luas gerak sendi
b) Mempertahankan kekuatan otot
c) Fasilitasi penyembuhan tulang
d) Mengurangi nyeri
e) Segera memberikan latihan peguatan otot
 
 
29. Seorang perempuan berusia 52 Tahun, datang ke Puskesmas dengan keluhan nyeri
lutut kanan, dirasakan memberat saat beraktivitas dan berkurang saat istirahat.
Pasien mengeluhkan kaku pada sendi lutut pada pagi hari . Riwayat trauma
disangkal. Pada pemeriksaan fisis ditemukan krepitasi pada lutut kanan. Pada
pemeriksaan laboratorium ditemukan LED 18 mm/jam. Apakah terapi non
farmakologi yang disarankan pada kasus di atas?
a) Kompres dingin selama 15 menit
b) Bersepeda selama 30 menit
c) Jogging selama 20 menit
d) Yoga selama 20 menit
e) Treadmill selama 30 menit

30. Level Vitamin D dalam darah selama masa anak sampai remaja menentukan Peak
Bone Mass (PBM) pada awal usia 20-an. Untuk mendapatkan Vitamin D yang
cukup, dianjurkan untuk mendapat pemaparan sinar matahari (exposure) 3-4 kali
seminggu selama ?
a) 5-10 menit/hari
b) 15-30 menit/hari
c) 45-60 menit/hari
d) 45-60 menit/hari
e) 60-100 menit/hari
 
31. 1. Sarkomer memendek 2. Golgi tendon tertarik 3. Tidak ada gerakan persendian 4.
Muscle spindle tertarik Apa hal-hal yang terjadi pada kontraksi isometris?
a) 1,2, dan 3 benar
b) 1 dan 3 benar
c) 2 dan 4 benar
d) 4 benar
e) Semua benar
 
32. Apa pewarnaan yang digunakan khusus untuk serat kolagen?
a) Giemsa
b) Bielshowsky
c) Romanosky
d) Von gieson
e) Toluidin Blue

33. Seorang perempuan usia 35 tahun datang ke klinik dengan keluhan nyeri pada jari-
jari kedua tangan yang dirasakan sejak 1 bulan terakhir, dan terasa kaku pada pagi
hari. Pasien juga mengeluh lemah sepanjang hari. Pada pemeriksaan fisik
didapatkan tekanan darah 120/80 mmHg, nadi 88 kali/menit, pernapasan 20
kali/menit, dan suhu 37.4 C. Nampak tanda-tanda inflamasi pada PIP, MCP dan
pergelangan ke dua tangan dan nyeri tekan +. Apa Pemeriksaan laboratorium yang
dapat membantu penegakkan diagnosis pada kasus ini?
a) ANA tes
b) Anti dsDNA tes
c) RF tes
d) LED
e) SGOT
 
34. Seorang laki-laki berusia 50 tahun datang ke puskesmas dengan keluhan nyeri
punggung bawah yang terutama dirasakan pada saat berubah posisi dari duduk ke
berdiri atau saat duduk lama. Keluhan ini dialami sejak 3 minggu yang lalu. Tidak
ada riwayat trauma, tidak ada riwayat penyakit metabolik dan riwayat demam
disangkal. Pemeriksaan fisik apakah yang dapat dilakukan untuk mengetahui adanya
keterlibatan saraf pada kasus ini ?
a) Tes Straight Leg Raising
b) Tes Thomas
c) Tes Gaenslen
d) Tes Patrick
e) Tes Pelvic rock
 
35. Seorang laki-laki berusia 25 Tahun, datang ke klinik dengan keluhan bengkak dan
nyeri pada lutut kanan setelah terjatuh saat bermain sepak bola. Lutut tidak
dirasakan bergoyang apabila berjalan. Tidak ada riwayat penyakit metabolik dan
hipertensi. Apakah Test khusus pada lutut yang dapat dilakukan pada kasus di atas?
a) Mc. Murray, Lachman test, Posterior drawer Test, Varus test
b) Thomas test, Mc. Murray, Lachman test, Posterior drawer Test
c) Mc. Murray, Lachman test, Posterior drawer Test, Thompson test
d) Thomas test, Mc. Murray, Lachman test, Posterior drawer Test, Finkelstein test
e) Lachman test, Posterior drawer Test, Thompson test, Thomas test
 
36. Seorang laki-laki berusia 59 tahun datang ke klinik dengan keluhan nyeri hebat pada
sendi tumit dan ibu jari kaki kanan. Keluhan yang sama pernah dialami 2 tahun yang
lalu pd sendi ibu jari kaki kanan. Pemeriksaan fisik ditemukan tanda vital dalam
batas normal dan tanda-tanda inflamasi pada sendi-sendi MTP 1 kanan serta nyeri
tekan. Apakah diagnosa pada kasus tersebut diatas?

a) Hiperurisemia
b) Gout Intermitten akut
c) Gout kronik bertofi
d) Hiperurisemia asimptomatik
e) Poliarthritis nodosa
 
37. Seorang laki-laki berusia 60 tahun, obesitas, datang dengan keluhan OA genu
dextra. Setelah dilakukan pemeriksaan fisik, tekanan darah 160/90 mmHg. Riwayat
ulkus peptikum dan konstipasi. Dokter kemudian memberikan obat penurun tekanan
darah Amlodipine 5 mg dosis 1dd1. Apa farmakoterapi nyeri yang terpilih dan aman
diberikan pada kasus diatas adalah?
a) meloxicam
b) Kodein
c) Ketorolac
d) Kalium diklofenak
 
 
38. Seorang perempuan berusia 35 tahun datang ke puskesmas dengan keluhan nyeri
kepala sejak 3 hari yang lalu. Pasien diberikan obat Ibuprofen untuk meredakan
nyeri. Bagaimanakah mekanisme kerja obat tersebut?
a) Menghambat enzim cyclooxigenase
b) Menghambat enzim lipoksigenase
c) Menghambat enzim fosfolipase
d) Menghambat enzim lipase
e) Menghambat enzim Na+/K+ ATP-ase
 
39. Seorang perempuan berusia 52 Tahun, datang ke Puskesmas dengan keluhan nyeri
lutut kanan, dirasakan memberat saat beraktivitas dan berkurang saat istirahat.
Pasien mengeluhkan kaku pada sendi lutut pada pagi hari . Riwayat trauma
disangkal. Pada pemeriksaan fisis ditemukan krepitasi pada lutut kanan. Pada
pemeriksaan laboratorium ditemukan LED 18 mm/jam. Edukasi apakah yang dapat
kita sarankan kepada pasien di atas?
a) Menggunakan toilet jongkok
b) Naik tangga dengan lutut kanan terlebih dahulu
c) Naik tangga dengan kaki kiri terlebih dahulu
d) Duduk bersila
e) Jogging di pagi hari
 
40. Seorang laki-laki berusia 59 tahun datang ke klinik dengan keluhan nyeri hebat pada
sendi tumit dan ibu jari kaki kanan. Keluhan yang sama pernah dialami 2 tahun yang
lalu pd sendi ibu jari kaki kanan. Pemeriksaan fisik ditemukan tanda vital dalam
batas normal dan tanda-tanda inflamasi pada sendi-sendi MTP 1 kanan serta nyeri
tekan. Bagaimana patomekanisme yang terjadi pada kasus tersebut?
a) Trauma langsung pada sendi tersebut.
b) Lepasnya kristal MSU dari depositnya
c) Pengendapan kristal MSU dalam depositnya
d) Hiperurisemia
e) Terbentuknya kristal kalsium
 
41. Seorang laki-laki berusia 25 Tahun, datang ke klinik dengan keluhan bengkak dan
nyeri pada lutut kanan setelah terjatuh saat bermain sepak bola. Lutut tidak
dirasakan bergoyang apabila berjalan. Tidak ada riwayat penyakit metabolik dan
hipertensi. Apakah edukasi yang mungkin yang dapat kita berikan pada pasien di
atas untuk dilakukan di rumah?
a) mengistirahatkan, kompres es, bandaging, elevasi
b) mengistirahatkan, kompres panas, bandaging, elevasi
c) kompres dingin, mengaktifkan lutut yang yang cidera
d) kompres panas, mengaktifkan lutut yang cidera
e) Kompres dingin, bandaging, mengaktifkan lutut yang cidera
 
42. Apakah nama nervus yang menginnervasi otot-otot quadriceps femoris?
a) N. Tibialis
b) N. Peroneus communis
c) N. Femoralis
d) N. Cutaneus femoris lateralis
e) N. Obturatorius
 
43. Apa peran ion Ca pada neuromuscular junction?
a) Membantu proses eksositosis neurotransmiter
b) Membuka pintu saluran ion Na
c) Membuka pintu saluran ion Ca
d) Membuka pintu saluran K
e) Membantu proses sintesis neurotransmitter

 
44. Seorang laki-laki 34 tahun datang ke poliklinik dengan keluhan tidak dapat
mengangkat kaki kanan sejak 8 bulan lalu. Pasien memeiliki riwayat patah tulang
disekitar lutut dan tulang kering akibat kecelakaan dan tidak dioperasi. Pasien
mengaku ditabrak dari samping saat mengendarai motor. Pada pemeriksaan fisik
didapatkan seperti gambaran berikut. Cedera saraf yang paling mungkin terjadi pada
soal no.3 adalah?

a) Saphenus
b) Sural
c) Sciatic
d) Tibialis
e) Peroneal
 
45. Di mana terdapat anyaman penyambung glatineus ?
a) Paru-paru
b) Gaster
c) Tali pusar
d) Kulit
e) Ileum

46. Penulangan endokondral zone III ditandai oleh?


a) Sel kondrosit mengalami pengapuran
b) Sel kondrosit mengatur diri secara linier
c) Sel kondrosit membesar
d) Sel kondrosit rusak
e) Sel kondrosit menyebar
 
47. Seorang laki-laki berusia 24 tahun datang ke puskesmas dengan keluhan nyeri pada
kaki kanan setelah terkilir ketika main footsal dengan posisi ankle inversi. Nyeri
yang dikeluhkan mengakibatkan pasien sulit untuk berjalan. Apakah tindakan yang
dapat kita lakukan pada pasien diatas ?
a) mengistirahatkan, kompres es, membandaging, sedikit meng-elevasi
b) mengistirahatkan, kompres panas, membandaging, sedikit meng-elevasi
c) kompres dingin, mengaktifkan ankle yang yang cidera
d) kompres panas, mengaktifkan ankle yang cidera
e) kompres dingin, bandaging, mengaktifkan ankle yang cidera
 
48. Seorang laki-laki berusia 50 tahun datang ke puskesmas dengan keluhan nyeri
punggung bawah yang terutama dirasakan pada saat berubah posisi dari duduk ke
berdiri atau saat duduk lama. Keluhan ini dialami sejak 3 minggu yang lalu. Tidak
ada riwayat trauma, tidak ada riwayat penyakit metabolik dan riwayat demam
disangkal. Pada kasus ini, gejala apakah yang dapat muncul dan menjadi perhatian
khusus ?
a) Hamtring Tightness
b) Nyeri saat perubahan posisi
c) Spasme otot paralumbal
d) Defisit sensoris
e) Nyeri punggung bawah terlokalisasi
 
 
49. Seorang laki-laki 34 tahun datang ke poliklinik dengan keluhan tidak dapat
mengangkat kaki kanan sejak 8 bulan lalu. Pasien memeiliki riwayat patah tulang
disekitar lutut dan tulang kering akibat kecelakaan dan tidak dioperasi. Pasien
mengaku ditabrak dari samping saat mengendarai motor. Pada pemeriksaan fisik
didapatkan seperti gambaran berikut. Apakah diagnosis yang paling mungkin pada
pasien ini?

a) Clawhand
b) Clawfoot
c) Dropfoot
d) Leg discrepancy
e) drophand
 
50. Seorang laki-laki umur 67 tahun datang ke klinik dengan keluhan nyeri dan bengkak
serta kemerahan pada ibu jari kaki dan tidak bisa memakai sepatu. Tidak ada
riwayat demam sebelumnya. Hasil pemeriksaan laboratorium ditemukan
peningkatan asam urat dan tidak ada leukositosis. Apakah pemeriksaan foto
radiologi kaki yang diminta untuk menegakkan diagnosis kasus ini? (diagnosis
Gout)
a) Posisi foto anterior posterior + obliq
b) Posisi foto anterior posterior + lateral
c) Posisi foto posterior anterior + obliq
d) Posisi foto anterior posterior
e) Posisi lateral
 
51. Seorang laki-laki, 20 tahun jatuh dari motor ke sisi kiri jalan, telah dilakukan
pemasangan pen pada Os. Tibia kiri 2 minggu yang lalu. Pada pemeriksaan fisis
didapatkan status general dalam batas normal. Apakah alat bantu jalan yang paling
ideal pada pasien di atas?
a) Tripod cane
b) Quadripod cane
c) Single axillary crutch
d) double axillary crutch
e) wheelchair
 
52. Seorang laki-laki berusia 50 tahun datang ke puskesmas dengan keluhan nyeri
punggung bawah yang terutama dirasakan pada saat berubah posisi dari duduk ke
berdiri atau saat duduk lama. Keluhan ini dialami sejak 3 minggu yang lalu. Tidak
ada riwayat trauma, tidak ada riwayat penyakit metabolik dan riwayat demam
disangkal. Bagaimanakah posisi tubuh berikut ini yang memiliki tekanan intradiskal
tertinggi saat kita mengedukasi pasien ?
a) Berbaring terlentang
b) Berbaring tengkurap
c) Duduk
d) Duduk bungkuk
e) Berdiri bungkuk
 
53. Seorang perempuan berusia 52 Tahun, datang ke Puskesmas dengan keluhan nyeri
lutut kanan, dirasakan memberat saat beraktivitas dan berkurang saat istirahat.
Pasien mengeluhkan kaku pada sendi lutut pada pagi hari . Riwayat trauma
disangkal. Pada pemeriksaan fisis ditemukan krepitasi pada lutut kanan. Pada
pemeriksaan laboratorium ditemukan LED 18 mm/jam. Apakah diagnosis yang
paling tepat pada kasus ini ?
a) Rheumatoid Arthritis
b) Osteoarthritis
c) Gout Arthritis
d) Gonathrosis
e) Bursitis Genu
 
54. Seorang perempuan usia 35 tahun datang dengan keluhan nyeri pada jari-jari kedua
tangan yang dirasakan sejak 1 bulan terakhir, dan terasa kaku pada pagi hari. Pasien
juga mengeluh lemah sepanjang hari. Pada pemeriksaan fisik didapatkan tekanan
darah 120/80 mmHg, nadi 88 kali/menit, pernapasan 20 kali/menit, dan suhu 37.4 C.
Nampak tanda-tanda inflamasi pada PIP, MCP dan pergelangan ke dua tangan dan
nyeri tekan +. Apakah diagnosis yang tepat pada kasus ini?
a) Osteofit
b) Erosi marginal
c) Penyempitan cela sendi
d) Pengapuran sendi
e) Atropi jaringan lunak.
 
55. Seorang laki-laki 29 tahun datang ke poliklinik dengan keluhan tidak dapat
mengangkat pergelangan tangan kanan sejak 6 bulan lalu. Pasien memeiliki riwayat
patah tulang lengan atas kanan akibat kecelakaan dan tidak dioperasi. Pada
pemeriksaan fisik didapatkan seperti gambaran berikut. Apakah diagnosis yang
paling mungkin pada pasien ini?

a) Clawhand
b) Drophand
c) Clawfoot
d) Dropfoot
e) Benedict hand
 
56. Seorang perempuan berusia 52 Tahun, datang ke Puskesmas dengan keluhan nyeri
lutut kanan, dirasakan memberat saat beraktivitas dan berkurang saat istirahat.
Pasien mengeluhkan kaku pada sendi lutut pada pagi hari . Riwayat trauma
disangkal. Pada pemeriksaan fisis ditemukan krepitasi pada lutut kanan. Pada
pemeriksaan laboratorium ditemukan LED 18 mm/jam. Modalitas terapi apakah
yang paling tepat diberikan pada kasus ini?
a) Low Level Laser Teraphy dan Short Wave Diathermy
b) Short Wave Diathermy dan Micro Wave Diathermy
c) Low Level Laser Teraphy dan Micro Wave Diathermy
d) Stimulasi elektrik dan Low Level Laser Teraphy
e) cing dan Micro Wave Diathermy
 
57. Seorang perempuan berusia 45 tahun datang ke puskesmas dengan keluhan utama
nyeri dan bengkak pada pergelangan tangan kiri sejak 1 jam yang lalu. Dari
anamnesis diketahui pasien terjatuh dengan tangan kiri menumpu badan. Pada
pemeriksaan fisik didapatkan bengkak dan gambaran tulang seperti bentuk garpu.
Pasien juga mengeluhkan hilangnya rasa pada perabaan di telapak tangan kirinya.
Apakah penanganan awal yang paling mungkin diberikan ?
a) Reduksi tertutup dan pemasangan casting
b) Imobilisasi dengan menggunakan casting
c) Imobilisasi dengan spalk
d) Pemberian analgetik
e) Pemberian antibiotik
 
58. Seorang perempuan berusia 24 tahun datang ke klinik dengan keluhan kemerahan
pada wajah dan nyeri pada sendi-sendi sejak 2 minggu terakhir. Sebelumnya, pasien
juga mengeluh lemah badan dalam 3 bulan terakhir. Pemeriksaan fisik didapatkan
tanda vital dalam batas normal dan terdapat malar rash dan artritis pada jari-jari
tangan dan lengan. Apa pemeriksaan selanjutnya untuk mendiagnosa penyakit pada
penderita tersebut? (diagnose SLE)
a) ANA tes
b) ANA tes dan anti ds DNA tes
c) ANA tes dan RF tes
d) ANA tes dan Radiologi
e) Anti ds DNA tes
 
59. Level Vitamin D dalam darah selama masa anak sampai remaja menentukan Peak
Bone Mass (PBM) pada awal usia 20-an. Untuk mendapatkan Vitamin D yang
cukup, boleh dianjurkan untuk mendapat pemaparan sinar matahari (exposure)
hanya pada jam tertentu saja selama siang hari untuk menghindari paparan UV yang
terlalu kuat. Kapan aktu yang terbaik adalah untuk berjemur?
a) Sebelum jam 10.00 dan setelaj jam 15.00
b) Sebelum jam 08.00 dan setelah jam 16.00
c) Sebelum jam 12.00 dan setelah jam 15.00
d) Sebelum jam 07.00 dan setelah jam 17.00
 
60. Seorang perempuan usia 35 tahun datang dengan keluhan nyeri pada jari-jari kedua
tangan yang dirasakan sejak 1 bulan terakhir, dan terasa kaku pada pagi hari. Pasien
juga mengeluh lemah sepanjang hari. Pada pemeriksaan fisik didapatkan tekanan
darah 120/80 mmHg, nadi 88 kali/menit, pernapasan 20 kali/menit, dan suhu 37.4 C.
Nampak tanda-tanda inflamasi pada PIP, MCP dan pergelangan ke dua tangan dan
nyeri tekan +. Apa rencana pengobatan yang tepat untuk kasus ini?
a) DMARDs
b) DMOADs
c) Vitamin
d) Analgetik dan DMORDs
e) Analgetik dan DMARDs

1. osteoporosis  timbul akibat ketidak seimbangan antara fungsi sel dalam tulang, yaitu..
a. Fungsi osteoblast meningkat-osteoklast normal
b. Fungsi osteoblast meningkat-osteoklast meningkat
c. Fungsi osteoblast meningkat-osteoklast menurun
d. Fungsi osteoblast normal-osteoklast meningkat
e. Fungsi osteoblast menurun-osteoklast menurun
2. Seorang laki-laki berusia 50 tahun datang ke puskesmas dengan keluhan nyeri punggung
bawah yang terutama dirasakan pada saat berubah posisi dari duduk ke berdiri atau saat
duduk lama. Keluhan ini dialami sejak 3 minggu yang lalu. Tidak ada riwayat trauma, tidak
ada riwayat penyakit metabolic, dan riwayat demam disangkal.
Pemeriksaan fisik apakah yang dapat dilakukan untuk mengetahui adanya keterlibatan saraf
pada kasus ini? PPT LOW BACK PAIN HAL.33
a. Tes straight les raising
b. Tes Thomas (kontraksi fleksi panggul)
c. Tes gaenslen (melihat inflamasi sacroiliaca)
d. Tes Patrick (FABER test)(melihat nyeri di sacroiliaca)
e. Tes pelvic rock  (melihat posisi segmental)
3. Pemeriksaan  fisik sesuai dengan gambar berikut dikenal juga sebagai test..

PPT DISPLASIA PANGGUL HAL.13


a. Barlow (to dislok the hip)
b. Ortolani (to relok the hip)
c. Galeazzi  (untuk melihat dislok)
d. Trendelenberg (gait g.medius lemah)
e. Allen test (cek arteri di wrist)
4. Tumor ganas tulang primer yang terbanyak adalah..    PPT TUMOR MUSKULOSKELETAL
HAL.6
a. Ewing sarcoma
b. Malignant limfoma
c. Osteosarcoma
d. Chondrosarkoma
e. Multiple myeloma
5. Laki-laki 17 tahun jatuh duduk setelah memanjat manga didepan rumahnya. Tiga hari
kemudian mengalami LBP. Datang ke klinik Orthopedi kemudian dikirim ke bagian Radiologi
untuk melakukan foto lumbosacral AP + lateral. Hasil pemeriksaan radiologi ternyata ada
pemipihan didaerah corpus vertebra lumbal 1 dan wedging anterior. 
Diagnose pada kasus di atas adalah..  PPT RADIOLOGI MUSKULO HAL.42
a. Fraktur depressi/impression 🡪 sering terjadi pada daerah tengkorak/skull
b. Fraktur greenstick 🡪 pada anak-anak
c. Fraktur kompressi 🡪 paling sering terjadi pada vertebra
d. Fraktur avulsi (fraktur di ujung2 tulang)
e. Fraktur linier (fraktur lurus)
6. Seorang bayi usia 6 bulan datang dibawa oleh orang tuanya untuk pemeriksaan tumbuh
kembang rutin. Pada pemeriksaan fisik didapat adanya kelainan pada panggul kanan pasien
sehingga anda meminta pemeriksaan penunjang berupa X-Ray pelvis AP dan didapatkan
hasil berikut. 
Diagnosis yang tepat adalah..  PPT DISPLASIA PANGGUL HAL. 21
a. Blount’s disease (Kaki O , Lengkung Tibia)
b. CTEV (Congenital Talipes Equina Varus)
c. DDH 
d. Scheuermann’s disease (Kifosis abnormal growth lumbar)
e. Osgood-schlatter disease (inflamasi patella)
7. Perempuan usia 70 tahun datang dengan keluhan nyeri pada punggung, dialami sejak 3
tahun terakhir dan memberat 2 bulan terakhir. Nyeri menjalar ke pantat sebelah kanan,
nyeri memberat ketika berdiri dan berjalan. Kelemahan pada kedua tungkai bawah tidak
ditemukan. Pasien tidak merasakan gangguan BAK dan BAB. Riwayat trauma disangkal oleh
pasien.
Tata laksana awal yang dapat diberikan.. PPT LOW BACK PAIN HAL. 46
a. NSAIDs, physical therapy, penurunan berat badan, bracing
b. Pemberian injeksi Steroid dan pemasangan brace
c. Tindakan operasi posterior stabilized
d. Pasang brace saja
e. Pemberian suplementasi kalsium dan vit D
8. Osteoporosis senile (laki-laki) terjadi akibat menurunnya fungsi sel.. .
a. Osteosit
b. Osteoblast
c. Osteoklast
d. Osteoprogenitor cells
e. Stormaed cells
9. Perempuan usia 70 tahun datang dengan keluhan nyeri pada punggung, dialami sejak 3
tahun terakhir dan memberat 2 bulan terakhir. Nyeri menjalar ke pantat sebelah kanan,
nyeri memberat ketika berdiri dan berjalan. Kelemahan pada kedua tungkai bawah tidak
ditemukan. Pasien tidak merasakan gangguan BAK dan BAB. Riwayat trauma disangkal oleh
pasien.
Pemeriksaan apa yang dilakukan untuk mengonfirmasi adanya iritasi pada saraf L5-S1
adalah..
a. Galleazzi test (sejajar kaki) (tk dislok panggul)
b. Lasegue test 🡪 nama lainnya TES Strech Leg raise (melihat lbp bawah saraf)
c. Femoral stretch nerve test (menekuk paha) (stess L2-L4)
d. Finger escape sign (wertenberg)myelopathy kelingkik)
e. Babinski sign (refleks plantar ) *Uppter Motor Neuron Epilepsy

10. Laki-laki 21 tahun ditabrak motor di daerah kepala bagian parietal kanan. Dikirim ke bagian
radiologi untuk pemeriksaan foto kepala AP + lateral ditemukan didaerah tersebut tulang
masuk kedalam dan asimetris.
Diagnnosa pada kasus di atas adalah… PPT RADIOLOGI MUSKULO HAL.42
a. Fraktur depression / impression 
b. Fraktur greenstick
c. Fraktur kompressi
d. Fraktur avulsi
e. Fraktur linier
11. Laki-laki umur 42 tahun datang dengan keluhan nyeri hebat pada lutut kanan, dialami
penderita sejak pagi tadi ketika bangun sholat subuh. Awalnya hanya terasa pegal-pegal
sedikit hangat pada perabaan. Riwayat trauma pada lutut kanan disangkal oleh pasien.
Tindakan yang bersifat terapeutik maupun diagnostic yang dapat dilakukan pada kasus
diatas adalah..
a. Istirahatkan sendi
b. Kompres dingin
c. Mobilisasi segera dilakukan untuk cegah kekakuan
d. Aspirasi
e. Berikan bidai sendi
12. Laki-laki umur 42 tahun datang dengan keluhan nyeri hebat pada lutut kanan, dialami
penderita sejak pagi tadi ketika bangun sholat subuh. Awalnya hanya terasa pegal-pegal
sedikit hangat pada perabaan. Riwayat trauma pada lutut kanan disangkal oleh pasien.
Nyeri yang dialami penderita ini karena.. . PPT ARTRITIS GOUT HAL. 15-17
a. Neurogenic (cedera saraf blkg)
b. Inflamasi
c. Mekanik
d. Psikogenik (karena presepsi)
e. Campuran (neurogenic, psikogenik, dan mekanik)
13. Wanita umur 55tahun dengan keluhan nyeri dan bengkak lutut kanan. Dikirim ke bagian
radiologi , dilakukan foto lutut AP + lateral, ditemukan adanya penyempitan celah sendi dan
osteofit distal femur.
Kemungkinan diagnose pada kasus di atas adalah…
a. Sinovioma
b. Osteosarcoma
c. Osteoid osteoma
d. Fraktur dan dislokasi
e. Osteoarthritis lutut
14. Laki-laki 54 tahun datang dengan keluhan utama nyeri pada siku kiri simetris yg sudah
berlangsung sejak 1 tahun lalu, awalnya dimulai dengan nyeri pada sendi-sendi dijarinya dan
pergelangan tangan. Keluhan dirasakan terutama di pagi hari. tidak ada demam, riwayat
tidak pernah jatuh.
Sel imun apakah yang menghasilkan pelepasan sitokin TNF-alfa dan IL-2 pada peradangan
sendi dari kasus diatas..
a. B cell
b. Makrofag
c. Limfosit T
d. Plasma sel
e. netrofil
15. Seorang perempuan 5 tahun dibawa ke UGD setelah jatuh saat bermain bulu tangkis.
Setelah dilakukan pemeriksaan tampak rupture pada tendo calcaneus.
Gerakan manakah dibawah ini yang tidak dapat dilakukan pada pedis?
a. Plantarflexi
b. Dorsofleksi
c. Adduksi
d. Abduksi
e. Rotasi medial
16. Laki-laki 15 tahun mengalami cedera pada tungkai bawah dibawah lututnya. Setelah
dilakukan pemeriksaan radiologi tampak fraktur pada collum fibula.
Struktur manakah yang paling mungkin mengalami trauma pada kasus diatas..
a. N. tibialis
b. N. peroneus communis🡪 setelah N.ischiadicus bercabang pada fossa poplitea, maka terbagi
menjadi N.peroneus communis disebelah lateral yang akan melingkari collum fibula dan
N.tibialis yang berjalan di medial
c. N. femoralis
d. N. cutaneous femoris lateralis
e. N. obturatorius
17. Pria usia 67tahun datang dengan keluhan luka infeksi pada pedis kanan yang dialami sejak 3
bulan yang lalu, riwayat menderita DM.
Berikut ini yang tidak termasuk alat dressing yang ideal untuk perawatan luka pasien adalah..
PPT INFEKSI MUSKULOSKELETAL HAL. 98
a. Melindungi dari invasi bakteri
b. Mempertahankan kelembapan secara optimal
c. Menyerap serum dari tempat luka
d. Melindungi jaringan granulasi
e. Mengurangi bengkak
18. Perempuan umur 24 tahun datang ke poliklinik dengan keluhan bercak kemerahan menetap
di kedua pipi yang terjadi setelah berenang di pantai 2 hari yg lalu. Sebelumnya pasien
merasakan adanya nyeri dan kaku sendi di kedua tungkai dan tangan setiap pagi. Tidak ada
riwayat penyakit seperti ini di dalam keluarga.
Hasil tes laboratorium apakah yang dapat ditemukan pada tes darah rutin pada kasus di
atas? PPT LABORATORY TES MUSCULO HAL.26
a. Anemia
b. Lekositosis
c. Limfosit dalam batas normal
d. Trombositosis
e. Eritrosit dalam batas normal
19. Seorang perempuan berusia 56 tahun datang ke puskesmas dengan keluhan nyeri pada
daerah lutut. Hal ini telah dirasakan sejak satu pecan yang lalu. Untuk mengurangi rasa nyeri
dapat diberikan suplementasi zat gizi .
Apa suplementasi tersebut?
a. Omega-9
b. Omega-6
c. Omega-5
d. Omega-3 🡪 fx omega 3 untuk membantu meningkatkan kadar Ca & deposit Ca ditulang
e. Omega-2
20. Seorang perempuan berusia 49 tahun datang ke puskesmas dengan keluhan nyeri pada lutut
dirasakan saat berjalan. Faktor yang dapat menyebabkan timbulnya keluhan nyeri adalah
obesitas. Ada 2 proses dari obesitas yang dapat menyebabkan nyeri pada lutut.
Apakah salah satu proses tersebut?
a. Makanan  yang tinggi lemak
b. Keluarnya produk inflamasi dari jaringan adipose
c. Kurangnya beraktivitas
d. Berat badan berlebih
e. Tulang mengalami osteoporosis
21. Perempuan 35 tahun dengan keluhan punggung kaki kirinya kurang rasa setelah trauma
beberapa waktu yang lalu. Setelah dokter melakukan pemeriksaan, didapatkan adanya
kekurangan sensasi pada sisi malleolus medialis.
Saraf manakah dibawah ini yang paling mungkin mengalami kelainan?
a. N. plantaris lateralis
b. N. saphenous 🡪 innervasi sensoris didaerah medial femur distal,cruris, pedis
c. N.peroneus profunda
d. N. peroneus superficialis
e. N. suralis
22. Seorang laki-laki berusia 65tahun datang dengan keluhan sering sakit kepala. Pada
pemeriksaan fisik pasien didiagnosis menderita hipertensi serta DM. dokter memberikan
obat penurun tekanan darah dan anti hiperglikemik serta obat untuk mencegah thrombosis
arteri. 
Apakah obat pencegah thrombosis golongan AINS yang cocok diberikan pada kasus 
tersebut?
a. Asam asetil salisilat (semua 3 : bisa antikoagulan)
b. Ibuprofen (obat antiinflamasi)
c. Asam mefenamat (analgetik rendah – sedang)
d. Natrium diklofenac (antiinflamasi rendah)
e. Meloksikam (peredah neri dan inflamasi)
23. Seorang ibu membawa anak perempuannya yang berusia 9 tahun dengan bengkak pada
betis kanan yang semakin lama membesar sejak 3 bulan ini.riwayat jatuh atau cedera
disangkal. Satu bulan ini anak tersebut mengeluhkan demam hilang timbul dan nyeri
sehingga enggan berjalan. Pasien kemudian dilakukan pemeriksaan radiologi, biopsy, dan
bone marrow aspiration kemudian ditegakkan diagnosis ewing sarcoma.
Gambaran reaksi periosteal khas yang ditemukan pada X-Ray kasus diatas adalah.. 
PPT RADIOLOGI MUSKULOSKELETAL HAL. 139
a. Sunburst appearance (Osteosarcoma)
b. Codman’s triangle (Ewing Sarcoma)
c. Onion skin appearance
d. Punched out appearance (Multiple Myeloma)
e. Popcorn appearance (Chondrosarcoma)
24. Bagian dari tulang panjang yang menentukan kemampuan tulang tersebut masih dapat
bertumbuh atau tidak, adalah..
a. Epiphysis
b. Diaphysis
c. Articular cartilage
d. Epiphyseal plate
e. Medullary cavity
25. Seorang perempuan 52tahun datang dengan keluhan nyeri lutut kanan, dirasakan memberat
saat beraktivitas dan berkurang saat istirahat. Pasien mengeluhkan kaku pada sendi lutut
pada pagi hari. riwayat trauma disangkal. Pada pemeriksaan fisis ditemukan krepitasi pada
lutut. Pada pemeriksaan laboratorium ditemukan LED 18mm/jam.
Pada kasus di atas kondisi apakah dapat kita berikan terapi Short Wave Diathermy?
a. Akut
b. Kronis
c. Akut dan kronis
d. Varises lutut
e. Kronis dan  varises lutut
26. Seorang laki-laki 54 tahun mengeluhkan benjolan pada paha kiri sejak 6 bulan lalu. Benjolan
semakin lama membesar dan 1 bulan terakhir ini terasa nyeri terutama saat berjalan. Dari
hasil pemeriksaan fisik ditemukan massa berukuran 10x11 cm dengan permukaan noduler,
berbatas tidak tegas dan immobile. Pasien kemudian dilakukan pemeriksaan radiologis dan
biopsy dan diagnosis liposarcoma.
Berikut ini pernyataan yang benar mengenai kasus diatas
a. Secara epidemiologi merupakan sarcoma malignan jaringan lunak yg paling sering
ditemukan
b. Lebih sering terjadi pada ekstremitas atas dibandingkan ekstremitas bawah
c. Cenderung terjadi diluar fascia
d. Lipoma merupakan faktor predisposisi terjadinya liposarcoma
e. Sering terjadi pada retroperitoneum
27. Seorang laki-laki 20 tahun mengeluh bahu kanan tidak dapat digerakkan stelah terjatuh saat
mengendarai motor. Hasil pemeriksaan radiologi ditemukan fraktur pada collum chirurgicum
humeri. 
Struktur manakah dibawah ini yg paling mungkin mengalami kelainan…
a. A. subclavia
b. A. axillaris
c. A. brachialis
d. V. brachialis
e. A. profunda brachia
28. Laki-laki 75 tahun cedera daerah axilla akibat terjatuh. Dirawat pada UGD dengan dislokasi
pad aarticulatio humeri dextra. Jika penderita tidak dapat mengekstensikan articulation
cubiti, articulation radiocarpales, dan articulation daerah manus.
Nervus apa yang terganggu..
a. N. ulnaris
b. N. musculocuaneus
c. N. medianus
d. N. axillaris
e. N. radialis
29. Perempuan 33 tahun , sejak 6 bulan terakhir mengeluh nyeri dan bengkak pada sendi
metakarpofalangeal dan interfalang proksimal di kedua tangan, pergelangan tangan, kedua
lutut dan pergelangan kaki. Pada pemeriksaan fisik tampak tanda-tanda radang pada sendi
tsb, kaku sendi dirasakan saat bangun pagi sampai siang hari. LED 94mm pada jam pertama
dan kadar urat serum 9,5 mg/dL.
Apakah farmakoterapi pilihan untuk kasus diatas?
a. Colchisisn
b. Methotrexate
c. Prednisone
d. Sulsasalazine
e. Glucosamine
30. Laki-laki 50 tahun datang dengan keluhan nyeri punggung bawah yang terutama dirasakan
pada saat berubah posisi dari duduk ke berdiri atau duduk lama .keluhan ini dialami sejak 3
minggu yang lalu. Tidak ada riwayat trauma, penyakit metabolic ataupun demam. Pada
pemeriksaan fisis didapatkan adanya deficit sensoris dan penurunan reflex fisiologis tungkai
bawah.
Apakah kemungkinan diagnosis pada pasien ini?
a. Muscle spasm
b. Radikulopati
c. Miopati
d. Spondylosis
e. Spondyloarthropi
31. Seorang perempuan 5 tahun dibawa ke unit gawat darurat setelah jatuh dari pohon dengan luka
robek multiple pada tungkai bawahnya. Nervus manakah dibawah ini yang menginnervasi kelompok
otot bagian anterior pada kasus tersebut?
a) n. Femoralis🡪 innervasi otot ventral femoris
b) n. Peroneus superficialis
c) n. Peroneus profunda🡪 innervasi otot ventral cruris
d) n. Plantaris medialis
e) n. Plantaris lateralis

32. Tes-tes laboratorium apakah yang dapat menunjang pemeriksaan pada penderita osteoporosis?
a) hormon progesteron
b) hormon estrogen
c) hormon tiroid
d) hormon paratiroid
e) hormon calcitonin

33. Perempuan 65 tahun datang dengan keluhan nyeri punggung dialami sejak 6 bulan terakhir.
Pasien riwayat terjatuh dengan posisi terduduk di dapur rumah. Nyeri mereda ketika dalam posis
duduk. Pada pemeriksaan fisis ditemukan kelemahan pada waktu dorsoflexi pergelangan kaki kanan.
Hipostesi pada 1st web space kaki kanan. Pada pemeriksaan xRay ditemukan gambaran 
Xray kasus diatas memberikan gambaran
a) spondylosis
b) spondilitis
c) spondilodiscitis
d) spondilolistesis
e) spondilolisis
34. Anak perempuan umur 15 tahun dengan batuk, nafsu makan kurang, keringat malam, BB
menurun. Saat ini datang dengan keluhan nyeri punggung. Pada pemeriksaan terdengar bunyi
pernapasan yang mencurigakan adanya tanda tanda bronchopneumonia kedua paru. Dan
pemeriksaan tulang belakang ada terlihat sclerosis dan gibbus pada torakolumbal.
Pemeriksaan radiologi konvensional yang dilakukan adalah foto :
a. Torakolumbal posisi AP dan lateral
b. Torakolumbal posisi AP + obliq
c. Torakolumbal lateral
d. Torakolumbal posisi oblique
e. Torakolumbal posisi AP
35. Seorang ibu berusia 35 tahun, mengalami nyeri pada sendi. Pada pemeriksaan lanjutan,
didapatkan Rheumatoid Factor positif.
Apakah factor risiko yang berpengaruh pada kejadian Rheumatoid Arthritis?
a. Berat badan
b. Merokok
c. Gizi
d. Genetik
e. Riwayat fraktur
36. Perempuan usia 70 tahun datang dengan keluhan nyeri pada punggung dialami sejak 3 tahun
terakhir memberat 2 bulan terakhir. Nyeri menjalar ke pantat sebelah kanan,  nyeri memberat ketika
berdiri dan berjalan. Kelemahan pada kedua tungkai bawah tidak ditemukan. Pasien tidak 
merasakan gangguan berkemih maupun buang air besar. Riwayat trauma disangkal oleh pasien.
Tiga bulan setelah pemberian obat pasien mengeluh nyeri punggung semakin memberat. Nerve
tension sign ditemukan, didapatkan pula adanya trofi muscle pada extremitas bawah kanan.
Pemeriksaaan penunjang yang mungkin menunjang diagnostic adalah
a. Xray
b. Ct Scan
c. USG
d. MRI
e. Facet Block
37. Fast fiber memiliki respon yang cepat dibandingkan dengan  tipe slow fiber. Ini dimungkinkan
karena?
a) serat otot yang kecil
b) banyak pembuluh darah
c) memiliki mithocondria yang banyak
d) mengandalkan enzim glikolisis untuk medapatkan ATP
e) banyak mioglobin
38. Seorang perempuan berusia 52 tahun, datang ke puskesmas dengan  keluhan nyeri lutut kanan,
driasakan memberat saat beraktivitas dan berkurang saat istirahat. Pasien mengeluhkan kaku pada
sendi lutut pada pagi hari. Riwayat trauma disangkal. Pada pemeriksaan fisis ditemukan krepitasi
pada lutut kanan dan dirasakan hangat pada palpasi serta atrofi pada otot Quadriceps. Pada
pemeriksaan laboratorium ditemukan LED 18mm/jam.
Modalitas terapi apakah yang paling tepat diberikan pada kasus ini?
a. Low level laser teraphy dan short wave diathermy
b. Short wave diathermy dan micro wave diathermy
c. Low level laser teraphy dan micro wave diathermy
d. Stimulasi elektrik dan low level laser teraphy 
e. Icing dan micro wave diathermy
39. Seorang laki laki 50 tahun datang ke puskesmas dengan keluhan nyeri punggung bawah terutama
dirasakan pada saat berubah dari posis duduk ke berdiri atau saat duduk lama. Keluhan ini dialami
sejak 3 minggu yang lalu. Tdk ada riwayat trauma, tdk ada riwayat penyakit metabolik, dan riwayat
demam disangkal.
Pada kasus ini, gejala apakah  yg dapat muncul dan menjadi perhatian khusus?
a. hamstring tightness
b. nyeri saat perubahan posisi
c. spasme otot paralumbal
d. defisit sensoris
e. nyeri punggung bawah terlokalisasi
40. anak laki laki 6 tahun diantar ke puskesma oleh ibunya  dengan keluhan kaki seperti O. Keluhan
ini dirasakan sejak 6 bulan yang lalu. Kelainan tulang ini disebabkan oleh tidak terjadinya maturasi
dari sel tulang.
Apa nama proses yg mengalami gangguan tsb?
a. deformitas
b. kalsifikasi
c. supurasi
d. osteogenesis
e. maturasi
41. Laki laki  umur 42 tahun datang dengan keluhan nyeri hebat pada lutut kanan, dialami penderita
sejak pagi tadi ketika bangun sholat subuh. Awalnya hanya terasa pegal2 sedikit hangat pada
perabaan. Riwayat trauma pda lutut kanan disangkal oleh penderita.
Cairan yang diperoleh pada tindakan dekompresi sendi pada lutut penderita berupa keruh
kehijauan. Jika dilakukan pemeriksaan mikroskopik cairan sendi tsb, maka akan ditemukan
a. Kristal MSU
b. Kristal CPPD
c. Kristal kolesterol
d. Kristal steroid
e. Kristal hidroksi apatit
42. Jika terjadi penurunan Ca dalam darah makan tubuh akan berusaha menaikkan kadar Ca dalam
darah dapat dilakukan dengan cara?
a) menaikkan pelepasan hormone PTH
b) menurunkan pelepasan calcitriol
c) menaikkan produksi calcitonin
d) memperbanyak ekskresi Ca melalui ginjal
e) meningkatkan deposit Ca di tulang
43. Seorang perempuan berusia 65 tahun datang ke puskesmas dengan keluhan nyeri pada
punggung jika berjalan. Pasien memiliki riwayat patah tulang 5 tahun  yang lalu. Pasien merasa nafsu
makan menurun. 
Riwayat kebiasaan makan pasien selalu juga jarang makan sayur dan susu. Saat ini pasien lebih
sering didalam kamar. Rendahnya asyupan nutrien tertentu dapat menjadi faktor risiko untuk
terjadinya penyakit tsb.
Apakah nutrisi tersebut?
a. Ca dan Na
b. Ca dan vitamin D
c. Ca dan vitamin A
d. Ca dan Vitamin E
e. Ca dan zinc
44. laki laki 25 tahun naik sepeda motor dan ditabrak mobil dari samping dan terkena betis kiri dan
terdapat luka terbuka. Sabtu minggu kemudian datang ke dokter dengan keluhan demam tinggi
dengan bengkak dan nyeri tungkai bawah kiri. Pada pemeriksaan lab didapatkan leukositosis lalu
dikirim ke bagian Radiologi untuk pemeriksaan radiologi.
Pada kasus diatas, hasil pemeriksaan radiologis ternyata ditemukan destruksi osteolitik 1/3 distal
dengan soft tissue swelling. Maka diagnosis radiologisnya adalah
a. Osteomielitis kronis
b. Osteomielitis akut
c. Avaskular nekrosis
d. Spondilitis TBC
e. Osteoporosis
45. seorang pria berusia 20 tahun menderita demam dan nyeri oto disebabkan oleh ISPA dan
mempunyai riwayat gastritis.
Apakah analgetik antipiretik yang efektif dan aman untuk pria tsb?
a. Asam asetil salisilat
b. Acetaminophen
c. Asam mefenamat
d. Kalium-diclofenac
e. Meloxicam
46. fungsi osteoblast membentuk matriks tulang juga dikontrol agar tidak berlebihan. 
Hal ini dimungkinkan oleh adanya Sclerotin yang dibentuk oleh sel:
a. Osteoblast sendiri (mekanisme autocrine)
b. Osteoclast
c. Osteosit
d. Osteoprogenitor cells
e. Stromal cells
47. anak bayi perempuan usia 3 tahun datang ke rumah sakit dengan orang tua mengeluh bengkok
pada tulang belakang. Keluhan ini ketika memandikannya, pada fisis ditemukan bahu kiri dan kanan
tidak simetris, ibunya juga mengeluhkan anaknya jadi membiru setiap menangis. Pada pemeriksaan
Xray ditemukan kegagalan segmentasi pada tulang belakang T7,8, dan 9.
Diagnosis yang mungkin pada balita ini adalah?
a. Congenital muscular torticollis
b. Congenital scoliosis
c. De novo scoliosis
d. spondilitis TB
e. adolescent idiopathic scoliosis
48. Pak Budi usia 58 tahun berjalan menuju ruang poliklinik puskesmas Melati dangan sedikit
pincang. Ia mengeluh kaki kirinya bengkak dan sangat nyeri sejak 2 hari lalu. Pemfis menunjukkan
sendi ibu jari kaki Pak Budi merah dan begkak. Hasil pemeriksaan lab menunjukkan kadar asam urat
Pak Budi 8,7 mg/dl.
Pernyataan berikut yg benar terkait makna biomedis skenario diatas adalah
a. asam urat dapat berperan sbg anti oksidan
b. asam urat merupakan senyawa toksik
c. kosentrasi normal dalam darah berkisar 9-12 mg/dl
d. asam urat kemungkinan bukan penyebab pembengkakan kaki pada kasus diatas
e. asam urat yg terbentuk berasal dari degradasi basa nitrogen sitosin, uracil, dan timing
49. seorang wanita 55 tahun datang ke poliklinik pratama dengan keluhan nyeri pada persendiaanya,
paa wajah tampak ruam discoid, dan hasil lab ditemukan proteinuria persisten dan ANA (+)
Apakah diagnosis penyakit yg paling mungkin ? PPT LABORATORY TEST MUSCULO HAL. 26
a. demam rematik
b. artritis rematik
c. SLE
d. Osteoporosis
e. OA
50. seorang ibu 35 tahun mengalami nyeri pada sendi. Pada pemeriksaan lanjutan, didapatkan RF
positif.
Apakah level pencegahan yang tepat pada kondisi tsb?
a. Primordial
b. Primer
c. Sekunder
d. Tersier
e. Spesific protection
51. Perempuan 65 tahun datang dengan keluhan nyeri punggung dialami sejak 6 bulan terakhir.
Pasien riwayat terjatuh dengan posisi terduduk di dapur rumah. Nyeri mereda ketika dalam posis
duduk. Pada pemeriksaan fisis ditemukan kelemahan pada waktu dorsoflexi pergelangan kaki kanan.
Hipostesi pada 1st web space kaki kanan. Pada pemeriksaan xRay ditemukan
Indikasi emergency operasi pada kasus ini adalah
a. Spondilolisis
b. Atrofi otot
c. Cauda equina syndrome
d. Unilateral radial pain
e. Herniated nucleus pulposus (HNP)
52. Seorang ibu mendapatkan keluhan dari anaknya, laki, berusia 9 tahun berupa pegal pada betis
kanan. Setelah diurut, keluhan tdk berkurang sehingga dibawa ke RS 2 weeks kemudian. Hasil
pemeriksaan ditemukan benjolan pada otot betis sebesar telur bebek. Hasil biopsi aspiras jarum
halus menunjukkan gambaran neoplasma malignan pada jaringan otot.
Berikut ini kemungkinan neoplasma yg dialami oleh kasus diatas
a. Leiomyoma (tumor jinak otot polos Ex uterus & Hepar)
b. Rhadomyosarcoma (maligna otot rangka)
c. Fibrosracome (tumor ganas fibroblast)
d. Myxosarcoma (tumor ganas langka tissue)
e. Rhabdomyoma (tumor jinak otot rangka)
53. hasil pemeriksaan  lab seorang pasien menunjukkan adanya peningkatan kadar AU di dalam
darahnya. Dokter kemudian meresepkan allopurinol untuk pasientsb.
Mekanisme kerja obat ini pada jalur degradasi purin adalah  PPT ARTRITIS GOUT HAL.43
a. Inhibitor sintesis de novo
b. Inhibitor salvage pathway
c. Inhibitor xanthine oxidase
d. Stimulator jalur pentosa fosfat
e. Stimulator nucleotida reduktase
54. pemerintah kota Makassar menetapkan beberapa area car free day agar warga dapat
berolahraga. Kegiatan olahraga ini dpaat memperkuat otot sekitar sendi. Menjaga kelenturan sendi
serta kepadatan tulang.
Apakah level pencegahan pada kegiatan tsb?
a. Primordial
b. Primer
c. Sekunder
d. Tersier
e. Spesific protection
55. Muhammad Zohri mengharumkan nama Indonesia dengan meraih emas pada kejuaaran dunia
atletik u-20. Ia memiliki catatan waktu 10,18 detik. Secara ilmiah kemampuan berlari yang sangat
cepat ini sangat didukung oleh faktor gen yg kemudian kemampuan otot menghasilkan ATP. 
Pernyataan brkt ini yg benar mewakili potensi kapasitas otot dominan yg dimiliki oleh Zohri adalah
PPT REVIEW BIOKIMIA HAL.17
a. Didominasi oleh serabut type 1
b. Kaya mioglobin
c. Diameter serat otot kecil
d. Banyak nengandung mitokondria
e. Kaya kadnungan glikogen
56. Laki laki  umur 42 tahun datang dengan keluhan nyeri hebat pada lutut kanan, dialami penderita
sejak pagi tadi ketika bangun sholat subuh. Awalnya hanya terasa pegal2 sedikit hangat pada
perabaan. Riwayat trauma pda lutut kanan disangkal oleh penderita.
 Jika pada pemfis ditemukan adanya balottement pada lutut kanan, maka masalah pada pasien
adalah 
a. Penebalan sinovium
b. Radang sendi lutut kanan
c. Penebalan meniscus
d. Bony enlargement
e. Efusi sendi lutut kanan
57. Berikut yg bukan keuntungan dari biopsi aspirasi jarum halus adalah   PPT TUMOR MUSCULO
HAL.69
a. Resiko perdarahan dan infeksi dapat dihindarkan
b. Spesimen dapat diambil dari beberapa bagian tumor
c. Mendapatkan gambaran histopatologi yg jelas
d. Hasil awal dapat dikethui secara tepat
e. Tidak perlu perawatan khusus
58. Untuk mengontrol agar osteoclast tidak meresorpsi matrix tulang secara berlebihan makan
osteoblast akan membentuk zat untuk menghambat fungsi osteoklast
Zat tsb adalah
a. Beta-catenin
b. TNF
c. Interleukin-1
d. Osteoprotegrin
e. IL-6
59. seorang perempuan 37 tahun datang ke dokter umum dengan keluhan nyeri dan bengkak jari jari
tangan yg disertai dengan kekakuan, sdh berlangsung 2 bulan. Pada pemfis didapatkan nyeri tekan
pada PIP II, III, dan MCP dextra dan sinistra. Pemeriksaan lab RF (+)
Parameter lab manakah yang masuk dalam kriteria ACR/EULAR 2010 untuk menegakkan diagnosis
kasus ini?
a. Penanda inflamasi:  LED dan jumlah leukosit, penanda serologi: RF dan ACPA
b. Penanda inflamasi:  LED dan CRP, penanda serologi: RF dan ACPA
c. Penanda inflamasi:  LED dan CRP, penanda serologi: RF dan ACPA
d. Penanda inflamasi:  LED dan CRP, penanda serologi: RF dan anti-CCP
e. Penanda inflamasi:  jumlah leukosit dan LED, penanda serologi: RF dan ACPA
60. Sorang laki laki 65 tahun dengan riwayat hipetensi, DM, dan penyakit sendi degenerative datang
ke klinik akibat pembengkakan  akut pada MTP-1 kiri. Pemfis menunjukkan sendi yg bengkak,
hangat, dan erythema. Tak ade riwayat trauma sebelumnya.
Apakah farmakoterapi pilihan pada kasus tsb?
a. Allopurinol (menurukan kadar asam urat)
b. Kalium diclofenac
c. Prednison (imunosupresan)(menekan sistem imun)
d. Metotraxate
e. Probenesid (urikosurik)(meningkatkan eksresi)
61. Seorang anak, 5 tahun, di diagnosis rickets
Hasil pemeriksaan laboratorium yang sesuai dengan diagnosis ini adalah:
a. Kalsitriol tinggi
b. Kalsium urin tinggi
c. Fosfat serum tinggi
d. Kalsium serum tinggi
e. Hormone paratiroid tinggi (PTH)

62. seorang anak perempuan usia 6 tahun datang dibawa orang tuanya dengan keluhan adanya
deformitas pada kedua kaki yang mulai tampak sejak anak tersebut mulai berjalan. Dari riwayat
tumbuh kembang pasien didapatkan pasien mulai berjalan pada usia 10 bulan dan pasien juga
memiliki riwayat berat badan obese. Dari pemeriksaan X-ray didapatkan adanya gambaran berikut: 
Diagnosa yang tepat sesuai dengan gambaran X-ray diatas adalah:
a. Blount’s disease
b. CTEV
c. DDH
d. Scheuermann’s disease

63. laki-laki umur 42 tahun, datang dengan keluhan nyeri hebat pada lutut kanan, dialami penderita
sejak pagi tadi ketika shalat subuh. Awalnya hanya terasa pegal2 sedikit hangat pada perabaan.
Riwayat trauma pada lutut kanan disangkal oleh penderita.
Tindakan farmakologis yang dapat dilakukan pada kasus di atas, berdasarkan data yang ada:
a. OAINS
b. Kolkisin
c. Allupurinol (fase intercritical)
d. Kolkisin + Allupurinol
e. OAINS + Kolkisin + Allupurinol

64. anak laki-laki usia 8 tahun datang dengan keluhan nyeri pada panggul kanan, dialami sejak
kurang lebih 1 minggu yang lalu, nyeri bertambah saat tungkai bawah kanan diluruskan. Pada
pemeriksaan laboratorium, LED hasilnya meningkat dan diluruskan. Pada pemeriksaan laboratorium,
LED hasilnya meningkat dan leukositosis. Pada pemeriksaan radiologi pada X-ray pelvis didapatkan
sklerotik pada panggul kanan.
Apakah kemungkinan diagnosis pada pasien ini?
a. Osteomyelitis akut
b. Osteomyelitis kronik
c. Osteomyelitis sub akut
d. Arthritis supuratif akut
e. Arthritis kronik

65. pada homeostatis dan remodeling tulang, osteoklast akan mereabsorbsi tulang yang sudah usang
atau mengalami kerusakan/injury sedangkan osteoblast akan membentuk matriks tulang yang baru
sebanyak yang direabsorbsi oleh osteoklast.
Kerjasama kedua sel ini melalui komunikasi sinyal antar sel (cell signaling) yang melalui jalur :
a. Wnt pathway
b. RANK pathway
c. Notch receptor pathway
d. Hedgehog pathway
e. JAK-STAT pathway

66. bayi laki-laki usia 6 bulan dengan posisi kepala berputar ke sebelah kiri. Tampak otot
sternocleidomastoid mastoideus sebelah kanan mengeras. Riwayat persalinan normal. Defisit
neurologis tidak ditemukan.
Tatalaksana yang mungkin pada pasien ini adalah:
a. Stretching secara pasif
b. Pemberian antibiotic
c. Operasi
d. Pemberian vitamin C
e. Pemberian steroid

67. laki 65 tahun datang dengan keluhan nyeri pada leher yang dialami sejak 2 tahun terakhir, nyeri
memberat satu bulan terakhir, nyeri terasa menjalar ke lengan kanan bawah hingga ke jari tangan,
riwayat trauma diangkat. Pada pemeriksaan fisis ditemukan kelemahan pada waktu melakukan
gerakan ekstensi wrist, dan hipotesa pada ibu jari tangan kanan. Tidak ditemukan gangguan berjalan
pada pasien ini maupun gangguan saat berkemih dan buang air besar.
Diagnosis yang memungkinkan pada pasien ini adalah:
a. Cervical Radiculopathy
b. Cervical myelopathy
c. Thoracic disc disease
d.  Lumbar disc disease
e. Conus medullaris syndrome

68.pasien laki-laki 34 tahun datang dengan keluhan rasa yang tidak nyaman pada punggung bawah,
disertai rasa cepat letih, tanpa disertai demam. Pada pemeriksaan fisik tidak ditemukan benjolan
pada punggung bawah. Pada pemeriksaan laboratorium ditemukan peningkatan tanpa leukositosis.
Pada pemeriksaan radiologi X-ray lumbal terjadi destruksi tulang Lumbal I.
Apa kemungkinan diagnosis pasien ini?
a. Spondylitis TB
b. Spondylitis pyogenik
c. Osteoporosis
d. Spondylolysis
e. Spondylolistesis

69. yang TIDAK TERMASUK tarsal bones adalah:


a. Talus
b. Calcaneus
c. Cuboid 
d. Capitatatum
e. Cuneiform

70. pada osteoarthritis kelainan primer terjadi pada:


a. Kapsul sendi
b. Bongkol sendi
c. Kartilago sendi
d. Ligament sendi
e. Synovium

71. .seorang anak perempuan usia 6 tahun datang dibawa orang tuanya dengan keluhan adanya
deformitas pada kedua kaki yang mulai tampak sejak anak tersebut mulai berjalan. Dari riwayat
tumbuh kembang pasien didapatkan pasien mulai berjalan pada usia 10 bulan dan pasien juga
memiliki riwayat berat badan obese. Dari pemeriksaan X-ray didapatkan adanya gambaran berikut: 
Kelainan yang dapat dijumpai pada penyakit diatas yaitu adanya:
a. Tibia valgus
b. Tibia varus

73. laki-laki usia 25 tahun datang ke unit gawat darurat dengan keluhan nyeri pada tungkai bawah
kanan setelah mengalami kecelakaan lalu lintas, pada pemeriksaan fisis ditemukan adanya luka
laserasi ukuran 10x 12 cm pada region cruris kanan dengan tampak tulang ekspose dan hematom,
pada pemeriksaan x-ray cruris kanan, ditemukan adanya fraktur tibia fibula 1/3 tengah tipe
transverse.
Pada fraktur terbuka, bagaimana pathogenesis terjadinya osteomyelitis? PPT INFEKSI MUSKULO
HAL.22
a. Fraktur tibia
b. Fraktur fibula
c. Kerusakan jaringan dan pembuluh darah
d. Focus primer bakteri
e. Besar luka

74. sekelompok ibu PKK mengadakan skrining densitas tulang pada seluruh anggotanya, untuk
mengetahui ada tidaknya osteoporosis.
Apakah level pencegahan pada kegiatan tersebut?
a. Specific protection
b. Primer
c. Sekunder
d. Tersier
e. Prompt treatment

75. seorang perempuan berusia 52 tahun, datang ke puskesmas dengan keluhan nyeri lutut kanan,
dirasakan memberat saat beraktivitas dan berkurang saat istirahat. Pasien mengelihkan kaku pada
sendi lutut pada pagi hari. Riwayat trauma disangkal. Pada pemeriksaan fisis ditemukan krepitasi
pada lutut kanan. Pada pemeriksaan laboratorium ditemukan LED 18 mm/jam.
Apakah diagnosis yang paling tepat pada kasus ini?
a. Rheumatoid Arthritis
b. Osteoarthritis
c. Gout Arthritis
d. Gonathrosis
e. Bursitis Genu

76. seorang perempuan berusia 52 tahun, datang ke puskesmas dengan keluhan nyeri lutut kanan,
dirasakan memberat saat beraktivitas dan berkurang saat istirahat. Pasien mengeluhkan kaku pada
sendi lutut pada pagi hari. Riwayat trauma disangkal. Pada pemeriksaan fisis ditemukan krepitasi
pada lutut kanan. Pada pemeriksaan laboratorium ditemukan LED 18 mm/jam
Edukasi apakah yang dapat kita sarankan pada pasien diatas?
a. Menggunakan toilet jongkok
b. Naik tangga dengan lutut kanan terlebih dahulu
c. Naik tangga dengan lutut kiri terlebih dahulu
d. Jogging dipagi hari
e. Duduk bersila
77. seorang pasien perempuan berusia 37 tahun datang dengan keluhan nyeri pada sendi-sendi jari
kedua tangannya. Nyeri terutama dirasakan pada pagi hari disertai keluhan terasa kaku yang
lamanya lebih dari 30 menit. Pemeriksaan awal yang harusnya diberikan sebelum dirujuk ke rumah
sakit adalah:
a. Colchisine
b. Metotrexate
c. Dexametason
d. Sulfasatzine
e. Meloxicam

78. bayi laki-laki usia 6 bulan datang dengan posisi kepala terputar ke sebelah kiri. Tampak otot-otot
sternocleidomastoid mastoideus sebelah kanan mengeras. Riwayat persalinan normal. Defisit
neurologis tidak ditemukan.
Patofisiologis yang diduga sebagai penyebab terjadinya kasus ini adalah: PPT SPINE DISORDER HAL.
46
a. Compartment syndrome dari otot sternocleidomastoideus
b. Instabilitas C1-C2
c. Rupture sternocleidomastoideus
d. Abses colli
e. Tumor tyroid

79. seorang laki-laki 23 tahun dengan keluhan punggung kaki kanannya kurang rasa setelah trauma
beberapa waktu lalu. Setelah dokter melakukan pemeriksaan, didapatkan adanya kekurangan
sensasi pada sisi lateral dorsum pedis.
Saraf manakah dibawah ini yang paling mungkin mengalami kelainan?
a. N. plantaris lateralis
b. N. saphenus
c. N. peroneus profunda
d. N. peroneus superficialis 
e. N. suralis

80. perempuan usia 70 tahun datang dengan keluhan pada punggung yang dialami sejak 3 tahun
terakhir memberat 2 bulan terakhir. Nyeri menjalar ke pantat sebelah kanan, nyeri memberat ketika
berdiri dan berjalan. Kelemahan pada kedua tungkai bawah tidak ditemukan. Pasien tidak
merasakan gangguan berkemih maupun buang air besar. Riwayat trauma disangkal oleh pasien.
Kelainan yang mungkin pada pasien ini adalah:
a. Lumbal Spinal Stenosis
b. Cervical Radiculopathy
c. Cervical Myelopathy
d. Perivertebral Muscle Spasme
e. Cauda Equina Syndrome

81. perempuan 37 thn datang dengan keluhan nyeri dan bengkak pada jari tangan yang disertai
dengan kekakuan sudah berlangsung sekitar 2 bulan. Pada pemeriksaan fisis didapatkan nyeri tekan
pada PIP II,III dan MCP dextra dan sinistra. Pemeriksaan lab : factor rheumatoid (+), LED 80/100.
Specimen apakah yang dibutuh untuk pemeriksaan lab. Pada kasus ini ?
a. darah lengkap urin sewaktu
b. darah lengkap cairan synovial
c. darah lengkap serum
d. urin sewaktu serum
e. serum cairan synovial
82. tumor ganas otot skelet yang terbanyak ditemukan pada anak-anak
a. fetal rhabdomyoma
b. genital rhabdomyoma
c. embryonal rhabdomyosarcoma
d. alveolar rhabdomyosarcoma
e. pleomorphic rhabdomyosarcoma
83.wanita 70 tahun didiagnosis osteoporis. Wanita ini sudah menopause. penyataan berikut ini yang
benar terkait pengaruh menopause terhadap aktifitas komponen sekitar tulang sehingga berpotensi
menyebabkan kondisi tersebut adalah 
a. osteosit mudah apoptosis
b. remodeling tulang meningkat
c. apoptosis osteoblast menurun
d. apoptosis osteoklas meningkat
e. resorpsi tulang menurun.
84. laki umur 42 thn, datang dengan keluhan nyeri hebat pada lutut kanan, dialami penderita sejak
pagi tadi ketika bangun sholat subuh. Awalnya hanya terasa pegal sedikit hangat pada perabaan.
Riwayat trauma pada lutut kanan disangkal oleh penderita. Tindakan yang bersifat non farmakologis
pada kasus ini yang dapat dilakukan ialah 
a. kompres air hangat
b. kompres air dingin
c. tens
d. penyinaran uv
e. interferensi
85. wanita 17 tahun mengalami haid pertamanya kasihan, mengeluhan mules di daerah perut sakit
kepala dan demam sejak 2 hari yang lalu. Pasien berobat ke puskesmas dan diberikan obat
paracetamol. Bagaimana mekanisme kerja obat tersebut ?
a. menghambat enzim ciclo oxigenase
b. mnghambat enzim lipoksigenase
c. menghambat enzim fosfolipase
d. mengahmbat enzim lipase
e. menghambat enzim na+/k + atp ase
86. kontraksi otot memerlukan suplai atp yang besar . pernyataan berikut ini yang benar mengenai
jalur metabolism yan digunakan untuk memproduksi atp ketika seseorang atlit sprint berlari kencang
pada suatu perlombaan adalah :
a. jalur aeobik lebih dominan
b. glikogenesis sangat aktif
c. ketogenesis sangat aktif
d. substrat metabolic utama adalah asam lemak
e. peningkatan konversi fosfokreatin menjadi keratin
87.berikut ini adalah yang benar mengalami tumor primer pada musculoskeletal  
PPT TUMOR MUSCULO HAL.16
a. tumor banyak pada puncak pertama (decade pertama kehidupan) adalah osteosarcoma
b. tumor banyak pada puncak pertama (decade kedua kehidupan) adalah ewing sarcoma
c. tumor banyak pada puncak kedua (diatas 40 tahun) adalah chondrosarcoma
d. tumor terbanyak pada puncak kedua (diatas 60 tahun) adalah osteosarcoma
e. tumor terbanyak pada puncak pertama (decade kedua kehidupan) adalah osteosarcoma
88. seorang anak perempuan berusia 6 tahun datang kepuskesmas diantar oleh bundanya dengan
keluhan nyeri tulangdan lemah. Tampak pasien tidak memiliki tinggi sesuai dengan umurnya dan
terlihat kelainan pada tungkaibawah. Pada pemeriksaan antropometri didapatkan tb 93cm .pasien
tersebut dicurigai mengalami defisiensi nutrient tertentu. Apakah nutrient tersebut ?
a. vit a
b. b
c. c
d. d
e. e
89. seorang wanita usia 15 thn (Rachel chu) datang dengan keluhan bengkok pada punggung. Pada
pemfis ditemukan posisi bahu dan pinggang tdk simetris kiri dan kanan, adam forward bend test (+).
Diagnosis yang mungkin pada kasus ini
a. lumbar disc herniation
b. cervical myelopathy
c. cauda equine syndrome
d. spondylitis tb
e. adolescent idiopathic scoliosis
90. seorang laki 25 thn (nick young), jatuh dengan cedera daerah axilla dirawat pada ruang gawat
darurat dengan dislokasi pada art. Humeri dextra. Struktur otot manakah yang tidak mengatur
kestabilan art. Pada kasus ini ?
a. m. supraspinatus
b. m. infraspinatus
c. m. subscapularis
d. m. teres major
e. m. teres minor

91. Laki – laki umur 42 tahun, datang dengan keluhan nyeri hebat pada lutut kanan, dialami
penderita sejak pagi tadi ketika bangun sholat subuh. Awalnya hanya terasa pegal – pegal
sedikit hangat pada perabaan. Riwayat trauma pada lutut kanan disangkal oleh penderita.
Dalam penegakan diagnose pada scenario di atas, salah satu informasi diperlukan :
a. Pemeriksaan fungsi ginjal
b. Adanya riwayat penyakit yang sama dalam keluarga
c. Ada riwayat nyeri hebat yang terjadi secara tiba – tiba pada jempo kaki
d. Saat ini kadar asam urat darah yang tinggi
e. Adanya lupus
92. Seorang perempuan berusia 52 tahun, datang ke puskesmas dengan keluhan nyeri lutut
kanan, dirasakan memberat saat beraktivitas dan berkurang saat istrahat. Pasien
mengeluhkan kaku pada sendi lutut pada pagi hari. Riwayat trauma disangkal. Spesial test
apakah yang dilakukan pada kasus di atas?
a. Anterior drawer test, patellar grinding test, Mc. Murray test
b. Patellar grinding test, empty can test, Mc. Murray test
c. Schober test, anterior drawer test, patellar grinding test
d. Tinner test, patellar grinding test, ott test
e. Player test, lachman test, Thomas test

93.

Seorang ibu datang membawa bayi berusia 1 bulan ke praktek anda dengan keluhan utama
adanya deformitas pada kaki kirinya seperti tampak pada gambar di atas. Deformitas yang
tampak pada kasus di atas adalah :  PPT MALFORMASI CONGENITAL HAL. 24
a. Cavus
b. Adductus
c. Varus
d. Equrus
e. Valgus
94. Saat berolahraga atau melakukan aktivitas fisik lain yang intens terjadi kooperasi jalur
metabolism antar-organ di dalam tubuh. Laktat yang diproduksi otot sel – sel otot dapat
diambil dan dikonversi menjadi piruvat untuk dijadikan sumber energi aerobik  oleh organ :
PPT REVIEW BIOKIMIA HAL.12
a. Otak dan paru – paru
b. Ginjal dan limpa
c. Hati dan jantung
d. Lambung dan ginjal
e. Kelenjar limfoid perifer dan sumsum tulang
95. Seorang perempuan berusia 49 tahun datang ke puskesmas dengan keluhan bengkak dan
nyeri pada persendian lutut. Pada pemeriksaan antropometri didapatkan BB 75 kg, dan TB
157 cm. pemeriksaan laboratorium menunjukkan kadar asam urat darah diperoleh 3,7 g/dl.
Pasien saat ini telah mendapatkan obat anti inflamasi. Pengaturan nutrisi/diet juga akan
diberikan. Apa tujuan pengaturan nutrisi/diet tersebut?
a. Penurunan tekanan darah
b. Penurunan asupan protein
c. Penurunan asupan ureum
d. Penurunan berat badan
e. Penurunan asupan xantin
96. Seorang ibu membawa anak perempuannya usia 9 tahun dengan bengkak pada betis kanan
yang semakin lama semakin membesar sejak 3 bulan ini. Riwayat jatuh atau cedera
disangkal. 1 bulan ini anak tersebut mengeluhkan demam hilang timbul dan nyeri sehingga
enggan berjalan. Pasien kemudian dilakukan pemeriksaan radiologi,  bisopsi dan bone
marrow aspiration kemudian ditegakkan diagnosis Ewing Sarcoma. Staging Enneking pada
kasus Ewing Sarcoma di atas adalah:
a. Stage IA
b. Stage IB
c. Stage IIA
d. Stage IIB
e. Stage III
97. Pada wanita menopause, osteoporosis terjadi karena fungsi osteoklast yang sangat
berlebihan dibandingkan fungsi osteoblast. Hal ini  terjadi karena pada menopause kadar
ekstrogen sangat menurun yang mengakibatkan meningkatnya produksi :
a. IL-1, IL-6, TNF
b. Insulin
c. Beta-catenin
d. Osteoprotegrin
e. Sclerostin
98. Seorang laki – laki berusia 50 tahun datang ke puskesmas dengan keluhan nyeri punggung
bawah yang terutama dirasakan pada saat berubah posisi dari duduk ke berdiri atau saat
duduk lama. Keluhan ini dialami sejak 3 minggu yang lalu. Tidak ada riwayat trauma, tidak
ada riwayat penyakit metabolik dan riwayat demam disangkal. Bagaimanakah posisi tubuh
berikut ini yang memiliki tekanan instradiskal tertinggi saat kita mengedukasi pasien :
a. Berbaring terlengtang
b. Berbaring tengkurap
c. Duduk
d. Duduk bungkuk
e. Berdiri bungkuk
99. Laki – laki 25 tahun naik sepeda motor ditabrak mobil dari samping dan terkena betis kiri dan
terdapat luka terbuka. Satu minggu kemudian datang ke dokter dengan keluhan demam
tinggi dengan bengkak dan nyeri tungkai bawah kiri. Pada pemeriksaan laboratorium
didapatkan leukositosis lalu dikirim ke bagian radiologi untuk pemeriksaan radiologi.
Pemeriksaan posisi foto radiologi apa yang diminta ?
a. Foto cruris kiri posisi obliq
b. Foto cruris kiri posisi lateral
c. Foto cruris kiri posisi AP
d. Foto cruris kiri posisi AP+Obliq
e. Foto cruris kiri posisi AP+Lateral
100. Seorang laki – laki berusia 50 tahun datang ke puskesmas dengan keluhan nyeri
punggung bawah yang terutama dirasakan pada saat berubah posisi dari duduk ke berdiri
atau saat duduk lama. Keluhan ini dialami sejak 3 minggu lalu. Tidak ada riwayat trauma,
tidak ada riwayat penyakit metabolic dan riwayat demam disangkal. Pada kasus ini, factor
apakah paling sering menyebabkan nyeri punggung bawah ?
a. Biomekanika postur patologis
b. Kompresi tumor extra moduler
c. Trauma cauda equine
d. Infeksi tuberculosis
e. Canal sternosis
101. Kekuatan kontraksi otot dapat ditingkatkan dengan cara di bawah ini ?
a. Menaikkan voltage melebihi voltage maksimalnya
b. Meningkatkan jumlah motor unit yang terlibat
c. Menambah suplai glukosa
d. Menambah jumlah glikogen
e. Menambah frekuensi stimulasi
102. Wanita 67 tahun datang dengan keluhan bengkak pada tangan kanannya sejak 2
minggu yang lalu, riwayat terkena tusukan jarum pada jari telunjuknya. Apa tanda khas yang
dapat ditemukan untuk mendiagnosis terjadinya tenosynovitis? PPT INFEKSI MUSCULO
HAL.73
a. Thomas sign Fleksor Hip
b. Kanafel sign (inflamasi gembung)
c. Gibbus (TB)
d. Lachman sign (ACL)
e. Dequervan sign (teno sinovytis : iritasi tendon)
103. Perempuan 65 tahun datang dengan keluhan nyeri punggung dialami sejak 6 bulan
terakhir. Pasien riwayat jatuh dengan posisi duduk di dapur rumah. Nyeri mereda ketika
dalam posisi duduk. Pada pemeriksaan fisis ditemukan kelemahan pada waktu dorsoflexi
pergelangan kaki kanan. Hipostesi pada 1 web space kaki kanan. Pada pemeriksaan xray
st

ditemukan. Dari pemeriksaan fisis pasien tersebut kemungkinan saraf yang terjepit adalah :
a. Nerve Root L2
b. Nerve Root L3
c. Nerve Root L4
d. Nerve Root L5
e. Nerve Root S1
104. Seorang ibu, yang berusia 45 tahun, memiliki indeks massa tubuh 32(Obesistas) dan
saat ini tidak memiliki keluhan secara klinis. Ibu tersebut kemudian dianjurkan untuk
mengurangi berat badannya untuk mencegah osteathritis pada sendi lutu atau panggulnya.
Apakah level pencegahan terhadap penyakit osteoarthritis yang diberikan kepada ibu
tersebut?
a. Primordial
b. Primer
c. Sekunder
d. Tersier
e. Deteksi dini
105. Laki – laki umur 42 tahun, datang dengan keluhan nyeri hebat pada lutut kanan,
dialami penderita sejak pagi tadi ketika bangun sholat subuh. Awalnya hanya terasa pegal –
pegal sedikit hangat pada perabaan. Riwayat trauma pada lutut kanan disangkal oleh
penderita. Jika kondisi akut pada kasus di atas sudah teratasi, maka pengobatan
farmakologis yang dapat diberikan adalah :
a. OAINS
b. Kolkisine
c. Alluprinol
d. Kolkisine+alluprinol
e. OAINS+Kolkisine+Alluprinol
106. Tes laboratorium diagnostic yang dapat menunjang diagnosis penderita gout
adalah :
a. Kadar glukosa dan cairan sendi
b. Kadar cholesterol dan cairan sendi
c. Kadar keratinin dan cairan sendi
d. Kadar ureum dan cairan sendi
e. Kadar asam urat dan cairan sendi
107. Wanita usia 45 tahun datang ke klinik dengan keluhan bengkak dan nyeri pada ibu
jari tangan kanan. Riwayat sebelumnya terkena tusukan jarum 2 minggu lalu. Pada
pemeriksaan fisis ditemukan adanya pembengkakan pada tangan kanan disertai rasa nyeri
pada saat akan diluruskan. Pada kasus seperti ini, yang tidak termasuk patofisiologi yang
dapat terjadi sehingga dapat terjadi tenosynovitis adalah : PPT INFEKSI MUSCULO HAL.71
a. Edema
b. Hipertropi sarung synovial
c. Sarung synovial inflamasi
d. Pengingkatan produksi cairan synovial
e. Hematom

108. Laki – laki umur 42 tahun, datang dengan keluhan nyeri hebat pada lutut kanan,
dialami penderita sejak pagi tadi ketika bangun sholat subuh. Awalnya hanya terasa pegal –
pegal sedikit hangat pada perabaan. Riwayat trauma pada lutut kanan disangkal oleh
penderita. Jika hasil pemeriksaan kadar asam urat darah saat ini  adalah 5,2 mg/dl, ada
riwayat keluhan nyeri yang terjadi secara tiba – tiba namun pada sendi tallo-kluralis kiri
sekitar 6 bulan lalu. Maka diagnosis pada scenario adalah :
a. Osteorthtritis genu kanan
b. Artritis septik
c. Artritis gout intermitten
d. Artritis gout akut
e. Hiperukemia asamtomatik
109. Seorang perempuan berusia 25 tahun datang ke puskesmas dengan keluhan jalan
tidak bias lurus. Hal ini dikarenakan tulang kaki lebih pipih. Kelainan ini disebabkan oleh
defisiensi vitamin D sehingga absorbs kalsium terganggu pada tulang. Apa penyebab
defisiensi tersebut ?
a. Gangguan ekskresi
b. Gangguan sekresi
c. Gangguan absorbs
d. Gangguan sintesa
e. Gangguan reabsorbsi
110. Seorang laki – laki 48 tahun datang ke poliklinik dengan keluhan nyeri punggung
sejak 5 bulan terakhir. Nyeri kemudian menjalar pada kedua tungkai bawah, sehingga
tungkai sering terasa kesemutan dan panas seperti terbakar. Saraf manakah di bawah ini
yang paling mungkin mengalami kelainan ?
a. N. femoralis
b. N. obturatorius
c. N. ischiadicus
d. N. genitofemoralis
e. N. iloinguinalis
111. Seorang laki – laki 45 tahun datang ke klinik dengan keluhan nyeri dan bengkak pada
ibu jari kaki kanan yang dialami 5 jam yang lalu. Riwayat nyeri berulang dialami sejak
setahun lalu, terutama pada malam hari dan jika cuaca dingin. Pemeriksaan fisk status lokalis
MTP I dekstra : dolor(+), kalor(+), bulge sign(-), balloon sign(-). Tes laboratorium yang dapat
dilakukan untuk memantau terjadinya komplikasi adalah :
a. Ureum-kreatinin
b. Tes darah rutin
c. Urin rutin
d. GOT/GPT
e. GDS
112. Bayi laki – laki usia 6 bulan databg dengan posisi kepala terputar ke sebelah kiri.
Tampak otot sternocleidomastoid mastoideus sebelah kanan mengeras. Riwayat persalinan
normal. Deficit neurologis tidak ditemukan. Kelainan lain yang mungkin bias terjadi pada
pasien ini adalah :
a. Flat foot
b. Ankle sprain
c. Metatarsus adductus
d. Abses coli
e. Tumor tyroid
113. Seorang perempuan berusia 50 tahun  datang ke puskesmas dengan keluhan nyeri
pada panggul. Pemeriksaan antropometri menunjukkan BB 70 kg, dan TB 153 cm.
manajemen nutrisi merupakan salah satu pengobatan yang diberikan pada pasien ini.
Apakah tipe diet yang diberikan ?
a. Kalori terkontrol 🡪 untuk maintain berat badan ideal
b. Rendah lemak 🡪 
c. Rendah kalori 🡪 untuk turunkan berat badan
d. Tinggi serat
e. Rendah karbohidrat
114. Seorang perempuan berusia 50 tahun datang ke puskesmas dengan keluhan patah
tulang di tangan setelah terjatuh dari kendaraan. Massa tulang pada wanita menurun pada
usia tertentu. Berapakah usia tersebut ?
a. 28 tahun
b. 30 tahun
c. 35 tahun
d. 40 tahun
e. 50 tahun

115.
Seorang ibu datang membawa bayi berusia 1 bulan ke praktek anda dengan keluhan utama
adanya deformitas pada kakinya seperti tampak pada gambar tersebut. Penyakit tersebut
dikenal sebagai : 
PPT MALFORMASI CONGENITAL HAL.24
a. Flat foot
b. Congental metatarsus adductus
c. Congental talpes equino varus
d. Balount disease
e. Arthrogryposis
116. Seorang laki – laki berusia 46 tahun datang ke puskesmas dengan keluhan terdapat
benjolan pada beberapa sendi tangan dan kaki. Pemeriksaan antropometri menunjukkan BB
55 kg dan TB 160 cm. pemeriksaan kadar asam urat dalam darah diperoleh 10.1 g/dl. Pasien
akan dilakukan pengaturan nutrisi. Bagaimanakah tata laksanana ? PPT ASPEK NUTRISI
KELAINAN MUSCULO HAL. 28
a. Batasi asupan cairan
b. Batasi konsumsi diet purin hingga 150 mg/hari
c. Kurangi mengkonsumsi makanan sumber karbohidrat
d. Konsumsi lemak normal
e. Batasu konsumsi diet purin hingga 150 g/hari
117. Seorang laki – laki, berusia 45 tahun mengeluh telinga berdengung sejak 3 hari yang
lalu setelah mengkonsumsi obat analgetik untuk nyeri bahu kanan. Pasien tidak mempunyai
riwayat gastritis. Apakah obat yang kemungkinan dikonsumsi pasien ?
a. Asam mefenamat
b. Ketoprofen
c. Ibuprofen
d. Celecoxib
e. Asam asetil salsilat
118. Seorang laki – laki 20 tahun mengeluh bahu kanan tidak dapat digerakkan setelah
terjatuh data mengendarai motor. Hasil pemeriksaan radiologi ditemukan fraktur pada
collum chirurgicum humeri. Struktur manakah di bawah ini yang paling mungkin mengalami
kelainan pada kasus di atas
a. N. axillaris
b. N. radialis
c. N. ulnaris
d. N. medianus
e. N. musculocutaneus
119. Seorang laki – laki 54 tahun, datang ke puskesmas dengan keluhan utama nyeri pada
siku kiri simetris yang sudah berlangsung sejak 1 tahun lalu, awalnya dimulai dengan nyeri
pada sendi – sendi jarinya dan pergelangan tangan. Keluhan dirasakan terutama di pagi hari.
Tidak ada demam, riwayat tidak pernah jatuh. Apakah tes laboratorium serologi yang dapat
diminta untuk mendukung diagnosis pasien tersebut ?
a. RF
b. LE test
c. Tes ANA
d. ASTO
e. Anti DNA
120. Seorang perempuan 37 tahun datang ke klinik dengan keluha nyeri dan bengkak
pada jari tangan yang disertai dengan kekakuan. Keluhan sudah berlangsung sekitar 2 bulan
lalu dan semakin memberat. Pada pemeriksaan fisis didapatkan nyeri tekan pada PIP II, III
dan MCP dextra dan sinistra. Pemeriksaan lab. LED 80/100. Pernyataan yang manakah di
bawah ini yang paling sesuai/berhubungan dengan kasus di atas?
a. RF positif menunjukkan adanya autoantibodi terhadap IgG
b. Nyeri dan bengkak disebabkan penumpukan Kristal monosodium urat
c. Proteinuria dapat dijumpai keran adanya kerusakan ginjal
d. Anemia dapat terjadi karena proses hemolysis
e. LED berada dalam batas normal

Anda mungkin juga menyukai